Tải bản đầy đủ (.docx) (140 trang)

BÀI TOÁN CHIA HẾT TRONG SỐ HỌC

Bạn đang xem bản rút gọn của tài liệu. Xem và tải ngay bản đầy đủ của tài liệu tại đây (543.98 KB, 140 trang )

<span class='text_page_counter'>(1)</span><div class='page_container' data-page=1></div>
<span class='text_page_counter'>(2)</span><div class='page_container' data-page=2>

Mục lục



<b>BÀI TOÁN CHIA HẾT TRONG SỐ HỌC...4</b>


<b>MỞ ĐẦU...4</b>


<b>NỘI DUNG...4</b>


<b>A. MỘT SỐ VẤN ĐỀ VỀ ĐỒNG DƯ CHIA HẾT...4</b>


<b>B. MỘT SỐ PHƯƠNG PHÁP CƠ BẢN CHỨNG MINH BÀI TOÁN </b>
<b>CHIA HẾT...8</b>


<b>C. MỘT SỐ BÀI TOÁN NÂNG CAO...15</b>


<b>D. BÀI TẬP TỰ LUYỆN...26</b>


<b>TÀI LIỆU THAM KHẢO...28</b>


<b>XÂY DỰNG VÀ SỬ DỤNG HỆ THỐNG CÂU HỎI, BÀI TẬP ĐỂ DẠY </b>
<b>HỌC NỘI DUNG CƠ SỞ VẬT CHẤT VÀ CƠ CHẾ DI TRUYỀN Ở CẤP ĐỘ</b>
<b>PHÂN TỬ...29</b>


<b>I. CƠ SỞ XÂY DỰNG CHUYÊN ĐỀ...29</b>


<b>1.1. Cơ sở lý luận...29</b>


<b>1.2. Cơ sở thực tiễn...30</b>


<b>II. NỘI DUNG CHUYÊN ĐỀ...30</b>



<b>2.1. Tổng quan về câu hỏi, bài tập...30</b>


<b>2.2. Logic nội dung kiến thức phần cơ sở vật chất và cơ chế di truyền ở </b>
<b>cấp độ phân tử – Sinh học 9, THCS...34</b>


<b>2.3. Hệ thống câu hỏi, bài tập để dạy học nội dung cơ sở vật chất và cơ </b>
<b>chế di truyền ở cấp độ phân tử...34</b>


<b>III. KẾT LUẬN...49</b>


<b>TÀI LIỆU THAM KHẢO...50</b>


</div>
<span class='text_page_counter'>(3)</span><div class='page_container' data-page=3>

<b>VAI TRÒ CỦA LÃNH TỤ NGUYỄN ÁI QUỐC - HỒ CHÍ MINH ĐỐI VỚI </b>


<b>THẮNG LỢI CỦA CÁCH MẠNG THÁNG TÁM NĂM 1945...51</b>


<b>I. ĐẶT VẤN ĐỀ...51</b>


<b>II. NỘI DUNG...51</b>


<b>1. Tìm ra con đường cứu nước, sáng lập Đảng Cợng Sản Việt Nam...51</b>


<b>2. Xác định đường lới giải phóng dân tộc và chủ trương khởi nghĩa vũ </b>
<b>trang...52</b>


<b>3. Sáng lập mặt trận Việt Minh và chuẩn bị lực lượng chính trị...54</b>


<b>4. Chuẩn bị lực lượng vũ trang và xây dựng căn cứ địa cách mạng...55</b>


<b>5. Xác định đúng thời cơ, kiên quyết phát động tổng khởi nghĩa...56</b>



<b>6. Sáng lập Nhà nước Việt Nam Dân chủ Cợng hịa...58</b>


<b>III. MỘT SỐ DẠNG CÂU HỎI...59</b>


<b>CÁC DẠNG CÂU HỎI LÝ THUYẾT CHỦ YẾU TRONG ĐỀ THI HỌC </b>
<b>SINH GIỎI LỚP 9 MÔN ĐỊA LÍ...63</b>


<b>A. MỞ ĐẦU...63</b>


<b>B. NỘI DUNG...63</b>


<b>I. Dạng câu hỏi trình bày...63</b>


<b>II. Dạng câu hỏi chứng minh...68</b>


<b>III. Dạng câu hỏi giải thích...71</b>


<b>IV. Dạng câu hỏi so sánh...76</b>


<b>C. KẾT LUẬN...81</b>


<b>Examining the plausibility of Extensive Reading as an approach to learning </b>
<b>English at a secondary school context...82</b>


<b>Part 1: Introduction...82</b>


<b>Reasons for choosing the study...82</b>


<b>Aims and significance of the study...82</b>



<b>Subjects of the study...83</b>


<b>Limits of the study...83</b>


<b>Methodology...83</b>


<b>Research time...83</b>


</div>
<span class='text_page_counter'>(4)</span><div class='page_container' data-page=4>

<b>1. Extensive reading: A definition...83</b>


<b>2. The benefits of extensive reading...83</b>


<b>3. Principles of an extensive reading programme...85</b>


<b>4. A model of an extensive reading club in Vinh Phuc senior secondary </b>
<b>school for the gifted...86</b>


<b>Part 3. Conclusion...89</b>


<b>REFERENCES...90</b>


<b>Appendix 1: Sample of a reading text. Genre: Literature. Grade 5...91</b>


<i><b>The Devoted Widow...91</b></i>


<b>GIẢI BÀI TẬP CO2...92</b>


<b>PHƯƠNG PHÁP GIẢI BÀI TỐN DỊNG ĐIỆN KHƠNG ĐỔI...100</b>



<b>Phương pháp giải tốn...100</b>


<b>Bài tập tự lụn...105</b>


<b>RÈN LUYỆN KĨ NĂNG LÀM KIỂU BÀI NGHỊ LUẬN XÃ HỘI CHO HỌC </b>
<b>SINH GIỎI MÔN NGỮ VĂN THCS...107</b>


<b>A. ĐẶT VẤN ĐỀ...107</b>


<b>B. PHẦN NỘI DUNG...108</b>


<b>I. NHẬN DIỆN ĐẶC ĐIỂM KIỂU BÀI NGHỊ LUẬN XÃ HỘI...108</b>


<b>II. KĨ NĂNG LÀM KIỂU BÀI NGHỊ LUẬN XÃ HỘI ĐÚNG, HAY VÀ </b>
<b>GIÀU CHẤT VĂN...109</b>


<b>III. HƯỚNG DẪN HỌC SINH ÔN LUYỆN KIỂU BÀI NGHỊ LUẬN XÃ </b>
<b>HỘI...124</b>


<b>C. KẾT LUẬN...138</b>


</div>
<span class='text_page_counter'>(5)</span><div class='page_container' data-page=5>

<b>BÀI TOÁN CHIA HẾT TRONG SỐ HỌC</b>



<b>Lê Xuân Đại – THPT Chuyên Vĩnh Phúc</b>


<b>Đối tượng bồi dưỡng: Học sinh khá, giỏi THCS</b>
<b>Dự kiến số tiết bồi dưỡng: 15 tiết</b>


<b>MỞ ĐẦU</b>



Để làm quen với số học thì việc đầu tiên, hãy biết đến các bài tốn chia hết, vì
nó là một khái niệm cơ bản và cũng là trọng tâm của số học. Những bài toán về
chia hết có thể nói là khơng thể thiếu trong số học nói riêng và tốn học nói chung.
Trên thế giới đã có rất nhiều bài tốn về chia hết rất hay, rất đẹp, và cũng có những
<i>phương pháp chứng minh nó thật thú vị và bổ ích. Khi cho trước số nguyên a và số</i>
<i>nguyên dương b, một trong những câu hỏi hiển nhiên được đặt ra là: Liệu a có chia</i>
<i>hết cho b khơng? Và làm cách nào để biết được điều đó? Đó là những điều mà</i>
chúng ta phải giải quyết thường xuyên khi gặp những bài tốn về số học.


Có thể nói những vấn đề về đồng dư chia hết là vấn đề rất cơ bản và là kiến
thức bản lề khi học về phân mơn số học. Thường thì học sinh hay lao ngay vào
những bài tốn về phương trình nghiệm ngun và các thủ thuật giải nó mà khơng
biết rằng chính những bài toán về phép chia hết lại là gốc dễ của mọi vấn đề. Hiểu
rõ tầm quan trọng này, tác giả xin đưa ra một số phương pháp cơ bản giải các bài
tốn chia hết, sau đó đưa ra cách khai thác và tiếp cận với những bài tốn khó hơn.
Qua đó hy vọng phần nào giúp bạn đọc có cách nhìn và sự định hướng đúng đắn
khi gặp các bài toán về số học.


<b>NỘI DUNG</b>


<b>A. MỘT SỐ VẤN ĐỀ VỀ ĐỒNG DƯ CHIA HẾT.</b>


<b>I. Phép chia trong tập số nguyên</b>


<i><b>1.1. Định nghĩa. Cho hai số nguyên ,</b>a b , a  . Ta nói b chia hết cho a, ký hiệu</i>0


M


</div>
<span class='text_page_counter'>(6)</span><div class='page_container' data-page=6>

<i>Ta còn nói a chia hết b hoặc b là bội của a và kí hiệu b a a b .</i>M; |



<i><b>1.2. Tính chất cơ bản.</b></i>


1.2.1. Nếu | ,<i>a b b  thì a b</i>0  .


1.2.2. Nếu |<i>a b và |a c thì |a mb nc</i> <i> với m,n nguyên.</i>
1.2.3. Nếu |<i>a b và |a b c</i> thì |<i>a c .</i>


<i>1.2.4. Với mọi số nguyên a khác 0 thì |a a .</i>
1.2.5. Nếu |<i>a b và |b c thì |a c .</i>


1.2.6. Nếu |<i>a b và |b a thì a b</i> .


1.2.7. Nếu |<i>a b và b  thì </i>0 |
<i>b</i>


<i>b</i>
<i>a</i> <sub>.</sub>


<i><b>1.3.Thuật chia Euclide. Cho a và b là những số nguyên, </b>b  . Khi đó tồn tại duy</i>0


nhất cặp số nguyên ( , )<i>q r sao cho a bq r</i>  , 0  .<i>r b</i>


<i>Ta gọi q là thương, r là phần dư của phép chia a cho b. Như vậy, a chia hết cho b</i>
khi và chỉ khi phần dư trong thuật chia Euclide bằng 0. Ta cũng thường gọi thuật
<i>chia Euclide là phép chia Euclide.</i>


<b>II. Số nguyên tố và hợp số</b>


<i><b>2.1. Định nghĩa. Số nguyên </b>n  gọi là số nguyên tố nếu nó chỉ có hai ước</i>1



nguyên dương là 1 và chính nó. Số ngun <i>n  khơng là ngun tố được gọi là</i>1
hợp số.


<i><b>2.2. Tính chất cơ bản.</b></i>


2.2.1. Mỗi số nguyên dương lớn hơn 1 đều có ước nguyên tố.


</div>
<span class='text_page_counter'>(7)</span><div class='page_container' data-page=7>

2.2.3. Có vơ hạn số ngun tố (số nguyên tố lớn nhất đã tìm ra là 232582657 1<sub>, nó</sub>
được tìm ra năm 2006 và nó có 9808358 chữ số).


<i>2.2.4. (Phân tích một số theo các thừa số nguyên tố). Mỗi số nguyên dương n </i>1
được phân tích duy nhất thành tích các thừa số nguyên tố: <i>n</i> <i>p p</i>11. 22...<i>pkk</i>



 


 <sub>, với </sub><i>p<sub>i</sub></i>


nguyên tố và <i>i</i>




  .


<i><b>III. Ước chung lớn nhất, bội chung nhỏ nhất (The greatest common divisor and</b></i>


<i>the least common multiple).</i>


<i><b>3.1. Định nghĩa. </b></i>



<i>3.1.1. Giả sử a,b là hai số nguyên không đồng thời bằng 0. Ước chung lớn nhất của</i>
<i>hai số a,b là số nguyên lớn nhất chia hết cả hai số đó. Ta thường dùng kí hiệu</i>
( , )<i><sub>a b để chỉ ước chung lớn nhất của hai số a và b. Hai số nguyên a,b được gọi là</sub></i>
nguyên tố cùng nhau nếu ( , ) 1<i>a b  .</i>


<i>3.1.2. Giả sử a,b là hai số nguyên khác 0. Bội chung nhỏ nhất của hai số a,b là số</i>
nguyên dương nhỏ nhất chia hết cho cả hai số đó. Ta thường dùng kí hiệu

<i>a b để</i>,


<i>chỉ bội chung nhỏ nhất của hai số a và b.</i>


<i><b>3.2. Tính chất.</b></i>


<i>3.2.1. Nếu p ngun tố thì ( , ) 1a p  hoặc ( , )a p</i>  .<i>p</i>
3.2.2. Nếu <i>d</i> ( , )<i>a b</i> thì <i>a da b db</i> ';  '; ( ', ') 1<i>a b</i> 
<i>3.2.3. Nếu d' là một ước chung của a và b thì ' | ( , )d</i> <i>a b .</i>
3.2.4. Nếu <i>px</i> ||<i>a và ||px</i> <i>b thì p</i>min( , )<i>x y</i> || ( , )<i>a b .</i>


Do đó với <i>a</i> <i>p p</i>11. 22...<i>pkk</i>; <i>b</i> <i>p p</i>11. 22...<i>pkk</i>; ,1 <i>i</i> 0


 


   


 


</div>
<span class='text_page_counter'>(8)</span><div class='page_container' data-page=8>

1 1 2 2 min( , )
min( , ) min( , )


1 2


( , ) . ... <i>k</i> <i>k</i>



<i>k</i>


<i>a b</i> <i>p</i>   <i>p</i>   <i>p</i>  


 <sub> và </sub>

max( , )1 1 max( 2, 2) max( , )


1 2


, . ... <i>k</i> <i>k</i>


<i>k</i>


<i>a b</i> <i>p</i>   <i>p</i>   <i>p</i>  


 <sub>.</sub>


<i>3.2.5. Nếu a bq r</i>  thì ( , ) ( , )<i>a b</i>  <i>b r</i> .


<i>3.2.6. Với a,b nguyên dương thì ab</i>( , ). ,<i>a b a b</i>

.


3.2.7. Nếu ( , ) ( , ) 1<i>a b</i>  <i>a c</i>  thì ( , ) 1<i>a bc  . Nếu ( , ) 1a b  thì ( , ) 1a b  .m</i> <i>n</i>


<i>3.2.8. Nếu a,b là hai số nguyên dương nguyên tố cùng nhau thì tồn tại hai số</i>
<i>nguyên dương u,v sao cho au bv</i> <sub> .</sub>1


<i>Tổng quát hơn: Nếu a,b là hai số nguyên dương thì tồn tại hai số nguyên u,v sao</i>
cho <i>au bv</i> ( , )<i>a b</i> .


<i><b>IV. Đồng dư (Modular arithmetics)</b></i>



<i><b>4.1. Định nghĩa. Cho ,</b>a b là số nguyên và n là số nguyên dương. Nếu a b</i> <sub> chia</sub>
<i>hết cho n thì ta nói a đồng dư với b modulo n, ký hiệu a b</i> (mod )<i>n</i> .


<i><b>4.2. Tính chất. Cho , , ,</b>a b c d là các số nguyên. Ta có các tính chất cơ bản:</i>


4.2.1. Nếu <i>a b</i> (mod )<i>n</i> thì <i>b a</i> (mod )<i>n</i> .


4.2.2. Nếu <i>a b</i> (mod )<i>n</i> và <i>b c</i> (mod )<i>n</i> thì <i>a c</i> (mod )<i>n</i> .


4.2.3. Nếu <i>a b</i> (mod )<i>n</i> và <i>c d</i> (mod )<i>n</i> thì <i>a c b d</i>   (mod )<i>n</i> .
4.2.4. Nếu <i>a b</i> (mod )<i>n</i> và <i>c d</i> (mod )<i>n</i> thì <i>ac bd</i> (mod )<i>n</i> .


4.2.5. Nếu <i>a b</i> (mod )<i>n</i> <i> thì với mỗi số nguyên k đều có ka kb</i> (mod )<i>n</i> .
4.2.6. Nếu <i>ai</i> <i>bi</i> (mod ),<i>n i</i>1,2,...,<i>k</i> thì <i>a a a</i>1 2... <i>k</i> <i>b</i>1 2b ...b (mod )<i>k</i> <i>n</i> .


Đặc biệt nếu <i>a b</i> (mod )<i>n</i> <i> thì với mỗi k nguyên dương ta có ak</i> <i>bk</i> (mod )<i>n</i> .
4.2.7. Nếu <i>ab ac</i> (mod )<i>n</i> và ( ,n) 1<i>a</i>  thì <i>b c</i> (mod )<i>n</i> .


</div>
<span class='text_page_counter'>(9)</span><div class='page_container' data-page=9>

Đặc biệt nếu <i>m m</i>1, 2,...,<i>mk</i> nguyên tố sánh đơi thì


1 2



(mod ),<i><sub>i</sub></i> 1,2,..., mod ... <i><sub>k</sub></i>
<i>a b</i> <i>m i</i> <i>k</i>  <i>a b</i> <i>m m m</i> <sub>.</sub>


<i><b>4.3. Định lý Fermat. Giả sử p nguyên tố, a là số nguyên sao cho ( , ) 1</b>a p  . </i>


Khi đó <i>ap</i>1 1 (mod )<i>p</i> .



Có thể đưa ra một chứng minh đơn giản cho định lý này như sau:


Xét <i>p  số ,2 ,...,( 1)</i>1 <i>a a</i> <i>p</i> <i>a</i>. Ta chứng minh rằng không tồn tại 2 số đồng dư với
<i>nhau trong phép chia cho p. Thật vậy, giả sử tồn tại ka</i><i>a</i> (mod )<i>p</i> với




, 1,2,... 1 ( )


<i>k</i>  <i>p</i>  <i>a k</i>   <i>p</i> <i>k</i>  <sub> (mâu thuẫn).</sub><i>p</i>


Vậy khi chia <i>p  số trên cho p ta nhận được </i>1 <i>p  số dư khác nhau là 1,2,...,</i>1 <i>p </i> 1
. Suy ra .2 ...(<i>a a</i> <i>p</i> 1)<i>a</i>1.2....(<i>p</i> 1) (mod )<i>p</i>


 (<i>p</i> 1)!.<i>ap</i>1(<i>p</i> 1)! (mod )<i>p</i> .
Vì ((<i>p</i> 1)!, ) 1<i>p</i>  suy ra <i>ap</i>1 1 (mod )<i>p</i> .


<i><b>Hệ quả. Nếu p nguyên tố thì </b>ap</i> <i>a</i> (mod )<i>p</i> .


<b>B. MỘT SỐ PHƯƠNG PHÁP CƠ BẢN CHỨNG MINH BÀI TOÁN CHIA </b>
<b>HẾT</b>


<b>1. Phương pháp dùng phép chia có dư.</b>


<i>Căn cứ vào số chia b, mà xét mọi khả năng phân tích a bk r</i> <sub> với</sub>


0;1;2;...; 1



<i>r</i> <i>b</i> <sub>. Sau đó, với mỗi khả năng phân tích này lý luận để suy ra lời</sub>
giải của bài toán. Chẳng hạn với <i>b  thì với mỗi số nguyên a có thể phân tích</i>3


thành một trong ba dạng <i>a</i>3 ;<i>k a</i>3<i>k</i> 1;<i>a</i>3<i>k</i>  .2


</div>
<span class='text_page_counter'>(10)</span><div class='page_container' data-page=10>

Với <i>a</i>3<i>k</i><sub> thì </sub><i>a</i>2  1 9<i>k</i>2<sub> , chia 3 dư 1.</sub>1


Với <i>a</i>3<i>k</i> <sub> thì </sub>1 <i>a</i>2  1 9<i>k</i>2 6<i>k</i><sub> , chia 3 dư 2.</sub>2
Với <i>a</i>3<i>k</i> <sub> thì </sub>2 <i>a</i>2  1 9<i>k</i>2 12<i>k</i> <sub> , chia 3 dư 2.</sub>5


Vậy trong mọi trường hợp thì <i>a  đều khơng chia hết cho 3.</i>2 1


<b>Ví dụ 2. Tìm tất cả các số nguyên dương n thỏa mãn .2</b><i>n</i> <i>n</i> 1 3M<sub>.</sub>


<i><b>Lời giải. Rõ ràng n không chia hết cho 3. Như vậy, n có một trong các dạng</b></i>


6<i>k</i> 1,6<i>k</i> 2,6<i>k</i>4,6<i>k</i> 5 (<i>k</i>¥)<sub>.</sub>


Nếu <i>n</i>6<i>k</i> <sub> thì do </sub>1 26 1 (mod3)<sub>, ta có: </sub> .2 1

 

26 .2 1 2 1 0 (mod3)
<i>k</i>


<i>n</i>


<i>n</i>       <sub>,</sub>


tức là .2<i>n</i> <i>n</i> 1 3M<sub>.</sub>


Nếu <i>n</i>6<i>k</i> <sub> thì </sub>2 .2 1 2 2

 

6 .22 1 8 1 0 (mod3)
<i>k</i>


<i>n</i>


<i>n</i>       <sub>, tức là .2</sub><i><sub>n</sub></i> <i>n</i> <sub></sub><sub>1 3</sub><sub>M</sub>



.
Nếu <i>n</i>6<i>k</i> <sub> thì </sub>4

 



6 4 6


.2<i>n</i> 1 4 2 <i>k</i>.2 1 2 1 2 (mod3)


<i>n</i>       <sub>.</sub>


Nếu <i>n</i>6<i>k</i> <sub> thì </sub>5

 



6 5


.2<i>n</i> 1 5 2 <i>k</i>.2 1 2 (mod3)


<i>n</i>     <sub>.</sub>


Như vậy, .2<i>n</i> <i>n</i> 1 3M<sub> khi và chỉ khi n có dạng 6</sub><i>k  hoặc 6</i>1 <i>k  , ¥</i>2 <i>k</i> <sub>.</sub>


<i><b>Nhận xét: Các số dạng .2</b>n</i> <i>n</i>  được gọi là số Cullen. Các số Cullen với1
2,3,...,140


<i>n </i> <sub> đều là hợp số, nhưng số Cullen với </sub><i><sub>n </sub></i><sub>141</sub><sub> là số ngun tố. Từ bài</sub>
tốn trên ta suy ra có vơ hạn số Cullen là hợp số, tuy nhiên cho đến nay vẫn chưa
biết có hữu hạn hay vơ hạn số Cullen là số nguyên tố.


<i><b>Ví dụ 3. Cho p là số nguyên tố lớn hơn 7. Chứng minh rằng 3</b>p</i>  2<i>p</i>  1 42M <i>p</i>
<i><b>Lời giải. Ta sẽ chứng minh 3</b>p</i>  2<i>p</i>  1 42M <i>p chia hết cho 2; 3; 7 và p.</i>



</div>
<span class='text_page_counter'>(11)</span><div class='page_container' data-page=11>

<i>Do p lẻ nên 3p</i>  2<i>p</i>  1 ( 1)<i>p</i>  1 0 (mod3)  3<i>p</i>  2<i>p</i>  1 3M


Khó khăn nhất của bài tốn là chứng minh 3<i>p</i>  2<i>p</i>  1 7M<sub>. Để giải quyết bước này ta</sub>
<i>xét hai dạng của p là p</i>6<i>k</i> và 1 <i>p</i>6<i>k</i>  .5


Ta có

 

 



3 2


6 1 6 1 3


3<i>p</i> 2<i>p</i> 1 3 <i>k</i> 2 <i>k</i> 1 3. 9 <i>k</i> 2. 8 <i>k</i> 1 3.2 <i>k</i> 2 1 0 (mod7)


           


Tương tự với <i>p</i>6<i>k</i>  . Vậy bài toán được chứng minh.5


<i><b>Bài tập tương tự:</b></i>


<b>Bài 1. Chứng minh rằng </b><i>a</i>3  <i>a<sub> chia hết cho 6 với mọi số nguyên a.</sub></i>


<b>Bài 2. Chứng minh rằng </b><i>a a </i>( 6 1)<i> chia hết cho 7 với mọi số nguyên a.</i>


<b>Bài 3. Chứng minh rằng </b><i>ab a</i>( 2  <i>b</i>2)(4<i>a</i>2  <i>b</i>2) chia hết cho 5 với mọi số nguyên
<i>a,b.</i>


<b>Bài 4. Tìm số phần tử của tập </b>


3 <sub>3</sub> <sub>2</sub>



|


2 1


   


<sub></sub>   <sub></sub>




 


¢ <i>x</i> <i>x</i> ¢


<i>S</i> <i>x</i>


<i>x</i> <sub>.</sub>


<b>2. Phương pháp sử dụng đồng dư</b>


Nội dung của phương pháp này đơn giản chỉ là dùng các tính chất và phép
biến đổi đồng dư để chứng minh tính chia hết hoặc tìm số dư trong một phép chia
nào đó.


<i><b>Ví dụ 1. Chứng mình rằng với mọi số ngun dương n thì </b></i>58<i>n</i> 23<sub> chia hết cho</sub>
24.


<i><b>Lời giải. Ta có: </b></i>



1 1



8 8.8 4.8


5<i>n</i> 5 <i>n</i> 25 <i>n</i> 1 mod 24


   <sub>, suy ra </sub>

58 23 24

M


<i>n</i>


(đpcm).


<i><b>Ví dụ 2. Chứng mình rằng với mọi số tự nhiên n thì </b></i>122 1<i>n</i> 11<i>n</i>2


</div>
<span class='text_page_counter'>(12)</span><div class='page_container' data-page=12>

<i><b>Lời giải. Ta có: </b></i>122<i>n</i>1 12.122<i>n</i> 12.144<i>n</i><sub>. Vì </sub>144 11 mod133

<sub> nên</sub>




144<i>n</i> 11 mod133<i>n</i> 12.144<i>n</i> 12.11 mod133<i>n</i>


   <sub> (1)</sub>


Mặt khác 11<i>n</i>2 121.11<i>n</i> 12.11 mod133<i>n</i>

(2).
Từ (1) và (2) suy ra 122<i>n</i>111<i>n</i>2 0 mod133

(đpcm).


<b>Ví dụ 3. Cho , ,</b><i>a b c là các số nguyên dương. Chứng minh rằng nếu a</i>2 <i>b</i>2 <i>c</i>2
chia hết cho 9 thì ít nhất một trong các số <i>a</i>2  <i>b b</i>2; 2  <i>c c</i>2; 2  <i>a</i>2 chia hết cho 9.


<i><b>Lời giải. Bài toán tương đương với việc chứng minh tồn tại ít nhất hai trong ba số</b></i>


2<sub>, ,</sub>2 2



<i>a b c</i> <sub> có cùng số dư khi chia cho 9.</sub>


Nhận xét rằng <i>n </i>2 0,1,4,7 mod9

<i> với mọi số nguyên n.</i>


Giả sử <i>a</i>2 <i>r</i>1

mod9 ;

<i>b</i>2 <i>r</i>2

mod9 ;

<i>c</i>2 <i>r</i>3

mod9

; <i>r i</i>

0,1,4,7

.




2 2 2


1 2 3 mod9 1 2 3 0 mod9


<i>a</i> <i>b</i> <i>c</i> <i>r</i> <i>r</i> <i>r</i> <i>r</i> <i>r</i> <i>r</i>


          <sub>.</sub>


Từ đó ta thấy ngay phải tồn tại ít nhất hai số <i>ri</i> bằng nhau và bài toán được chứng
minh.


<b>3. Phương pháp quy nạp toán học</b>


Phương pháp quy nạp tỏ ra rất hữu hiệu với những bài tốn chia hết phụ
<i>thuộc biến n và có dạng lũy thừa phức tạp. Ta đưa ra một vài bài toán minh họa</i>
cho phương pháp này.


<b>Ví dụ 1. Chứng minh rằng </b><i>xn</i> 4<i>n</i> 15<i>n chia hết cho 9 với mọi số tự nhiên n.</i>1


<i><b>Lời giải. Với </b>n  thì </i>0 <i>x</i>0 0 9M.



<i>Giả sử mệnh đề đúng với n, tức là xn</i>M9. Ta chứng minh mệnh đề đúng với <i>n  .</i>1


Ta có



1


1 4 15( 1) 1 4 4 15 1 9(5 2) 4 9(5 2)


<i>n</i> <i>n</i>


<i>n</i> <i>n</i>


<i>x</i>  <i>n</i> <i>n</i> <i>n</i> <i>x</i> <i>n</i>


</div>
<span class='text_page_counter'>(13)</span><div class='page_container' data-page=13>

Do đó <i>xn</i>1M9. Vậy <i>xn chia hết cho 9 với mọi số tự nhiên n.</i>


<b>Ví dụ 2. Cho </b>


7
..
.


7


7


<i>n</i>


<i>x </i> <i><b><sub> (n lần). Chứng minh rằng với mọi </sub></b><sub>n  thì </sub></i><sub>2</sub> <i>xn</i> 17 20M .



<i><b>Lời giải. Với </b>n  , ta dễ kiểm tra được </i>2 <i>x</i>2 17 20M .


<i>Giả sử mệnh đề đã đúng với n, tức là xn</i> 17 20M  <i>xn</i> 20<i>an</i> 3, ta chứng minh
mệnh đề đúng với <i>n  . Thật vậy, ta có</i>1


 

5

 

5


20 3 3 4


1 7 7 7 . 7 20 3 20 1 3 (mod 20)


<i>n</i> <i>n</i>


<i>n</i> <i>n</i> <i>a</i> <i>a</i>


<i>x</i> <i>a</i>


<i>n</i>


<i>x</i>  <i>m</i> <i>n</i>


         <i>x<sub>n</sub></i><sub></sub><sub>1</sub>17 20M .


Vậy <i>xn</i> 17 20M với mọi n nguyên dương, <i>n  .</i>2


<i><b>Ví dụ 3. Chứng minh rằng với mọi n nguyên dương thì </b></i>23<i>n</i> 1 3M<i>n</i><sub>.</sub>


<i><b>Lời giải. Với </b>n  mệnh đề hiển nhiên đúng</i>1


<i>Giả sử mệnh đề đúng với n, tức là </i>23<i>n</i> 1 3M<i>n</i>  23<i>n</i> <i>k</i>.3<i>n</i>  1<sub>.</sub>



Ta có



1 3


3 3 3 2 2 1 1 1


2<i>n</i> .3<i><sub>n</sub></i> 1 .3 <i><sub>n</sub></i> .3 <i><sub>n</sub></i> .3<i><sub>n</sub></i> 1 3 . 1; <i><sub>n</sub></i>


<i>k</i> <i>k</i> <i>k</i> <i>k</i> <i>t</i> <i>t</i>


 <sub></sub> <sub></sub> <sub></sub> <sub></sub>


          <sub>.</sub>


Như vậy, 231 1 3 1


 M


<i>n</i> <i><sub>n</sub></i>


(đpcm).


<i><b>Nhận xét 1: Từ bài toán ta suy ra kết quả sau:</b></i>


<i>Tồn tại vô hạn số nguyên dương n sao cho 2n</i> 1M<i>n . Rõ ràng cách hỏi này rất khó</i>
<i>vì để làm được nó chúng ta phải đoán nhận ra một dạng tổng quát nào đó của n,</i>
trong bài tốn này thì chỉ cần chọn <i>n  là được. Đây vẫn là dạng bài khó đặc</i>3<i>k</i>
biệt là với học sinh THCS.



<i>Tuy nhiên nếu đưa thêm điều kiện n ngun tố thì có thể tìm được n thỏa mãn đề</i>
bài. Thật vậy, theo định lý Fermat, 2<i>n</i>  2M<i>n , suy ra </i>3<i>Mn</i> <i>n</i>3<sub>. </sub>


</div>
<span class='text_page_counter'>(14)</span><div class='page_container' data-page=14>

<i><b>Nhận xét 2: Ta có thể đưa ra một bài tốn mà cách hỏi có bản chất khác hẳn như</b></i>


sau: Chứng minh rằng phương trình 2<i>x</i>  1 <i>xy</i> có vơ hạn nghiệm ngun dương.


Rõ rằng một họ nghiệm của phương trình này là




3


2 1


; 3 ; ( )


3


 <sub></sub> 


<sub></sub> <sub></sub> 


 


 


¥


<i>k</i>


<i>k</i>


<i>k</i>


<i>x y</i> <i>k</i>


.


<i><b>Bài tập tương tự: </b></i>


<i><b>Bài 1. Chứng minh rằng với mọi số tự nhiên n: </b></i>


a) 25<i>n</i>3 5 .3<i>n</i> <i>n</i>2<sub> chia hết cho 17.</sub>
b) 52<i>n</i>1 2<i>n</i>4 2<i>n</i>1


  <sub> chia hết cho 23.</sub>
c) 20 2122 ... 2 5<i>n</i>1<sub> chia hết cho 31.</sub>


<b>Bài 2. Chứng minh rằng tổng lập phương của ba số tự nhiên liên tiếp luôn chia hết</b>


cho 9.


<b>Bài 3. Chứng minh rằng với mọi số nguyên không âm b, số </b>33<i>n</i>3 26<i>n</i> 27


  <sub> chia</sub>


hết cho 169.


<b>4. Phương pháp sử dụng định lý Fermat</b>



Với những bài tốn khó hơn, đặc biệt là những bài toán chia hết liên quan
đến số nguyên tố, nếu biết dùng khéo léo định lý Fermat sẽ cho ta lời giải nhanh và
gọn gàng. Định lý Fermat được phát biểu thật đơn giản và dễ nhớ nhưng lại rất
mạnh, việc dùng được nó cho phù hợp và tinh tế là điều quan trọng nhất khi dạy
cho học sinh cấp THCS. Có thể nói định lý Fermat đem lại vẻ đẹp trong các bài
tốn chia hết nói riêng và các bài tốn số học nói chung.


<i><b>Ví dụ 1. Cho p là số nguyên tố lẻ, đặt </b>Sk</i> 1<i>k</i> 2<i>k</i> ... <i>p kk</i>,




    <sub>  .</sub>


Biết rằng <i>S pk</i>M <i>. Chứng minh rằng k không chia hết cho p  .</i>1


<i><b>Lời giải. Giả sử k chia hết cho </b>p  . Theo định lý Fermat ta có</i>1
1 <sub>1 (mod ), </sub> <sub>1,...,</sub> <sub>1</sub>


<i>p</i>


<i>a</i>  <i>p</i> <i>a</i> <i>p</i>


</div>
<span class='text_page_counter'>(15)</span><div class='page_container' data-page=15>

Do <i>k p</i>M  1 nên <i>ak</i> 1 (mod ), <i>p</i>  <i>a</i> 1,...,<i>p</i> , suy ra 1 <i>Sk</i>  <i>p</i> 1 (mod )<i>p</i> , mâu


thuẫn với <i>S pk</i>M <i>. Vậy k không chia hết cho p  .</i>1


<b>Ví dụ 2. Chứng minh rằng với mọi số tự nhiên k ta có </b>226<i>k </i>2  chia hết cho 19.3


<i><b>Lời giải. Ta có </b></i>26 64 1 (mod9)  26<i>k</i> 1 (mod9) 26<i>k </i>2 4 (mod9)



Do hai vế đều chẵn nên 26<i>k</i>2 4 (mod18) 26<i>k</i>2 18<i>t</i>4 (<i>t</i>  .)
Mặt khác, theo định lý Fermat, 218 1 (mod19) 218<i>t</i> 1 (mod19)
Như vậy,


6 2 6 2


2 18 4 4 2 4


2 <i>k</i> 2 <i>t</i><sub></sub> 2 (mod19) 2 <i>k</i> 3 2 +3 0 (mod19)


      <sub> (đpcm).</sub>


<b>Ví dụ 3. Chứng minh rằng </b>270 370<sub> chia hết cho 13.</sub>


<i><b>Lời giải. Theo định lý Fermat ta có: </b></i>212 1 (mod13) 260 1 (mod13).


Mặt khác, 25 6 (mod13) 210 3 (mod13). Do đó 270 2 .260 10 3 (mod13)
Lại có 33 1 (mod13) 369 1 (mod13) 370 3 (mod13).


Như vậy, 270 370 0 (mod13) (đpcm).


<i><b>Ví dụ 4. Cho p nguyên tố và a là số nguyên. Chứng minh rằng nếu |</b>p a  thìp</i> 1
2<sub>|</sub> <i>p</i> <sub>1</sub>


<i>p a  .</i>


<i><b>Lời giải. Ta có |</b>p a  suy ra p</i> 1 ( , ) 1<i>a p</i>   <i>ap</i>1 1 (mod )<i>p</i> .
Suy ra



1 <sub>1 (</sub> <sub>1)</sub> <sub>(</sub> <sub>1)</sub> <sub>(</sub> 2 <sub>1)</sub> 2 <sub>1</sub>



  


  M     M   M


<i>p</i> <i>p</i> <i>p</i> <i>p</i>


<i>a</i> <i>a</i> <i>p</i> <i>a</i> <i>a a</i> <i>p</i> <i>a</i> <i>p</i>


.
Tương tự <i>ap</i>3  1 ;...;M<i>p</i> <i>a</i> 1M<i>p . Do đó ak</i> 1 (mod ) <i>p</i>  <i>k</i> 1,<i>p</i> 1.


Do đó



1 2 2


1 ( 1)   ... 1


       M


<i>p</i> <i>p</i> <i>p</i>


<i>a</i> <i>a</i> <i>a</i> <i>a</i> <i>a</i> <i>p</i>


(đpcm).


<b>Ví dụ 5. Cho p nguyên tố lẻ, đặt </b>


9 1



8


<i>p</i>


</div>
<span class='text_page_counter'>(16)</span><div class='page_container' data-page=16>

<i>a) Chứng minh rằng m là hợp số lẻ và m không chia hết cho 3.</i>
b) Chứng minh rằng <i>m</i> 1 2M<i>p</i>.


c) Chứng minh rằng 3<i>m</i>11 (mod )<i>m</i> .


<i><b>Lời giải. a) Ta có </b></i>


3 1 3 1


.


4 2


<i>p</i> <i>p</i>


<i>m</i>  


<i>, suy ra m là hợp số.</i>


Mặt khác <i>m</i>9<i>p</i>1 <i>pp</i>2 ... 9 1<i>  , suy ra m lẻ và m chia 3 dư 1.</i>
b) Theo định lý Fermat thì 9<i>p</i>  9M<i>p mà 9 9 8 9 9 8p</i>  M <i>p</i>  M<i>p .</i>


Vậy


9 9



1


8


<i>p</i>


<i>m</i>  


<i> chia hết cho p.</i>


c) Ta có


1 2 9 1 1


3 1 3 1 3 1


8


  


 M  M   M


<i>p</i>


<i>m</i> <i>p</i> <i>m</i> <i><sub>m</sub></i>


(đpcm).


<i><b>Ví dụ 6. Chứng minh rằng với mọi số nguyên tố p, tồn tại vô số nguyên dương n</b></i>



thoả mãn 2  M<i>n</i> <i>n p .</i>


<i><b>Lời giải. Nếu </b>p  thì mọi n chẵn đều thoả mãn điều kiện đề bài.</i>2


Nếu <i>p  , khi đó theo định lý Fermat, ta có: </i>2 2<i>m p</i> 1 1 mod

<i>p</i>

,  <i>m</i> ¥*.
Lấy <i>n m p</i> (  1) với <i>m</i>1 mod

<i>p</i>

.


Khi đó <i>n m p</i> (  1) 1 (mod ) <i>p</i> và 2<i>n</i>  <i>n</i>2<i>n</i>  1 0 (mod ) <i>p</i> .


<i>Do có vơ số số nguyên dương m sao cho m</i>1 mod

<i>p</i>

nên tồn tại vô số số
<i>nguyên dương n thoả mãn 2  Mn</i> <i>n p (đpcm).</i>


<i><b>Ví dụ 7. Cho p là nguyên tố lớn hơn 5. Chứng minh rằng số </b></i> ( 1) c/s1


11...11




1442443


<i>p</i>


<i>S</i>


</div>
<span class='text_page_counter'>(17)</span><div class='page_container' data-page=17>

<i><b>Lời giải. Ta có S = </b></i>
1
10 1


9



<i>p</i>




<i><b>. Vì (10, ) 1</b>p  nên theo định lý Fermat, ta được </i>
1


10  1
 M
<i>p</i>


<i>p . Mặt khác </i><sub>10</sub> 1 <sub>1 9</sub>
 M
<i>p</i>


<i><b> và ( ,9) 1</b>p</i>  nên 10<i>p</i>1  1 9M<i>p</i> <i>S p . </i>M
<b>Bài tương tự</b>


<i><b>Bài 1. Cho p, q là hai số nguyên tố phân biệt. Chứng minh rằng </b>pq</i>1<i>qp</i>1 1M<i>pq .</i>
<i><b>Bài 2. Tìm các số nguyên tố p và q sao cho p</b></i>3<i><sub> – q</sub></i>5<i><sub> = (p + q)</sub></i>2<sub> </sub>


<b>C. MỘT SỐ BÀI TOÁN NÂNG CAO</b>


Trong phần này đưa ra một số bài tốn nâng cao điển hình về số học liên
quan đến đồng dư và chia hết, sau đó phân tích và nhấn mạnh một số kết quả và
tính chất quan trọng đã được sử dụng và khai thác trong bài tốn. Qua đó góp phần
nào cho các bạn có cách nhìn và cách tiếp cận với các bài toán số học THCS.


<b>Bài toán 1. Cho </b><i>p  là một số lẻ và n là số nguyên dương. Chứng minh rằng</i>2



1<i>pn</i> 2<i>pn</i> ... (<i><sub>p</sub></i> 1)<i>pn</i>


    <i><sub> chia hết cho p.</sub></i>


<i><b>Lời giải. Đặt </b>k</i> <i>pn thì k lẻ. Ta có</i>


1 2 1


( ) <sub></sub>   ( ) ... ( )  <sub></sub>


   <sub></sub>      <sub> M</sub>


<i>k</i> <i>k</i> <i>k</i> <i>k</i> <i>k</i>


<i>d</i> <i>p d</i> <i>p d</i> <i>d</i> <i>p d</i> <i>p d</i> <i>p</i>


<i>Lấy tổng khi cho d chạy từ 1 đến </i>
1
2
<i>p </i>


ta được điều phải chứng minh.


<i><b>Chú ý: Với ,</b>a b nguyên phân biệt, ta có an</i>  <i>b a b nn</i>M  ¥ <sub>.</sub>
Với ,<i>a b nguyên , ta có an</i> <i>b a b nn</i>M  ¥ <i><sub>, n lẻ.</sub></i>


Tính chất này rất hiển nhiên nhưng được sử dụng nhiều trong những bài tốn chia
hết có lũy thừa của một số nguyên.


</div>
<span class='text_page_counter'>(18)</span><div class='page_container' data-page=18>

Cho đa thức ( )<i>P x có các hệ số nguyên. Khi đó ( )P a</i>  <i>P b</i>( ) luôn chia hết


<i>cho a b</i> <sub> với mọi ,</sub><i>a b nguyên phân biệt. Ta thử đưa ra một ví dụ áp dụng tính chất</i>
này.


<i><b>Bài tốn 1.1. Tồn tại hay khơng đa thức ( )</b>P x hệ số nguyên sao cho (1) 2014P</i>  ;
(3) 2015


<i>P</i>  <sub>?</sub>


<i><b>Hint. Nếu tồn tại đa thức ( )</b>P x thỏa mãn đề bài thì (3)P</i>  <i>P</i>(1) 1 phải chia hết


cho 3 1 2 <sub> , đây là điều vô lý. Vậy không tồn tại ( )</sub><i>P x thỏa mãn.</i>


<i><b>Bài toán 1.2. Cho đa thức ( )</b>P x hệ số nguyên. Chứng minh rằng không tồn tại ba</i>


số nguyên phân biệt , ,<i>a b c thỏa mãn ( ) ; ( ) ; ( )P a</i> <i>b P b</i> <i>c P c</i>  .<i>a</i>


<i><b>Bài toán 1.3. Cho m nguyên dương sao cho 2</b>m</i>  1<sub> là số nguyên tố. Chứng minh</sub>
<i>rằng m là số nguyên tố.</i>


<i><b>Hint. Giả sử m là một hợp số, suy ra </b>m</i><i>pq p q</i> ( , ¥*)


Ta có 2<i>m</i>  1 2 <i>pq</i>  1 2M<i>p</i>  1<sub> và 2</sub><i>p</i>  1 1 <sub> nên 2</sub><i>m</i>  1<sub> là hợp số, trái giả thiết.</sub>


<i><b>Nhấn mạnh: Tính chất rất quan trọng: Nếu m,n nguyên dương thỏa mãn </b>m n</i>M thì


1 1


 M 


<i>m</i> <i>n</i>



<i>a</i> <i>a</i> <sub>.</sub>


<i><b>Bài toán 2. Cho n là số nguyên dương lớn hơn 1 thỏa mãn 3</b>n</i>  1M<i>n . Chứng minh</i>
<i>rằng n chẵn.</i>


<i><b>Phân tích bài tốn. Để chứng minh một số n là chẵn ta chỉ cần chứng minh ước</b></i>


</div>
<span class='text_page_counter'>(19)</span><div class='page_container' data-page=19>

<i><b>Bổ đề: Cho a nguyên, n và p nguyên dương thỏa mãn </b>an</i> 1 (mod )<i>p</i> <i>. Gọi h là số</i>
nguyên dương nhỏ nhất thỏa mãn <i>ah</i> 1 (mod )<i>p</i> <i>. Khi đó n chia hết cho h.</i>


<i><b>Chứng minh: Ta biểu diễn </b>n kh r</i>  , 0  .<i>r h</i>


Ta có

 

. (mod ) 1 (mod )


<i>k</i>


<i>n</i> <i>kh r</i> <i>h</i> <i>r</i> <i>r</i> <i>r</i>


<i>a</i> <i>a</i>  <i>a</i> <i>a</i> <i>a</i> <i>p</i> <i>a</i> <i>p</i>


    


. Nếu <i>r  thì điều này</i>0
<i>mẫu thuẫn với việc chọn h là số mũ nhỏ nhất thỏa mãn ah</i> 1 (mod )<i>p</i> .


Vậy <i>r  , tức là n chia hết cho h. </i>0


<i><b>Lời giải bài toán 2. </b></i>



<i>Gọi p là ước nguyên tố nhỏ nhất của n, ta chứng minh p  .</i>2
Ta có 3<i>n</i> 1 (mod )<i>p</i>  <i>p</i> .3


<i>Gọi h là số nguyên dương nhỏ nhất sao cho 3h</i> 1 (mod )<i>p</i> (*)
<i>Theo bổ đề trên thì n chia hết cho h. </i>


Cũng theo định lý Fermat ta có 3<i>p</i>1 1 (mod )<i>p</i> , nên cũng theo bổ đề thì |<i>h p </i> 1
Nếu <i>h  thì gọi q là ước nguyên tố của h , suy ra h q</i>1  và |<i>q n . Mà</i>


1


<i>p</i>  <i>h</i> <i>p h q<sub>  , mâu thuẫn với p là ước nguyên tố nhỏ nhất của n.</sub></i>
Vậy <i>h  , từ đó theo (*) ta được </i>1 <i>p  . Do đó n chẵn (đpcm).</i>2


<i><b>Một số bài tốn tương tự (sự dụng bổ đề)</b></i>


<i><b>Bài toán 2.1. Cho p nguyên tố. Gọi q là ước nguyên tố bất kì của 2</b>p</i>  1<sub>. Chứng</sub>
minh rằng <i>q  chia hết cho p.</i>1


<i><b>Hướng dẫn. Gọi h là số nguyên dương nhỏ nhất thỏa mãn 2</b>h</i> 1 (mod )<i>q</i> .
Khi đó |<i>h p , dễ thấy 1h  suy ra h p</i> .


</div>
<span class='text_page_counter'>(20)</span><div class='page_container' data-page=20>

<i><b>Bài toán 2.2. Cho p nguyên tố, a là số nguyên, 1</b></i><i>a</i>  . Đặt <i>p</i> 1


1


0


<i>p</i>
<i>k</i>


<i>k</i>


<i>A</i> <i>a</i>






<sub></sub>



<i>. Gọi q</i>
<i>là ước nguyên tố bất kì của A. Chứng minh rằng q  chia hết cho p.</i>1


<i><b>Bài toán 2.3. Cho n nguyên, </b>n  thỏa mãn </i>1 3<i>n</i>  1M<i>n . Chứng minh rằng n chẵn và</i>3
<i>n không chia hết cho 4. </i>


<i><b>Bài toán 2.4. Cho a là số tự nhiên, n nguyên lớn hơn 1 thỏa mãn </b>an</i> 1M<i>n . </i>2
<i>a) Chứng minh rằng n lẻ.</i>


b) Chứng minh rằng <i>a  không là lũy thừa của 2.</i>1


<i><b>Hướng dẫn. a) Nếu n chẵn thì </b>a là số chính phương, suy ran</i>


0,1 (mod 4) 1 1,2 (mod 4)


<i>n</i> <i>n</i>


<i>a</i>   <i>a</i>   <sub>. Nhưng </sub><i>n </i>2 0 (mod 4)<sub>, nên đây là điều</sub>
mẫu thuẫn.



<i>b) Gọi p là ước nguyên tố nhỏ nhất của n, vì n lẻ nên p lẻ.</i>
Ta có 1 (mod )

1 (mod )


<i>n</i>
<i>n</i>


<i>a</i>  <i>p</i>   <i>a</i>  <i>p</i> <sub>. Gọi h là số nguyên dương nhỏ nhất</sub>
sao cho

1 (mod )


<i>h</i>


<i>a</i> <i>p</i>


  <sub>. </sub>


Khi đó |<i>h p</i> 1; |<i>h n</i>. Từ đó chứng minh được <i>h  (tương tự các bài tốn trên)</i>1
Suy ra <i>a</i>1M<i>p</i>. Vậy <i>a  có ước nguyên tố lẻ, tức là </i>1 <i>a  không thể là lũy thừa</i>1
của 2 (đpcm).


<i><b>Bài tốn 2.5. Tìm tất cả n nguyên dương sao cho 2</b>n</i>  1M<i>n .</i>


<i><b>Bài toán 2.6. Cho p là số nguyên tố lẻ, q,r là những số nguyên tố thỏa mãn ương</b></i>


sao cho <i>qr</i> 1M<i>p . Chứng minh rằng hoặc </i> <i>p</i> 1 2M<i>r</i> hoặc <i>q</i>2  1M<i>p .</i>


<i><b>Bài toán 3 (HSG lớp 9 Vĩnh Phúc 2009). Tìm tất cả các số nguyên dương n sao</b></i>


</div>
<span class='text_page_counter'>(21)</span><div class='page_container' data-page=21>

<i><b>Lời giải. Ta sẽ khai thác giả thiết là với mọi a lẻ thỏa mãn </b>a</i>2  đều có tính chất<i>n</i>
|



<i><b>a n để chặn được a. Muốn vậy ta gọi a là số nguyên dương lẻ lớn nhất thỏa mãn</b></i>


2


<i>a</i> <sub> , khi đó </sub><i>n</i> <i>n</i>(<i>a</i>2)2<sub>.</sub>


Nếu <i>a  thì </i>7 <i>a</i> 4;<i>a</i> 2;<i>a là 3 số lẻ chia hết n, mà các số này nguyên tố sánh đôi</i>
nên (<i>a</i> 4)(<i>a</i> 2) |<i>a n</i> (<i>a</i> 4)(<i>a</i> 2)<i>a</i>(<i>a</i>2)2  <i>a</i>3 7<i>a</i>2 4<i>a</i> 4 0 .


Dễ thầy điều này mẫu thuẫn với <i>a  . Do đó </i>7 <i>a </i>

1;3;5

.
Từ đó tìm ra <i>n </i>

1,2,3,4,5,6,7,8,9,12,15,18,21,24,30,45

.


<i><b>Nhận xét: Đây là dạng bài toán quen thuộc với học sinh THCS, bài tốn u cầu</b></i>


tìm các số nguyên thỏa mãn một quan hệ chia hết nào đó. Để giải quyết các bài
tốn dạng này ta thường khai thác tính chất chia hết để chặn các biến hoặc đánh giá
thêm tính chất số học cho các biến. Có thể sử dụng thêm định lý Fermat để xử lý
cho nhanh trong nhiều tình huống. Ta xét một vài bài toán tương tự.


<i><b>Bài toán 3.1. Cho p nguyên tố và a,n nguyên dương thỏa mãn 2</b>p</i> 3<i>p</i> <i>an</i><sub>. Chứng</sub>
minh rằng <i>n  .</i>1


<i><b>Hint. Với </b>p  thì </i>2 <i>an</i> 13 <i>n</i><sub> .</sub>1
Với <i>p  thì 5| (2 3 ) 5|</i>3 <i>p</i>  <i>p</i>  <i>an</i>  5 |<i>a</i>.


Nếu <i>n  thì </i>1



1 2 1 1


2 3



25 | 5 | 5 | 2 3.2 ... 3 5 | .2


2 3


<i>p</i> <i>p</i>


<i>n</i> <i>p</i> <i>p</i> <i>p</i> <i>p</i>


<i>a</i>     <i>p</i> 


     




Suy ra <i>p</i> 5 <i>an</i> 753 <i>n</i> (vơ lí). Bài tốn được chứng minh.1


<i><b>Bài tốn 3.2. Tìm tất cả n ngun dương sao cho khi xóa đi chữ số cuối cùng của</b></i>


<i>n ta được một số là ước của n.</i>


<i><b>Hint. Gọi b là chữ số cuối cùng của n và a là số thu được khi đã xóa b, ta có</b></i>


10


</div>
<span class='text_page_counter'>(22)</span><div class='page_container' data-page=22>

Nếu <i>b  thì n ln thỏa mãn.</i>0


Nếu <i>b  thì do |</i>0 <i>a b nên a là một chữ số của n. Khi đó các số n thỏa mãn là</i>
11, 12,…,19, 22, 24, 26, 28, 33, 36, 39, 44, 48, 55, 56, 77, 88, 99.



<i><b>Bài toán 3.3. Cho n là số nguyên dương chẵn và ,</b>a b là hai số nguyên dương</i>


nguyên tố cùng nhau. Tìm ,<i>a b biết an</i> <i>bn<sub> chia hết cho a b</sub></i><sub> .</sub>


<i><b>Hint. Ta có </b></i>



2 2 2 4 2 2


( ) ...


<i>n</i> <i>n</i> <i>n</i> <i>n</i> <i>n</i>


<i>a</i> <i>b</i> <i>a</i> <i>b</i> <i>a</i>  <i>a b</i> <i>b</i> 


     


Từ (<i>a b</i> ) | (<i>a</i>2  <i>b</i>2) (<i>a b</i> ) | (<i>an</i>  <i>bn</i>). Cùng với giả thiết suy ra (<i>a b</i> ) | 2<i>an</i> và
(<i><sub>a b</sub></i>) | 2<i><sub>b</sub>n</i>


 <sub>. Suy ra </sub>(<i>a b</i> ) | gcd 2 ,2

<i>an</i> <i>bn</i>

 (<i>a b</i> ) | 2 <i>a b</i> 1<sub>.</sub>


<i><b>Bài toán 3.4. Tìm tất cả các số nguyên dương n sao cho </b>n</i>2015   là một số<i>n</i> 1
nguyên tố.


<i><b>Hint. Đặt </b>P n</i> 2015  <i>n</i> 1


<i>Với n=1 thì P=3 là số nguyên tố, vậy n=1 thoả mãn</i>
Với <i>n  , ta có </i>1 <i>P n n</i> 2( 2013 1) ( <i>n</i>2  <i>n</i> 1).


Ta có <i>n</i>2013 1 ( ) <i>n</i>3 671 nên 1 <i>n</i>2013 chia hết cho 1 <i>n  , suy ra </i>3 1 <i>n</i>2013  chia hết1


cho <i>n</i>2   . <i>n</i> 1


<i>Vậy P chia hết cho n</i>2 <i>  . Trong trường hợp này P không là số nguyên tố. Vậyn</i> 1
<i>n=1 là giá trị duy nhất cần tìm.</i>


<i><b>Bài tốn 3.5. Tìm tất cả ,</b>x y nguyên dương để x</i>4 4<i>y</i>4 là số nguyên tố.


<i><b>Bài toán 3.6. Tìm tất cả các số nguyên tố , ,</b>x y z sao cho xy yz xz xyz</i>   .


</div>
<span class='text_page_counter'>(23)</span><div class='page_container' data-page=23>

<i><b>Lời giải. Ta có: </b>n</i>5 <i>n</i>4  1 <i>n</i>5 <i>n</i>4 <i>n</i>3  <i>n</i>3 <i>n</i>2  <i>n n</i> 2  <i>n</i> 1

 



2 <sub>1</sub> 3 <sub>1</sub>


<i>n</i> <i>n</i> <i>n</i> <i>n</i>


    


.


Chú ý là cả 2 nhân tử đều lớn hơn 1 nên <i>n</i>5 <i>n</i>4 <sub> không là số nguyên tố.</sub>1


<i><b>Nhận xét: Đây là dạng bài quen thuộc nhất trong số học. Để chứng minh một số là</b></i>


số nguyên tố hoặc hợp số ta thường cố gắng phân tích nhân tử và đánh giá vào các
ước số của nó, có những trường hợp ta có thể dùng phương pháp phản chứng. Một
số bài toán tương tự:


<i><b>Bài toán 4.1. Cho , ,</b>a b c nguyên khác 0 và a c</i> <sub> thỏa mãn </sub>



2 2
2 2


<i>a</i> <i>a</i> <i>b</i>
<i>c</i> <i>c</i> <i>b</i>





 <sub>. Chứng</sub>
minh rằng <i>a</i>2 <i>b</i>2 <i>c</i>2<sub> khơng là số ngun tố.</sub>


<i><b>Bài tốn 4.2 (42</b>nd<sub> IMO). Cho a b c d</sub></i><sub>  </sub> <sub> là các số nguyên dương thỏa mãn</sub>


( )( )


<i>ac bc</i> <i>b d a c b d a c</i>      <i><sub>. Chứng minh rằng ab cd</sub></i><sub></sub> <sub> không là số</sub>
nguyên tố.


<i><b>Bài toán 4.3. Cho , , ,</b>a b c d là các số nguyên dương thỏa mãn</i>


2 2 2 2


<i>a</i> <i>ab b</i> <i>c</i> <i>cd d</i> <i><sub>. Chứng minh rằng a b c d</sub></i>   <sub> là hợp số.</sub>


<i><b>Hint. Từ giả thiết ta suy ra </b></i>


2 2


(<i>a b</i> )  <i>ab</i>(<i>c d</i> )  <i>cd</i>  <i>ab cd</i> (<i>a b c d a b c d</i>   )(    )


<i>Phản chứng rằng p a b c d</i>    nguyên tố, ta có <i>ab cd</i> 0 (mod )<i>p</i> .


Mặt khác <i>d</i> (<i>a b c</i>  ) (mod )<i>p</i>  <i>ab c a b c</i> (   ) 0 (mod ) <i>p</i> , suy ra
(<i>c a c b</i> )(  ) 0 (mod ) <i>p</i> <sub> (*)</sub>


</div>
<span class='text_page_counter'>(24)</span><div class='page_container' data-page=24>

<i><b>Nhận xét: Tư tưởng rất hay và đẹp đẽ của bài toán nằm ở bước phản chứng, từ</b></i>


<i>bước này cho tam thêm giả thiết rằng p a b c d</i>    là số nguyên tố, từ đó cho
<i>ta các phép đánh giá theo mod p đơn giản mà hiệu quả. Nếu khơng có p nguyên tố</i>
như vậy thì các phép suy luận về chia hết là khó khăn hơn hẳn.


<b>Bài tốn 5. Tìm ước chung lớn nhất của các số </b><i>an</i> 23<i>n</i> 36<i>n</i> 2 56<i>n</i> 2


 


   <sub> với</sub>


0,1,...,2014


<i>n </i> <sub>.</sub>


<i><b>Lời giải. Ta có </b>a </i>0 35 5.7


Xét theo mod5 ta được <i>an</i> 23<i>n</i> 93 1<i>n</i> 23<i>n</i> ( 1)3 1<i>n</i> (mod5)


 


    


Với <i>n  , </i>1 <i>a  </i>1 9 0 (mod5), suy ra 5 không là một ước chung của các số <i>an</i>.



Xét theo mod7 ta được


3 3 3 3


8<i>n</i> 9.9 <i>n</i> 25.25 <i>n</i> 1 2.2 <i>n</i> 4.4 <i>n</i> 0 (mod7)


<i>n</i>


<i>a  </i>      <sub>.</sub>


Do đó <i>an chia hết cho 7 với mọi số tự nhiên n. Suy ra ước chung lớn nhất của của</i>


các số <i>a a</i>0, ,...,1 <i>a</i>2014 bằng 7.


<i><b>Bài tương tự: (IMO 2005). Cho dãy số </b>an</i> 2<i>n</i> 3<i>n</i> 6<i>n</i>  1 (<i>n</i> . Chứng minh1)


<i>rằng với mỗi số nguyên tố p luôn tồn tại một số hạng của dãy chia hết cho p.</i>


<i><b>Hint. Với </b>p </i>2,3 thì <i>a </i>2 48 thỏa mãn.


Với <i>p  , ta có </i>3 2 2 2 3 2 6 2 1 6 2 3.2 1 2.3 1 6 1 6


<i>p</i> <i>p</i> <i>p</i> <i>p</i> <i>p</i> <i>p</i>


<i>p</i> <i>p</i>


<i>a</i>    <i>a</i>   


          



Theo định lý Fermat, ta có 6<i>ap</i>2    3 2 1 6 0 (mod ) <i>p</i>  <i>p a</i>| <i>p</i>2 (đpcm).
<i><b>Nhận xét: Từ kết trên ta có thể giải được bài tốn sau: Tìm tất cả các số ngun</b></i>


dương mà chúng nguyên tố cùng nhau với mọi số hạng của dãy trên. Rõ ràng chỉ
có duy nhất số 1 thỏa mãn yêu cầu bài toán.


</div>
<span class='text_page_counter'>(25)</span><div class='page_container' data-page=25>

<i><b>KQ1. Cho số nguyên tố </b></i> <i>p</i>3<i>k . Chứng minh rằng với x, y là hai số nguyên ta</i>2


có <i>x</i>3 <i>y</i>3 (mod )<i>p</i>  <i>x y</i> (mod )<i>p</i> .


<i><b>Chứng minh. Giả sử </b>x</i>3 <i>y</i>3 (mod )<i>p</i> , ta chứng minh <i>x y</i> (mod )<i>p</i> .
Nếu <i>x p</i>M thì <i>y p</i>M , ta có điều phải chứng minh.


Nếu ( , ) ( , ) 1<i>x p</i>  <i>y p</i>  , theo định lý Fermat, ta có


1 <sub>1 (mod );</sub> 1 <sub>1 (mod )</sub> 3 1 <sub>1 (mod );</sub> 3 1 <sub>1 (mod )</sub>


<i>p</i> <i>p</i> <i>k</i> <i>k</i>


<i>x</i>  <i>p y</i>  <i>p</i> <i>x</i>  <i>p y</i>  <i>p</i>


     <sub>.</sub>


Lại có <i>x</i>3<i>k</i> <i>y</i>3<i>k</i>(mod )<i>p</i>  <i>x</i>3 1<i>k</i> <i>x y</i>. 3<i>k</i> (mod )<i>p</i>  <i>y</i>3 1<i>k</i> <i>x y</i>. 3<i>k</i> (mod )<i>p</i> .
Do đó <i>x y</i> (mod )<i>p</i> (đpcm).


<i><b>KQ2. Cho số nguyên tố </b></i> <i>p</i>3<i>k</i>  . Chứng minh rằng với ,2 <i>x y là hai số nguyên ta</i>





2 2 |


|


|
<i>p x</i>


<i>p x</i> <i>xy y</i>


<i>p y</i>


  <sub> </sub>


 <sub>.</sub>


Kết quả này được suy ra trực tiếp từ kết quả 1.


<i><b>KQ3. Cho số nguyên a. Chứng minh rằng mọi ước số nguyên tố lẻ của </b>a  đều</i>2 1


có dạng <i>p</i>4<i>k</i>  .1


Từ đó suy ra mọi ước nguyên dương lẻ của <i>a  đều có dạng 4</i>2 1 <i>k  .</i>1


<i><b>Chứng minh. Gọi p là ước nguyên tố lẻ bất kì của </b>a  , suy ra </i>2 1 <i>a</i>2 1 (mod )<i>p</i>


(1). Hiển nhiên ( , ) 1<i>a p  nên theo định lý Fermat thì ap</i>11 (mod )<i>p</i> .
<i>Nếu p có dạng p</i>4<i>k</i>  thì 3 <i>a</i>4<i>k</i>2 1 (mod )<i>p</i> (2).



Mặt khác từ (1) suy ra

 



2 1 <sub>2</sub> <sub>1</sub>


4<i>k</i> 2 2 <i>k</i> <sub>1</sub> <i>k</i> <sub>1 (mod )</sub>


<i>a</i> <sub></sub> <i>a</i>   <i>p</i>


   


</div>
<span class='text_page_counter'>(26)</span><div class='page_container' data-page=26>

<i><b>KQ4. Cho </b></i> 1


<i>uv</i>
<i>p</i>


<i>s</i>


 


là số nguyên tố lẻ, , ,<i>u v s nguyên dương, v lẻ. </i>
Chứng minh rằng khi đó <i>xu</i>  <i>y pu</i>M  <i>x p y p .</i>M, M


<i><b>Chứng minh. Giả sử </b>xu</i>  <i>y p và ( , ) ( , ) 1u</i>M <i>x p</i>  <i>y p</i>  .


Theo định lý Fermat, ta có <i>xp</i>1 1 (mod );<i>p yp</i>11 (mod )<i>p</i> .


Mặt khác <i>xuv</i>  1<i>xs p</i>( 1)  1M<i>xp</i>1 1;<i>yuv</i>  1M<i>yp</i>1 1, do đó <i>xuv</i>  <i>yuv</i>  2M<i>p (1)</i>
<i>Mà v lẻ nên xuv</i>  <i>yuv</i>M<i>xu</i>  <i>yu</i>  <i>xuv</i>  <i>yuv</i>M<i>p (2).</i>


Từ (1) và (2) suy ra 2Mp, vơ lí. Vậy <i>x p y p</i>M, M (đpcm).



<i><b>KQ5. Giả sử </b></i> <i>p</i>2 .<i>tk</i>  là số nguyên tố lẻ với ,1 <i>k t là các số tự nhiên, k lẻ.</i>


Chứng minh rằng với hai số tự nhiên ,<i>x y ta có </i> 2  2 M  M M;


<i>t</i> <i>t</i>


<i>x</i> <i>y</i> <i>p</i> <i><sub>x p y p .</sub></i>


<i><b>Chứng minh. Giả sử </b></i> 2  2 M


<i>t</i> <i>t</i>


<i>x</i> <i>y</i> <i><sub>p . Chỉ cần xét ( , ) ( , ) 1</sub>x p</i>  <i>y p</i> <sub> . </sub>
Theo định lý Fermat ta có <i>xp</i>11 (mod )<i>p</i> và <i>yp</i>1 1 (mod )<i>p</i>
Suy ra 2 . 1 (mod )


<i>t<sub>k</sub></i>


<i>x</i>  <i>p</i> <sub> và </sub><i>y</i>2 .<i>tk</i> 1 (mod )<i>p</i>  <i>x</i>2 .<i>tk</i>  <i>y</i>2 .<i>tk</i> 2 (mod )<i>p</i> <sub> (*)</sub>
Theo giả thiết 2  2 M


<i>t</i> <i>t</i>


<i>x</i> <i>y</i> <i><sub>p và k lẻ nên </sub><sub>x</sub></i>2 .<i>tk</i> <sub></sub><i><sub>y</sub></i>2 .<i>tk</i><sub>M</sub>


<i>p , mâu thuẫn với (*). </i>
Vậy ,<i>x y đều chia hết cho p.</i>


Đặc biệt hóa khi cho <i>t</i> 1<sub> và </sub><i>t</i>2<sub> ta thu được các kết quả sau:</sub>



<i><b>KQ5.1. Cho số nguyên tố dạng </b>p</i>4<i>k</i>3. Chứng minh rằng nếu các số tự nhiên
,


<i>x y thỏa mãn </i> 2 2


 M


<i>x</i> <i><sub>y p thì x và y đều chia hết cho p.</sub></i>


</div>
<span class='text_page_counter'>(27)</span><div class='page_container' data-page=27>

Trên đây là 5 kết quả rất quan trọng và có ứng dụng rất mạnh với những bài
toán về đồng dư chia hết. Xin đưa ra một vài bài toán áp dụng minh họa:


<i><b>Bài toán 7.1. Cho đa thức </b>P x</i>( )<i>x</i>33<i>x</i>2 104<i>x</i>2015<i> và a,b là hai số nguyên</i>
bất kì. Chứng minh rằng nếu ( )<i>P a</i> <i>P b</i>( ) (mod101) thì <i>a b</i> (mod101).


<i><b>Hint. Ta viết </b>P x</i>( ) ( <i>x</i>1)3101<i>x</i>2014, chú ý là 101 là số nguyên tố dạng
3<i>k </i>2


Do đó <i>P a</i>( )<i>P b</i>( ) (mod101) (<i>a</i>1)3 (<i>b</i>1) (mod101)3  <i>a b</i> (mod101)


<i><b>Bài toán 7.2. Cho đa thức </b>P x</i>( )<i>x</i>314<i>x</i>2  2<i>x</i>2005<i> và a, b là hai số nguyên</i>
bất kì. Chứng minh rằng nếu ( )<i>P a</i> <i>P b</i>( ) (mod101) thì <i>a b</i> (mod101).


<i><b>Bài toán 7.3. Cho x, y nguyên dương sao cho </b></i>


4 4


1997.1999
<i>x</i> <i>y</i>


<i>A</i> 


là số nguyên. Tìm
<i>giá trị nhỏ nhất của A. </i>


<b>Bài toán 9. Cho , ,</b><i>x y p là các số nguyên, p  thỏa mãn điều kiện </i>1 <i>x</i>2014M<i>p y</i>; 2014M<i>p</i>


<i>. Chứng minh rằng 1 x y  không chia hết cho p.</i>


<i><b>Phân tích bài tốn . Rõ ràng cách hỏi của bài toán là khá đơn giản tuy nhiên đây</b></i>


lại là bài tốn khó nếu khơng hiểu rõ ý nghĩa của phép chia hết.


Từ giải thiết <i>x</i>2014M<i>p y</i>; 2014M<i>p và p là số nguyên gần như chẳng cho ta điều gì cả, ta</i>
<i>ước rằng nếu p là số nguyên tố, bởi lẽ nếu p nguyên tố thì từ x</i>2014M<i>p y</i>; 2014M<i>p sẽ suy</i>
ra ngay <i>x y p</i>, M <i>. Từ đó suy ra 1 x y  không chia hết cho p.</i>


</div>
<span class='text_page_counter'>(28)</span><div class='page_container' data-page=28>

<i>cũng suy ra 1 x y  không chia hết cho p. Đến đây mọi thứ đã quá rõ ràng và bài</i>
toán được giải quyết ngắn gọn.


<i><b>Ta rút ra một điều rất căn bản nhưng quan trọng: "Để chứng minh a</b></i>


<i>không chia hết cho b, ta chỉ cần chứng minh a không chia hết cho một ước nguyên</i>
<i>tố nào đó của b".</i>


<b>Bài tốn 10. Cho ,</b><i>x y nguyên khác 1</i> <sub> thỏa mãn </sub>


3 <sub>1</sub> 3 <sub>1</sub>


1 1



<i>x</i> <i>y</i>


<i>y</i> <i>x</i>


 




  <sub> là số nguyên.</sub>


Chứng minh rằng <i>x</i>2016  chia hết cho 1 <i>y  .</i>1


Lời giải. Đặt


3 <sub>1</sub> 3 <sub>1</sub>


;
1 1
<i>x</i> <i>y</i>
<i>a</i> <i>b</i>
<i>y</i> <i>x</i>
 
 


  <i><sub> thì a,b là số hữu tỉ và a b</sub></i><sub> nguyên.</sub>


Mặt khác


3 3



2 2


1 1


. ( 1)( 1)


1 1


<i>x</i> <i>y</i>


<i>ab</i> <i>x</i> <i>x</i> <i>y</i> <i>y</i>


<i>y</i> <i>x</i>


 


     


  <sub> là số nguyên nên ,</sub><i>a b là</i>


nghiệm hữu tỷ của phương trình hệ số nguyên <i>t</i>2  (<i>a b t ab</i> )   . Từ đó suy ra0
,


<i>a b nguyên. Do đó </i> 3 <sub>1</sub> <sub>1</sub> 6 <sub>1</sub> <sub>1</sub>


 M   M 


<i>x</i> <i>y</i> <i>x</i> <i>y</i> <sub>. </sub>



Vậy

 



336


2016 <sub>1</sub> 6 <sub>1</sub> 6 <sub>1</sub> <sub>1</sub>


   M  M 


<i>x</i> <i>x</i> <i>x</i> <i>y</i>


(đpcm).


<i>Có thể tổng quát với mọi n nguyên dương chia hết cho 6 ta có xn</i>  1M<i>y</i>1.


<i><b>Bài tương tự: Cho ,</b>x y nguyên khác 1</i> <sub> sao cho </sub>


4 <sub>1</sub> 4 <sub>1</sub>


1 1


<i>x</i> <i>y</i>


<i>y</i> <i>x</i>


 




  <sub> là số nguyên.</sub>



Chứng minh rằng <i>x y  chia hết cho 1</i>4 44 1 <i>x  .</i>


<b>D. BÀI TẬP TỰ LUYỆN.</b>


<b>Bài 1. </b><i>Tìm tất cả các số tự nhiên n sao cho n </i>1; <i>n </i>3; <i>n </i>7; <i>n </i>9; <i>n </i>13 và
15


<i>n </i> <sub> đều là số nguyên tố.</sub>


</div>
<span class='text_page_counter'>(29)</span><div class='page_container' data-page=29>

<b>Bài 3. </b>Chứng minh rằng <i>n</i>7  <i>n</i>M42<i><sub> với mọi số n nguyên dương.</sub></i>


<b>Bài 4. Cho , ,</b><i>a b c nguyên dương thỏa mãn a b c</i>  M30<sub>. Chứng minh rằng </sub>


5 5 5 <sub>30</sub>


  M


<i>a</i> <i>b</i> <i>c</i> <sub>.</sub>


<b>Bài 5. </b>Chứng minh rằng 53<i>n</i> 7 12M <i><sub> với mọi số n nguyên dương</sub></i>


<b>Bài 6. </b><i>Giả sử n là số tự nhiên không chia hết cho 17. Chứng minh rằng hoặc</i>


8 <sub>1 17</sub>


 M


<i>n</i> <sub> hoặc </sub> 8 <sub>1 17</sub>


 M



<i>n</i> <sub>.</sub>


<b>Bài 7. </b><i>Với số nguyên n nào ta có </i>12 22 ... ( <i>n</i> 1)2M<i>n</i>.


<b>Bài 8. </b><i>Chứng minh rằng nếu p và </i>8<i>p </i>2 1 là số nguyên tố thì 8<i>p</i>2 2<i>p</i>1 là số
nguyên tố.


<i><b>Bài 9. Chứng minh rằng không tồn tại số tự nhiên n sao cho </b></i>2010<i>n</i> 1<sub> chia hết cho</sub>
1010<i>n</i> 1


 <sub>. </sub>


<i><b>Bài 10. Tìm tất cả các số nguyên tố p sao cho </b>p </i>2 11 có đúng 6 ước số nguyên
dương.


<i><b>Bài 11. Tìm tất cả các số nguyên tố p sao cho hệ phương trình sau có nghiệm</b></i>


nguyên:


2


2 2


2 1


2 1


<i>x</i> <i>p</i>
<i>y</i> <i>p</i>



  





 





<i><b>Bài 12. Chứng minh rằng với mọi số nguyên m đều tồn tại một số nguyên n sao</b></i>


cho: <i>n</i>3 11<i>n</i>2 87<i>n m</i> <sub> chia hết cho 191.</sub>


<b>Bài 13. Cho hai số nguyên ,</b><i>a b thoả mãn </i>24<i>a</i>2  1 <i>b</i>2<sub>. Chứng minh rằng có một</sub>
và chỉ một trong hai số ,<i>a b chia hết cho 5.</i>


<b>Bài 14. Tìm tât cả các số nguyên tố p, q sao cho (5</b>p<sub> – 2</sub>p<sub>)(5</sub>q<sub> – 2</sub>q<sub>) chia hết cho pq. </sub>


</div>
<span class='text_page_counter'>(30)</span><div class='page_container' data-page=30>

a) <i>4xy x y z</i>   2 (Euler)


b) <i>x</i>2  <i>y</i>3 7 (Lebesgue)


<b>Bài 16. Chứng minh rằng m</b>2<sub> – n</sub>2<sub> là số nguyên tố thì m, n là 2 số tự nhiên liên tiếp.</sub>
<i><b>Bài 17. Cho , ,</b>a b c nguyên dương thỏa mãn ab cd</i> <sub>. Chứng minh rằng </sub>


<i>n</i> <i>n</i> <i>n</i> <i>n</i>


<i>A a</i> <i>b</i> <i>c</i> <i>d</i> <i><b><sub> là hợp số với mọi n tự nhiên. </sub></b></i>



<i><b>Bài 18. Chứng minh rằng </b>A n</i> 44<i>n<sub> luôn là hợp số với mọi n nguyên dương.</sub></i>


<b>Bài 19. Cho ,</b><i>a b là các hai số nguyên và p là số nguyên tố lẻ. Chứng minh rằng</i>


nếu <i>p là ước số của </i>4 <i>a</i>2<i>b</i>2<sub> và </sub><i>a a b thì </i>(  )2 <i>p cũng là ước của (</i>4 <i>a a b</i> )


</div>
<span class='text_page_counter'>(31)</span><div class='page_container' data-page=31>

<b>TÀI LIỆU THAM KHẢO</b>


<i><b>1. Bài giảng số học, Đặng Hùng Thắng- NXB Giáo Dục Việt Nam.</b></i>
<i><b>2. Number Theory, Titu Andreescu-Dorin Andrica.</b></i>


</div>
<span class='text_page_counter'>(32)</span><div class='page_container' data-page=32>

<b>XÂY DỰNG VÀ SỬ DỤNG HỆ THỐNG CÂU HỎI, BÀI TẬP ĐỂ </b>


<b>DẠY HỌC NỘI DUNG CƠ SỞ VẬT CHẤT VÀ CƠ CHẾ DI </b>



<b>TRUYỀN Ở CẤP ĐỘ PHÂN TỬ</b>



<b>Trần Thị Dung – THPT Chuyên Vĩnh Phúc</b>
<b>CÁC THUẬT NGỮ VIẾT TẮT</b>


1. đvC : đơn vị Cacbon
2. GV : giáo viên
3. HS : học sinh
4. HSG : học sinh giỏi


5. NTBS : nguyên tắc bổ sung
6. nu : nucleotit


7. mtcc : môi trường cung cấp
8. THCS : trung học cơ sở


<b>I. CƠ SỞ XÂY DỰNG CHUYÊN ĐỀ</b>
<b>1.1. Cơ sở lý luận</b>


<i><b>* Xuất phát từ yêu cầu về đổi mới phương pháp dạy học</b></i>


Giáo dục phải đặt trọng tâm vào việc khơi dậy sự say mê học tập, kích thích
sự tò mò và sáng tạo của học sinh để các em có khả năng kiến tạo kiến thức từ
những gì nhà trường mang đến cho các em. Giáo dục hiện nay đang chuyển hướng
từ dạy học truyền thống – đặt trọng tâm vào việc truyền thụ các tri thức khoa học
cho người học thông qua giáo viên, sang một nền giáo dục tiên tiến – giúp cho
người học nhận ra được những năng lực trí tuệ của mình để đi tìm tiếp lời giải cho
những vấn đề chưa hẳn hồn toàn đã biết theo con đường phù hợp nhất với năng
lực trí tuệ của cá nhân.


<i><b>* Xuất phát từ vai trò của câu hỏi, bài tập trong dạy học Sinh học 9 </b></i>


</div>
<span class='text_page_counter'>(33)</span><div class='page_container' data-page=33>

<i><b>* Xuất phát từ đặc điểm nội dung phần Cơ sở vật chất và cơ chế di truyền ở cấp</b></i>
<i><b>độ phân tư bao gồm cấu trúc, các hoạt động liên quan đến ADN, ARN, protein và</b></i>
các biến đổi của ADN. Đây là những nội dung tương đối phức tạp, đòi hỏi khả
năng tư duy, làm việc tích cực của học sinh tuy nhiên trong q trình học tập học
sinh cịn thụ động trong việc tiếp thu kiến thức.


Muốn phát triển năng lực sáng tạo, khả năng tư duy của học sinh, việc dạy
học phải chuyển từ việc ghi nhớ nội dung kiến thức sang việc tự thực hiện các quá
trình học tập nhằm kiến tạo tri thức khoa học thông qua quan sát, phân tích, tổng
hợp, khái qt hóa, hình thành giả thuyết, đánh giá…


<b>1.2. Cơ sở thực tiễn</b>


<i><b>* Xuất phát từ thực trạng dạy học Sinh học hiện nay ở các trường THCS </b></i>



- Giáo viên chủ yếu dạy học theo kiểu truyền thống: truyền thụ kiến thức cho
học sinh theo một chiều và ít sử dụng các câu hỏi yêu cầu cao về tư duy hoặc xây
dựng một hệ thống các câu hỏi, bài tập về một chủ đề học tập theo một logic.


- Học sinh chưa tích cực tự đọc tài liệu, tự nghiên cứu và tư duy sáng tạo
trong quá trình dạy học. Đặc biệt, học sinh thi HSG Sinh học thường bị đánh giá là
kém thông minh hơn các học sinh thi HSG Tốn, Vật lý, Hóa học vì vậy khả năng
độc lập và sức “ì” của các em cũng lớn hơn.


Xuất phát từ những lý do trên, chúng tôi lựa chọn và xây dựng chuyên đề
<i><b>“Xây dựng và sư dụng hệ thống câu hỏi, bài tập để dạy học nội dung cơ sở vật</b></i>
<i><b>chất và cơ chế di truyền ở mức độ phân tư, Sinh học 9 - THCS”</b></i>


<b>II. NỘI DUNG CHUYÊN ĐỀ</b>
<b>2.1. Tổng quan về câu hỏi, bài tập</b>
<b>2.1.1. Khái niệm câu hỏi, bài tập</b>
<i><b>2.1.1.1. Khái niệm câu hỏi</b></i>


Câu hỏi là một mệnh đề trong đó chứa đựng cái đã biết và cái cần tìm. Câu
hỏi thường có 2 vế, một vế chứa động từ nghi vấn và một vế chứa nội dung cần trả
lời.


<i><b>2.1.1.2. Khái niệm bài tập</b></i>


</div>
<span class='text_page_counter'>(34)</span><div class='page_container' data-page=34>

Theo các nhà lý luận học Liên Xơ cũ thì bài tập đó là một dạng bài làm gồm
những bài toán, những câu hỏi hay đồng thời cả bài toán và câu hỏi mà khi hoàn
thành chúng, học sinh lĩnh hội được một tri thức hay một kĩ năng nhất định hoặc
hoàn thiện chúng.



Về thành phần cấu tạo, bài tập có điểm giống câu hỏi là chứa đựng điều đã
biết và điều cần tìm, điều cần tìm là điều dựa vào điều đã biết, điều cần tìm và điều
đã biết quan hê chặt chẽ với nhau, từ những điều đã biết ta có thể dùng phép biến
đổi tương đương để dẫn đến những điều cần tìm. Nhưng mối quan hệ giữa điều đã
biết với những điều cần tìm chặt chẽ hơn mối quan hệ giữa điều đã biết và điều cần
tìm trong câu hỏi ở chỗ: Những điều đã biết trong bài tập phải vừa đủ để người
thực hiện bài tập chỉ biến đổi những điều đã biết bằng những đại lượng tương
đương ắt sẽ dẫn đến kết luận. Từ đó ta có thể hiểu bản chất của việc giải bài tập là
sự thực hiện phép biến đổi tương đương, để chứng minh rằng điều đã cho và điều
cần tìm là hoàn toàn phù hợp.


<b>2.1.2. Cấu trúc câu hỏi, bài tập</b>


Mỗi câu hỏi, bài tập đều có hai thành phần là điều đã biết và điều cần tìm,
chúng có quan hệ với nhau về mặt cấu trúc, tuy nhiên thành phần nào đứng trước
thành phần nào đứng sau khơng địi hỏi nghiêm ngặt mà phụ thuộc vào tư duy
logic của con người.


Phần thứ nhất (điều đã biết) là tài liệu có tính chất “ngun liệu” bao gồm:
- Đoạn tư liệu trong SGK.


- Đoạn tư liệu trích trong các tài liệu tham khảo.
- Các tập hợp từ, cụm từ cho trước.


- Các thơng tin gợi ý cho trước.
- Các hình vẽ cho trước.


- Các thí nghiệm và kết quả cho trước.


Phần thứ 2 (điều chưa biết) là các câu hỏi hướng dẫn học sinh hoạt động tư


duy, xử lý các tư liệu đã có bao gồm:


- Tóm tắt nội dung, lập sơ đồ hệ thống hóa.


</div>
<span class='text_page_counter'>(35)</span><div class='page_container' data-page=35>

- Điền từ, cụm từ, đoạn thơng tin thích hợp vào bảng, vào ô trống, vào hình
vẽ.


- Mô tả hình vẽ, ghi chú thích cho hình vẽ, phân tích tìm nội dung cơ bản
qua hình vẽ.


- Phát biểu tính quy luật của các hiện tượng.
- Lập bảng so sánh.


- Giải thích thí nghiệm.
- Xác định mối quan hệ.


- Xác định ý nghĩa hay giá trị của kiến thức.


<b>2.1.3. Các nguyên tắc xây dựng câu hỏi, bài tập để đáp ứng dạy học theo</b>
<b>hướng dạy học phát huy tính tích cực của học sinh</b>


- Quán triệt mục tiêu dạy học: để có được câu hỏi, bài tập tốt thì giáo viên
cần dựa vào mục tiêu phải đạt, nói cách khác là dựa vào cái cần học.


- Câu hỏi, bài tập phải phát huy tính tích cực. Muốn làm được như vậy thì
câu hỏi, bài tập phải đảm bảo tính vừa sức, tính kế thừa và phát triển phù hợp
với tâm sinh lý lứa tuổi của học sinh nhằm phát huy tính tự giác, tích cực và
sáng tạo đặc biệt với HS giỏi cần có những câu hỏi, bài tập mang tính phân hóa
để phát triển tư duy của các em.



- Câu hỏi, bài tập phải đảm bảo tính chính xác, khoa học. Do câu hỏi, bài
tập dùng để mã hóa nội dung bài học nên chúng cần được hoàn thiện và bổ
sung đảm bảo tính chính xác, khoa học. Đây chính là một điều kiện để các câu
hỏi, bài tập đáp ứng được mục tiêu dạy học.


- Câu hỏi, bài tập phải đảm bảo nguyên tắc hệ thống. Tính hệ thống này
được quy định bởi chính nội dung khoa học phản ánh đối tượng khách quan có
tính hệ thống, bởi logic hệ thống trong hoạt động tư duy của học sinh và bởi
bản chất logic của câu hỏi, bài tập. Vì vậy, từng câu hỏi, bài tập khi đưa vào sử
dụng phải được sắp xếp theo một logic hệ thống cho từng nội dung trong SGK,
cho một bài, cho một chương trình, đồng thời cần có câu hỏi phản ánh tính hệ
thống của nội dung học tập.


- Câu hỏi, bài tập phải đảm bảo tính thực tiễn, đặc biệt đối với mơn Sinh
học để học sinh rèn luyện được khả năng vận dụng kiến thức của mình.


</div>
<span class='text_page_counter'>(36)</span><div class='page_container' data-page=36>

+ Phải chứa đựng các mâu thuẫn nhận thức để học sinh luôn ở trạng thái có
nhu cầu giải quyết mâu thuẫn nhận thức đó.


+ Phải phù hợp với nội dung cơ bản của từng bài, từng chương để sau khi trả
lời học sinh có thể lĩnh hội được kiến thức trọng tâm của bài.


+ Phải đảm bảo cho học sinh có đủ tri thức hay nguồn tài liệu tra cứu, gia cơng
tìm lời giải.


+ Trong mỗi bài học, câu hỏi, bài tập đưa ra phải đảm bảo nguyên tắc đi từ dễ
đến khó, điều đó sẽ tạo sự thích thú cho học sinh để tiếp tục nghiên cứu tìm
lời giải đáp cho câu hỏi, bải tập tiếp theo.


+ Câu hỏi, bài tập phải có nội dung u cầu ngắn gọn, rõ ràng, chính xác.



+ Câu hỏi, bài tập phải mang tính hệ thống, phù hợp với logic cấu trúc của bài,
của chương sao cho khi trả lời học sinh thu nhận được kiến thức có hệ thống
theo logic xác định.


+ Câu hỏi, bài tập phải có nhiều khả năng huy động tính sáng tạo, chủ động
của học sinh nghĩa là câu hỏi, bài tập phải vừa sức, khơng q khó, khơng
q dễ mà phải phù hợp với năng lực nhận thức của học sinh.


+ Câu hỏi, bài tập khơng phải mang tính chất đơn thuần là trình bày kiến thức
từ SGK mà câu hỏi, bài tập phải có u cầu phân tích, giải thích hay chứng
minh cho các kiến thức mà học sinh đọc được từ SGK hay các tài liệu tham
khảo khác. Theo thang phân loại mức độ nhận thức của Bloom, có 6 mức độ
nhận thức từ thấp đến cao là:


Biết  Hiểu  Vận dụng  Phân tích  Tổng hợp  Đánh giá


Do đó câu hỏi, bài tập giáo viên lựa chọn đối với học sinh giỏi cần chú trọng
đến các cấp độ nhận thức cao để phát huy tính tư duy, sáng tạo của học sinh.


<b>2.1.4. Vai trò của câu hỏi, bài tập trong dạy học</b>


- Là sản phẩm của quá trình tư duy, chứa đựng kiến thức khoa học, chứa đựng
thế năng tâm lý, tạo động lực thúc đẩy sự tìm tịi, sáng tạo.


- Là mơ hình định hướng việc dạy và việc học.


</div>
<span class='text_page_counter'>(37)</span><div class='page_container' data-page=37>

- Định hướng nghiên cứu SGK tìm ra kiến thức mới, hướng dẫn ơn tập, củng
cố, hồn thiện kiến thức, là công cụ cơ bản để kiểm tra, đánh giá  từ đó điều
chỉnh việc dạy và việc học.



- Tạo năng lực dạy học sáng tạo, định hướng mang tính cấp bách hiện nay trên
tồn cầu  huy động năng lực cao từ người học.


<b>2.2. Logic nội dung kiến thức phần cơ sở vật chất và cơ chế di truyền ở cấp độ</b>
<b>phân tử – Sinh học 9, THCS</b>


- Phần Di truyền và biến dị chương trình Sinh học 9 – THCS được viết theo
logic: nêu các quy luật Menđen, sau đó giải thích các quy luật này rồi mới
nghiên cứu về cơ sở vật chất và cơ chế di truyền ở cấp tế bào (nhiễm sắc thể,
vận động của NST trong nguyên phân, giảm phân, quy luật di truyền liên kết
giữa các gen nằm trên cùng 1 NST, di truyền giới tính), cơ sở vật chất và cơ
chế di truyền ở cấp phân tử (cấu trúc ADN, gen) và các biến đổi liên quan đến
ADN, NST.


- Tuy nhiên, logic như vậy sẽ gây khó khăn cho học sinh trong quá trình lĩnh
hội tri thức, theo ý kiến của tác giả, nên tách riêng cơ sở vật chất và cơ chế di
truyền ở cấp độ phân tử, cấp độ tế bào riêng bao gồm cả cấu trúc, chức năng
và biến dị ở 2 cấp độ này.


- Như vậy, nội dung kiến thức phần cơ sở vật chất và cơ chế di truyền ở cấp độ
phân tử – Sinh học 9, THCS sẽ bao gồm từ bài 15 (ADN) đến bài 21 (Đột
biến gen).


+ Bài 15: ADN


+ Bài 16: ADN và bản chất của gen
+ Bài 17: Mối quan hệ giữa gen và ARN.
+ Bài 18: Protein.



+ Bài 19: Mối quan hệ giữa gen và tính trạng.


+ Bài 20: Thực hành: quan sát và lắp mơ hình ADN.
+ Bài 21: Đột biến gen.


<b>2.3. Hệ thống câu hỏi, bài tập để dạy học nội dung cơ sở vật chất và cơ chế di </b>
<b>truyền ở cấp độ phân tử</b>


</div>
<span class='text_page_counter'>(38)</span><div class='page_container' data-page=38>

Câu 1: Trình bày đặc điểm cấu trúc hóa học của phân tử ADN mạch kép?


Câu 2: Mô tả cấu trúc không gian của ADN? Hệ quả của NTBS được thể hiện ở
những điểm nào?


Câu 3: Mô tả quá trình nhân đơi của ADN?


Câu 4: Nêu bản chất hóa học và chức năng của gen?
Câu 5: Trình bày cấu trúc của phân tử ARN?


Câu 6: Mơ tả q trình tổng hợp mARN?


Câu 7: Trình bày cấu trúc hóa học và cấu trúc không gian của phân tử protein?
Câu 8: Nêu mối quan hệ giữa gen và ARN, giữa ARN và protein?


Câu 9: NTBS được biểu hiện trong mối quan hệ ở sơ đồ dưới đây như thế nào?


Gen (một đoạn ADN) mARN Protein


Câu 10: Nêu bản chất của mối quan hệ giữa gen và tính trạng qua sơ đồ:
3



Gen (một đoạn ADN) mARN Protein Tính


trạng


Câu 11: Đột biến gen là gì? Cho ví dụ?


Câu 12: Hãy tìm thêm một số ví dụ về đột biến gen phát sinh trong tự nhiên hoặc
do con người tạo ra?


<b>2.3.1.2. Mức độ hiểu</b>


Câu 1: Vì sao ADN có cấu tạo rất đa dạng và đặc thù?


Câu 2: Một đoạn mạch đơn của phân tử ADN có trình tự sắp xếp như sau:
- A – T – G – X – T – A – G – T – X –


Hãy viết đoạn mạch đơn bổ sung với nó?


Câu 3: Giải thích vì sao 2 ADN con được tạo ra qua cơ chế nhân đơi lại giống
ADN mẹ?


Câu 4: Một đoạn mạch ADN có cấu trúc như sau:
Mạch 1: - A – G – T – X – X – T –


Mạch 2 : - T – X – A – G – G – A -


Viết cấu trúc của 2 đoạn ADN con được tạo thành sau khi đoạn mạch ADN mẹ nói
trên kết thúc q trình tự nhân đơi ?


Câu 5 : Một đoạn mạch của gen có cấu trúc như sau :


Mạch 1: - A – G – T – X – X – T –


1 2


</div>
<span class='text_page_counter'>(39)</span><div class='page_container' data-page=39>

Mạch 2 : - T – X – A – G – G – A -


Xác định trình tự các đơn phân của đoạn mạch ARN được tổng hợp từ mạch 2 ?
Câu 6 : Một đoạn mạch ARN có trình tự các nucleotit như sau :


- A – U – G – X – U – U – G – A – X –


Xác định trình tự các nucleotit trong đoạn gen đã tổng hợp ra đoạn mạch ARN
trên ?


Câu 7 : Vì sao nói protein là đại phân tử hữu cơ có tính đa dạng và đặc thù ?


Câu 8 : Tại sao đột biến gen thường có hại cho bản thân sinh vật ? Nêu vai trò và ý
nghĩa của đột biến gen trong thực tiễn sản xuất ?


Câu 9 : Theo NTBS thì về mặt số lượng đơn phân những trường hợp nào sau đây
là đúng ?


a. A + G = T + X
b. A = T ; G = X


c. A + T + G = A + X + T
d. A + X + T = G + X + T


Câu 10 : Bậc cấu trúc nào sau đây có vai trị chủ yếu xác định tính đặc thù của
protein ?



a. Cấu trúc bậc 1
b. Cấu trúc bậc 2
c. Cấu trúc bậc 3
d. Cấu trúc bậc 4


Câu 11 : Protein thực hiện được chức năng của mình chủ yếu ở những bậc cấu trúc
nào sau đây ?


a. Cấu trúc bậc 1


b. Cấu trúc bậc 1 và bậc 2


c. Cấu trúc bậc 2 và bậc 3 và bậc 4
d. Cấu trúc bậc 1, bậc 3 và bậc 4


Câu 12 : loại ARN nào sau đây có chức năng truyền đạt thông tin di truyền ?
a. tARN


b. mARN
c. rARN


d. Cả 3 loại ARN trên


<b>2.3.1.3. Mức độ vận dụng, phân tích, tởng hợp, đánh giá</b>


</div>
<span class='text_page_counter'>(40)</span><div class='page_container' data-page=40>

Câu 2 : Giải thích nguyên tắc bổ sung và nguyên tắc bán bảo tồn trong q trình
nhân đơi ADN ? Ý nghĩa của q trình nhân đơi ADN ?


Câu 3 : Hãy giải thích vì sao trên mỗi chạc chữ Y chỉ có một mạch của phân tử


ADN được tổng hợp liên tục, mạch còn lại được tổng hợp một cách gián đoạn ?
Câu 4 : Một đoạn gen có trình tự các nucleotit như sau :


3’ XGA GAA TTT XGA 5’ (mạch mã gốc)
5’ GXT XTT AAA GXT 3’


Hãy xác định trình tự chuỗi mARN được tổng hợp từ gen trên ?
Câu 5 : So sánh cấu trúc phân tử ADN mạch kép và phân tử ARN ?


Câu 6 : So sánh q trình nhân đơi ADN và phiên mã tổng hợp mARN ở sinh vật
nhân thực ?


Câu 7 : Phân tích thành phần hóa học của một axit nucleic cho thấy tỉ lệ các loại
nucleotit như sau :


A = 20% ; G = 35% ; T = 20%. Axit nucleic này là loại nào ? Giải thích ?
Câu 8 : Xét 5 phân tử axit nucleic có tỉ lệ các loại đơn phân như sau :


Phân
tử I


A = 21% G = 29% T = 21% X = 29% U = 0%


Phân tử II A = 29% G = 21% T = 29% X = 21% U = 0%
Phân tử III A = 21% G = 21% T = 29% X = 29% U = 0%
Phân tử IV A = 21% G = 29% T = 0% X = 29% U = 21%
Phân tử V A = 21% G = 29% T = 0% X = 21% U = 29%
Hãy cho biết :


a. Những phân tử nào thuộc loại ADN và những phân tử nào thuộc loại ARN ?


Giải thích ?


b. Những phân tử ADN nào là kép có 2 mạch ? Phân tử nào chỉ có một mạch ?
Giải thích ?


Câu 9


a. So sánh đơn phân cấu tạo ADN và ARN?


b. Cấu trúc các loại ARN? Từ đó dự đốn thời gian tồn tại của các loại ARN
trong tế bào?


Câu 10. Sau đây là 3 đoạn mạch chứa thông tin di truyền trong nhân tế bào. Hãy
gọi tên, đánh dấu chiều của 3 đoạn mạch, hoàn chỉnh các đơn phân của từng mạch?
G..?...?....T….?....A…..?...?


</div>
<span class='text_page_counter'>(41)</span><div class='page_container' data-page=41>

?....?..G...A…?...U….X…..?


Câu 11 : So sánh ADN của sinh vật nhân sơ và sinh vật nhân thực ?


Câu 12 : Tại sao cùng là thịt nhưng khi ăn thịt gà, thịt bị, thịt lợn lại có vị khác
nhau ?


Câu 13 : Giải thích nguyên nhân của bệnh tiểu đường ?


Câu 14 : Khi thiếu hụt gluxit và lipit, cơ thể sẽ làm gì để tạo được năng lượng duy
trì sự sống ?


Câu 15 : Tại sao khi đun nóng hoặc thay đổi độ pH thì có thể làm thay đổi chức
năng của protein ?



Câu 16 : Dạng đột biến điểm nào làm thay đổi cấu trúc của gen nhưng có thể
khơng dẫn tới sự thay đổi chuỗi polypeptit do gen đó quy định ? Giải thích ?


Câu 17 : Trong trường hợp 1 gen quy định 1 tính trạng, khi nào gen đột biến dễ
biểu hiện ra kiểu hình ngay và khi nào gen đột biến khó biểu hiện ra kiểu hình
nhất ?


<i>2.3.2. Hệ thống các dạng bài tập tính tốn</i>


<b>2.3.2.1. Các bài tập liên quan đến ADN và cơ chế tự nhân đôi của ADN</b>
a. Bài tập về cấu tạo ADN


<b>DẠNG 1: Tính sớ lượng Nu của Gen (ADN).</b>
<b>* Tính sớ lượng Nu của từng Gen.</b>


<i>- Xét trên 1 mạch đơn của Gen.</i>


+ Gọi A1, T1, G1, X1 lần lượt là số nu từng loại của mạch 1.


+ Gọi A2, T2, G2, X2 lần lượt là số nu từng loại của mạch 2.


+ Gọi N là tổng số Nu của Gen.
Theo NTBS:


A1 + T1 + G1 + X1 = A2 + T2 + G2 + X2 = N/2.


A1 = T2, T1 = A2, G1 = X2, X1 = G2.


<i>- Xét trên cả 2 mạch đơn Gen.</i>



A= T =A1 + A2 = A1 + T1 = A2 + T2


G = X = G1 + G2 = G1 + X1 = G2 + X2


<b>A + G = N/2; 2A + 2G = N</b>
<i><b>* Tính tỉ lệ % từng loại Nu của Gen.</b></i>


<b>Do A+ G = N/2 → %A + %G = 50%N.</b>
Nếu xét trên cả 2 mạch:


</div>
<span class='text_page_counter'>(42)</span><div class='page_container' data-page=42>

%G = %X = (%G1 + %G2) /2 = (%X1 + %X2) /2 = …


<i><b>Ví dụ 1: Trên 1 mạch của gen có 12% A, 35%G.</b></i>
Trên mạch thứ 2 có 25%A, 450G.


1. Tính tỉ lệ %, số lượng từng loại Nu trên mỗi mạch đơn.
2. Tính tỉ lệ %, số lượng từng loại Nu của cả Gen.


<b>Bài làm</b>


<i><b>1. % và số lượng từng loại trên mỗi mạch: </b></i>
Theo nguyên tắc bổ sung ta có:


A1 = T2 = 15% ; T1 = A2 = 25% ; G1 = X2 = 35% ; X1 = G2 = 450 nu.


→ X1 = G2 = 450 nu = 100% - (15% + 25% + 35%) = 25%.


Vậy số lượng nu trên một mạch của Gen = (450/25)× 100 = 1800 nu.
Vậy % và số lượng từng loại trên mỗi mạch là:



Mạch 1 Mạch 2 % Sô lượng


A1 = T2 = 10% = 15% x 1800 = 270 nu.


T1 = A2 = 25% = 25% x 1800 = 450 nu.


G1 = X2 = 35% = 35% x 1800 = 630 nu.


X1 = G2 = 30% = 25% x 1800 = 450 nu.


<i><b>2. % và số lượng từng loại Nu của Gen.</b></i>


%A = %T = (%A1 + %A2) / 2 = (15% + 25%)/2 = 20%.


%G = %X = (%G1 + %G2) / 2 = (35% + 25%)/2 = 30%


Suy ra A = T = 270 + 450 = 720 nu.
G = X = 630 + 450 = 1080 nu.


<i><b>DẠNG 2: Tính chiều dài, số vòng xoắn, khối lượng gen/ ADN</b></i>
<i><b>1. Tính chiều dài gen</b></i>


- Chiều dài gen bằng chiều dài 1 mạch.


- Mỗi mạch có N/2 nucleoti, mỗi nucleotit có kích thước 3.4 A0


Gọi L : là chiều dài Gen
Ta có: L = 3,4 x N/2 (A0<sub>)</sub>



 N = 2L/ 3,4


<i><b>2. Số vòng xoắn và khối lượng của gen/ADN</b></i>
<b>a. sớ vịng xoắn :</b>


Mỗi vòng xoắn gồm 10 cặp Nu = 20 Nu và chiều dài trục thẳng đứng là 34 A0<sub>.</sub>


</div>
<span class='text_page_counter'>(43)</span><div class='page_container' data-page=43>

 N = 20.C  L = C x 34 (A0<sub>)</sub>


<b>b. Khới lượng gen</b>


Một nu có khối lượng 300 đvC. Gọi m là khối lượng gen ta có
m = N x 300 (đvC)  N = M /300 (nu)


<i><b>Bài tập áp dụng</b></i>


1 Gen có M = 900.000 đvC, có hiệu giữa số nucleotit loại G và loại A bằng 10% số
nu của gen


a. Tính chiều dài gen?


b. Tính tỉ lệ % và số lượng nu từng loại của gen trên?
<i><b>Bài làm</b></i>


<i><b>a. Chiều dài gen:</b></i>


N = M/300 = 900.000 /300 = 3000 nu
L = N/2 x 3.4 = 3000/2 x 3.4 = 5100 A0


<i><b>b. Số lượng và tỉ lệ % từng loại nu của gen </b></i>


Theo đề bài ta có: G – A = 10% N


G + A = 50% N


2G = 60% N  G = X = 30% N


<sub>  A = T = 20% N</sub>


Vậy: A = T = 20% x 3000 = 600 nu.
G = X = 30% x 3000 = 900 nu.


<i><b>DẠNG 3: Tính số liên kết hố học trong gen/ ADN</b></i>
<i><b>1. Tính số liên kết hoá trị giữa đường và axit trong gen</b></i>


- Trong cấu trúc của 1 nuclêơtit thì có 1 liên kết hoá trị giữa đường và axit. Trong 1
mạch đơn của gen có N/2 nuclêơtit nên số liên kết hố trị giữa đường và axit loại
này là N/2.


- Giữa các nuclêôtit nằm trên mỗi mạch đơn cũng có liên kết hố trị giữa axit của
nuclêôtit này với đường của nuclêôtit kế tiếp  trên 1 mạch của gen có N/2 nuclêơtit
nối với nhau bằng N/2 – 1 liên kết hoá trị.


<i><b>Do đó tổng số liên kết hố trị trong gen là: 2[N/2 + (N/2 – 1)] = 2(N – 1)</b></i>
<i><b>2. Tính số liên kết Hiđro của gen.</b></i>


Trong gen các nu liên kết theo NTBS A liên kết với T = 2 liên kết H
G liên kết với X = 3 liên kết H.
Gọi H là số liên kết Hidro  H = 2A + 3G.


</div>
<span class='text_page_counter'>(44)</span><div class='page_container' data-page=44>

Một gen có hiệu số giữa A và 1 loại nuclêơtit khác là 20% và có 2760 liên kết H.


Tính số nuclêơtit từng loại của Gen.


<i><b>Bài làm</b></i>
Theo đầu bài ta có :


A – G = 20% N
A + G = 20% N


2A = 70% N => A = T = 35% N
G = X = 15% N
Gen có 2760 liên kết H vì vậy ta có:


H = 2A + 3G = 2760.


Hay: 2 x 35 % N + 3 x 15% N = 2760
 N = 2400 nu.


Vậy A = T = 35% x 2400 = 840 nu.
G = X = 15% x 2400 = 360 nu.


<b>b. Bài tập về cơ chế tự nhân đơi của ADN</b>


<i>Dạng 1: Tính số lượng Nu mơi trường cung cấp cho gen/ ADN nhân đôi</i>


<i>1. Khi gen nhân đơi 1 lần </i>


Một Gen có N nuclêơtit nhân đôi 1 lần tạo 2 Gen con chứa số nuclêơtit là
2N trong đó có N nuclêơtit là của Gen mẹ (theo ngun tắc bán bảo tồn).


<i>Do đó số nu mơi trường cung cấp: 2N - N = N.(Bằng chính số nuclêơtit trong gen</i>



<i>đó)</i>


<i>2. Khi Gen nhân đơi nhiều lần</i>


- 1 Gen nhân đôi 1 lần tạo 2 = 21 <sub>Gen con.</sub>


- 1 Gen nhân đôi 2 lân tạo 4 = 22 <sub>Gen con.</sub>


- 1 Gen nhân đôi 3 lần tạo 8 = 23 <sub>Gen con.</sub>


……….
Vậy 1 Gen nhân đôi x lần tạo ra 2x <sub>gen con.</sub>


Nếu Gen chứa N nu nhân đơi x lần thì số nu mơi trường cung cấp là:
2x <i><b><sub>N – N = (2</sub></b><b>x</b><b><sub> - 1).N </sub></b></i>


Số nuclêôtit từng loại môi trường cung cấp là:
<i><b>Amt = Tmt = (2</b><b>x</b><b><sub> - 1) A gen</sub></b></i>
<i><b>Gmt = Xmt = (2</b><b>x</b><b><sub> - 1) G gen</sub></b></i>


</div>
<span class='text_page_counter'>(45)</span><div class='page_container' data-page=45>

Số nuclêôtit từng loại môi trường cung cấp cho a gen nhân giống nhau đôi x lần là:
<i><b>Amt = Tmt = a.(2</b><b>x</b><b><sub> - 1) A gen</sub></b></i>


<i><b>Gmt = Xmt = a. (2</b><b>x</b><b><sub> - 1) G gen</sub></b></i>
<i><b>Bài tập áp dụng</b></i>


Một gen có chiều dài 3060 A0<sub> hãy xác định số nuclêơtit của gen, số nuclêôtit môi</sub>


trường cần cung cấp khi gen này nhân đơi 3 lần?


<i><b>Bài làm</b></i>


Ta có N = 2L/ 3.4 = 2 x 3060/ 3.4 = 1800 nu.


Khi gen nhân đơi 3 lần thì số nu mơi trường cung cấp là:
Tổng nu MTCC = ( 23 <sub> - 1).1800 = 12.600 nu.</sub>


<i>Dạng 2: Tính số liên kết Hidrơ và số liên kết hố trị bị phá vỡ và được hình thành</i>


<i>trong q trình nhân đơi của gen</i>


<i><b>1. Tính số liên kết hidrơ bị phá vỡ và được hình thành.</b></i>


- Khi gen nhân đơi 1 lần tạo ra 2 (21<sub>)</sub> <sub>gen con thì số lần tách mạch là 1 = 2</sub>1 <sub>–</sub>


1 và có (21 <sub>– 1).H liên kết Hiđrô bị phá vỡ.</sub>


- Khi gen nhân đôi 2 lần tạo ra 4 (22<sub>)</sub> <sub>gen con thì số lần tách mạch là 3 = 2</sub>2 <sub>–</sub>


1 và có (22 <sub>– 1).H liên kết Hiđrơ bị phá vỡ.</sub>


- Khi gen nhân đôi 3 lần tạo ra 8 (23<sub>)</sub> <sub>gen con thì số lần tách mạch là 7 = 2</sub>3 <sub>–</sub>


1 và có (23 <sub>– 1).H liên kết Hiđrô bị phá vỡ.</sub>


<i>Nếu gen chứa H liên kết Hiđrơ và nhân đơi x lần thì tổng số liên kết Hiđrô bị</i>
<i>phá vỡ là:</i>


<i> (2x<sub> – 1).H</sub></i>



<i>Vì có 2x <sub>gen con được tạo ra nên tổng số liên kết Hiđrơ hình thành là</sub></i>


<i> 2x<sub>.H</sub></i>


<i><b>2. Tính số liên kết hố trị được hình thành.</b></i>


- Số liên kết hố trị được hình thành sau q trình nhân đơi là tổng số kiên kết
hố trị nối các nuclêơtit lấy từ mơi trường tạo thành chuỗi polynuclêôtit mới.


- Khi gen nhân đôi x lần, tạo 2x <sub>gen mới có tổng số 2.2</sub>x <sub> mạch polynuclêơtit.</sub>


Trong số đó có 2 mạch cũ lấy từ gen mẹ.


 Số liên kết hoá trị giữa các nuclêơtit được hình thành sau q trình nhân đơi x lần
của gen là : (2.2x<i><b><sub> – 2).(N/2 – 1) = (2</sub></b><b>x</b><b><sub> – 1).(N – 2)</sub></b></i>


<i><b>Bài tập áp dụng: </b></i>


</div>
<span class='text_page_counter'>(46)</span><div class='page_container' data-page=46>

a. Tính số liên kết hiđrơ bị phá vỡ và liên kết hiđrơ được hình thành trong q trình
nhân đơi của gen.


<i><b>b. Tính số liên kết hố trị được hình thành. </b></i>
<i><b>Bài làm</b></i>


<i><b>a. Số liên kết hidrơ bị phá vỡ và được hình thành.</b></i>
Gọi N là số nu của gen, ta có.


(23<sub> – 1).N = 16800 (nu)</sub>


N = 16800 : (23<sub> – 1) = 2400 nu.</sub>



Gen có A : G = 3 : 7, A + G = N/2 = 2400/2 = 1200 nu  G = X = 840; A = T = 360
 Số liên kết Hiđrô của gen = 2A+ 3G = 2.360 + 3.840 = 3240 H.


Gen nhân đôi 3 lần nên:


 Số liên kết Hiđrô bị phá vỡ là : (23<sub> – 1) . 3240 = 22680 H</sub>


Số liên kết Hiđrô được hình thành là : 23 <sub>. 3240 = 25920 H</sub>


<i><b>b. Số liên kết hố trị được hình thành. </b></i>


<i>(2x<sub> – 1).(N – 2) = (2</sub></i>3<sub> – 1) . (3240 – 2) = 22,666 liên kết</sub>


<i><b>Dạng 3: Tính thời gian nhân đơi của gen</b></i>
<i><b>1. Tốc độ nhân đôi của gen</b></i>


<i>Tốc độ tự sao của gen được tính bằng số nuclêơtit của mơi trường liên kết</i>


<i>vào mạch khuôn của gen trong 1 giây.</i>


<i><b>2. Thời gian nhân đôi của gen</b></i>


- Thời gian tự nhân đôi của gen bằng số nuclêôtit trên 1 mạch gen chia cho số
nuclêôtit liên kết được trên 1 mạch trong 1 giây.


- Hoặc bằng tổng số nuclêôtit của gen chia cho số nuclêôtit liên kết được trên 2
mạch khuôn của gen trong 1 giây.


<b>2.3.2.2. Các bài tập liên quan đến ARN và cơ chế phiên ma</b>


<i><b>Dạng 1: Tính số lượng nu của ARN</b></i>


Ta gọi Ar , Ur, Gr, Xr là các nu của ARN.


Phân tử ARN được tổng hợp từ mạch 3’ – 5’ của gen (mạch khuôn mẫu) theo
NTBS vì vậy số nuclêơtit của phân tử ARN (ký hiệu Nr) = số nu trên 1 mạch của


gen hay = N/2 nuclêơtit.
Theo NTBS ta có :


Ar = T gốc Ur = A gốc Gr = X gốc Xr = G gốc


</div>
<span class='text_page_counter'>(47)</span><div class='page_container' data-page=47>

Mà A gốc = Ur và T gốc = Ar


 A gen = T gen = Ur + Ar


Tương tự ta có G gen = X gen = Xr + Gr


 %A gen = %T gen = ( %Ar + %Ur)/2


% G gen = % X gen = (%Gr + %Xr)/2


<i><b>Dạng 2: Tính chiều dài, khối lượng của ARN</b></i>
<i><b>1. Tính chiều dài của ARN</b></i>


ARN được tổng hợp trên 1 mạch khuôn mẫu 3’ – 5’ của gen nên chiều dài
ARN bằng chiều dài của gen tổng hợp ra nó


LARN = Lgen = N/2 x 3,4 (A0) = Nr x 3,4 (A0)



<i><b>2. Tính khối lượng của ARN</b></i>


Số nu của ARN bằng ½ số nu của gen nên ta có
mARN= Nr x 300 đvC = N/2 x 300 đvC = mgen /2


 Nr = mARN/ 300 đvC


<i><b>Dạng 3: Tính số nu mtcc cho quá trình phiên mã ARN</b></i>


- Khi gen phiên mã 1 lần tạo ra 1ARN, lấy của môi trường Nr = N/2


- Khi gen phiên mã 2 lần tạo 2 ARN, lấy của môi trường 2.Nr = N


- Khi gen phiên mã k lần tạo ra k ARN, lấy của môi trường k.Nr = k. N/2


Vậy tổng số nu môi trường cung cấp cho quá trình phiên mã ARN = k.Nr = k.N/2


 Số lần tổng hợp ARN = tổng số nu mơi trường cung cấp cho q trình tổng hợp
ARN chia cho số nu một mạch của gen (hoặc số nu của ARN).


<i><b>2.3.2.4. Bài tập về Protein và cơ chế dịch mã tổng hợp Protein</b></i>
<i><b>Dạng 1: Tính số phần tư Protein</b></i>


- 1 riboxom trượt 1 lần trên 1 ARNm tổng hợp được 1 chuỗi polypeptit.


- 2 riboxom trượt 1 lần trên 1 ARNm tổng hợp được 2 chuỗi polypeptit.


 n riboxom trượt 1 lần trên 1 ARNm tổng hợp được n chuỗi polypeptit.


 n riboxom trượt 1 lần trên k ARNm tổng hợp được n.k chuỗi polipeptit.



Kết luận số polipeptit (protein bậc 1) được tổng hợp bằng số lượt trượt của
Riboxom nhân với số phân tử ARNm.


<b>Dạng 2</b><i><b> : </b><b> Tính số axit amin mơi trường cung cấp cho quá trình tổng hợp Protein</b></i>
Do bộ 3 kết thúc khơng mã hóa aa nên số aa mơi trường cung cấp được tính
theo số bộ 3 mã hóa aa


<i><b>- Nếu gen mã hóa tổng hợp 1 phân tư Pr</b></i>


</div>
<span class='text_page_counter'>(48)</span><div class='page_container' data-page=48>

<i><b>- Nếu có x phân tư Pr được tổng hợp từ cùng 1 gen </b></i>


Tổng số aa môi trường cung cấp = (N/2 .3 – 1 ) .x = (Nr/ 3 – 1) .x


<i><b>- Số a.a trong các phân tư Pr</b></i>


Trong quá trình tổng hợp Pr, bộ 3 kết thúc khơng mã hóa a.a, bộ 3 mở đầu mã
hóa aa Met nhưng aa này sau đó tách ra khơng tham gia vào cấu trúc phân tử Pr
hoàn chỉnh nên :


 Số aa trong 1 Pr hoàn chỉnh = N/2.3 – 2 = Nr/3 – 2.


 Nếu có x phân tử Pr được tổng hợp từ cùng 1 gen thì:


Tổng số A.a trong phân tử Pr hoàn chỉnh = (N/2.3 – 2 ). x = (Nr/3 – 2 ). x


<i><b>Dạng 3: Tính số phân tư nước dược giải phóng trong quá trình tổng hợp protein</b></i>
Khi tổng hợp protein, cứ tạo được 1 liên kết peptit thì giải phóng 1 phân tử
nước. Vậy số phân tử nước giải phóng đúng bằng số lượng axit amin trong protein
hoàn chỉnh.



= N/6 – 2 = Nr/3 - 2


<i><b>Bài tập áp dụng</b></i>


Một Gen có chiều dài 5100 A0<sub>. Gen đó nhân đơi 2 lần, mỗi gen con tạo ra</sub>


tổng phiên mã 3 lần, trên mỗi ARN có 1 Riboxom trượt qua 1 lần để tổng hợp
protein.


a. Tính số phân tử Protein do gen đó tổng hợp.


b. Tính số aa mơi trường cung cấp cho q trình tổng hợp Protein.
c. Tính số aa có trong các phân tử Protein hồn chỉnh.


d. Tính số phân tử nước được giải phóng khi tổng hợp 1 phân tử protein từ
gen trên?


<i><b>Bài làm</b></i>
<i><b>a. Số phân tư Protein</b></i>


- Gen nhân đôi 2 đợt tạo ra 22 <sub> = 4 Gen con.</sub>


- Mỗi gen con tạo ra tổng hợp ARN 3 lần => số ARN = 4 x 3 = 12 mARN
- 12 mARN có 1 Riboxom trượt qua 1 lần đã tổng hợp được 12 phân tử Protein.
<i><b>b. Số aa môi trường cung cấp</b></i>


- Số nu của gen:


N = L/3.4 = 5100 / 3.4 = 3000 Nu


- Do bộ 3 kết thúc khơng mã hóa A.a nên


Tổng số aa môi trường cung cấp = (N/2.3 – 1).12 = (3000/6 – 1) . 12 = 598
a.a


</div>
<span class='text_page_counter'>(49)</span><div class='page_container' data-page=49>

- Do aa mở đầu không tham gia thành phần Pr hoàn chỉnh nên số aa trong 12 phân
tử Pr = (3000/6 – 2).12 = 5976 a.a


<b>d. Sớ phân tử nước được giải phóng khi tởng hợp 1 phân tử Protein </b>
(3000 : 6 – 2) = 498 phân tử


<b>2.3.2.5. Bài tập về đột biến gen</b>


Từ những tính tốn liên quan đến các dạng bài tập ở trên có thể tìm ra đặc
điểm của gen sau đột biến, so sánh với gen trước đột biến để tìm ra dạng đột biến và
suy luận hậu quả của nó.


<b>2.3.3. Bài tập tự giải</b>


Bài 1: Một phân tử ADN của vi khuẩn có 3.106 <sub>căp nucleotit , tỉ lệ A/G = 1,5. Tính</sub>


số nucleotit từng loại và số liên kết hiđrơ của ADN nói trên?


Bài 2 Trên mạch I của một phân tử ADN có tỉ lệ (A1 + G1) : (T1 + X1) = 0,5.


a) Phân tử ADN này có Tỉ lệ (A + G) / (T + X) như thế nào? Giải thích.


b) Tính tỉ lệ (A2 + G2) : ( T2 + X2) trên mạch còn lại (mạch II ) của phân tử ADN


nói trên.



<i><b>Bài 3 Trên một mạch đơn của ADN nhân ở một lồi vi kh̉n có A = 15%, X = </b></i>
20% , T = 25% , G = 40% tổng số đơn phân của mạch. Tính tỉ lệ % từng loại
nucleotit trong cả phân tử ADN nói trên?.


<b>Bài 4 Trong các bộ ba sau đây, ba kết thúc là những bộ ba nào?</b>
(1): 3' AGU 5' (4): 3’UAG 5’.
(2): 3' UAA 5' (5): 5' UGA 3'


(3): 3' GAU 5'. (6): 5' AAU 3'


<b>A. (1) , (3) và (5).</b> <b> B. (2) , (4) và (5). </b>
<b>C. (2) , (4) và (6).</b> <b> D. (1) , (2) và (3).</b>


Bài 5: Một gen có khối lượng phân tử 72.104<sub> đvC. Hiệu số số lượng nucleotit loại</sub>


G với nucleotit khác trong gen bằng 380. Trên mạch gốc của gen có T = 120, trên
mạch bổ sung có X = 320 nucleotit.


a. Số lượng nucleotit mỗi loại trên gen và trên từng mạch đơn gen?


b. Số lượng ribonucleotit mỗi loại và chiều dài mARN được tổng hợp từ gen đó?
c. Số lượng axit ain cần cung cấp để tổng hợp nên 1 protein?


d. Có bao nhiều lượt tARN được điều đến để tổng hợp nên 5 phân tử protein?
Bài 6: Một gen có khối lượng phân tử 9.105<sub> đvC, trong đó A = 300 nucleotit.</sub>


a. Tìm chiều dài của gen?


</div>
<span class='text_page_counter'>(50)</span><div class='page_container' data-page=50>

d. số lượng liên kết hóa trị giữa các nucleotit của gen?



e. Nếu gen đó được tạo nên từ 3 loại nucleotit A, T, G thì có bao nhiêu kiểu mã
bộ ba có trong phân tử gen đó? Viết các kiểu bộ ba?


Bài 7: Khi tổng hợp một protein đã giải phóng ra 298 phân tử nước do việc hình
thành các liên kết peptit.


a. Chiều dài của phân tử mARN tổng hợp nên protein?
b. Khối lượng phân tử của gen?


c. Số lượng nucleotit mỗi loại trên gen? Biết rằng tổng số liên kết hidro của gen
là 2300.


Bài 8: Một phân tử mARN có 2399 liên kết hóa trị giữa các ribonucleotit và trong
mỗi ribonucleotit.


a. Tìm chiều dài của phân tử mARN?
b. Khối lượng phân tử của mARN?


c. Khi tổng hợp 1 protein có bao nhiêu lượt tARN tới đối mã?


d. Nếu cho rằng các bộ ba đối mã trên tARN được xâ dựng từ 2 loại ribonucleotit
G, X. Xác định các kiểu bộ ba mã sao trên mARN biết rằng số bộ ba kết thúc trên
mARN là UAG.


Bài 9: Một phân tử protein hồn chỉnh có 200 axit amin.


a. Có bao nhiêu phân tử nước được giải phóng do việc hình thành các liên kết
peptit để tạo ra phân tử protein trên?



b. Khối lượng phân tử protein?


c. số liên kết hóa trị trên phân tử mARN?


d. Số lượng liên kết hóa trị, liên kết hidro trên gen cấu trúc, biết rằng A = 200
nucleotit.


Bài 10: Một gen cấu trúc có tổng số liên kết hidro giữa các cặp bazo nitric bổ sung
là 3600. Tổng số liên kết hóa trị giữa các nucleotit của gne bằng 2998.


a. Tìm số lượng nucleotit mỗi loại của gen?
b. Chiều dài của gen?


c. số lượng liên kết hóa trị trên mARN được tổng hợp từ gen đó?


d. Số lượng axit amin có trong phân tử protein hồn chỉnh? Để tạo ra phân tử
protein từ gen trên cần phải hình thành bao nhiêu liên kết peptit?


</div>
<span class='text_page_counter'>(51)</span><div class='page_container' data-page=51>

a. Xác định số lượng từng loại ribonucleotit của phân tử mARN được tổng hợp
từ gen trên?


b. Số lượng phân tử mARN được tổng hợp?


Biết rằng khi tổng hợp mARN môi trường đã cung cấp 1500 ribonucleotit loại A.
Bài 12: Trình tự các axit amin trong phân tử protein hoàn chỉnh như sau:


Valin – Histidin – Alanin – Histidin – Arginin – Lizin – Serin


a. Xác định cấu trúc mARN? Biết rằng bộ ba mã hóa các axit amin trên mARN
như sau:



Valin: GUU Alanin: GXX


Histidin: XAX Lizin: AAA Arginin: XGX Serin: AGU
b.Tìm các bộ ba đối mã trên tARN


c. Trình tự phân bố các nucleotit trên gen?


Biết rằng bộ ba mở đầu trên mARN là AUG, bộ ba kết thúc là UAA.


Bài 13: Một gen có chiều dài 2040A0<sub> sao mã 1 lần, mARN có 2 riboxom trượt qua</sub>


không lặp lại. Xác định số lượng axit amin cần cung cấp cho quá trình tổng hợp
protein?


Bài 14: Một gen có khối lượng phân tử 45.104<sub> đvC. Có hiệu số A với loại nucleotit</sub>


khơng bổ sung bằng 30% số nucleotit của gen. mARN được tổng hợp từ gen đó có
U = 60% số ribonucleotit. Trên một mạch đơn của gen có G = 14% số nucleotit
của mạch và A = 450 nucleotit.


a. Số lượng nucleotit mỗi loại của gen và của từng mạch đơn trên gen?
b. Số lượng từng loại ribonucleotit của mARN?


c. Số lượng axit amin cần cung cấp cho quá trình tổng hợp protein? Nếu cho rằng
gen phiên mã 5 lần, trung bình mỗi mã sao có 8 riboxom trượt qua khơng lặp lại?
d. Tính khoảng cách đều giữa các riboxom (theo A0<sub>), nếu biết thời gian tổng hợp</sub>


xong 1 phân tử protein là 125 giây. Thời gian tiếp xúc của mARN với 8 riboxom
hết 153 giây. Các riboxom cách đều nhau khi trượt trên mARN.



e. Nếu gen tự nhân đôi 4 đợt, nhu cầu về mỗi loại nucleotit cần cung cấp là bao
nhiêu? Nếu cho rằng vận tốc nhân đôi của gen là 50 cặp nu/1 giây. Qua các đợt
nhân đơi các gen cịn nhân đơi liên tiếp với vận tốc giống nhau. Khơng tính thời
gian chuyển tiếp của các đợt nhân đôi. Xác định thời gian cần thiết để gen nhân đôi
với các đợt nêu trên?


Bài 15: Một gen quy định cấu trúc của một polypeptit gồm 298 axit amin có tỉ lệ
A:G = 4:5.


</div>
<span class='text_page_counter'>(52)</span><div class='page_container' data-page=52>

b. Tính số lượng nucleotit từng loại do mơi trường nội bào cung cấp khi gen tự sao
liên tiếp 5 lần?


c. Đột biến xảy ra không làm thay đổi số nucleotit của gen nhưng làm cho gen sau
đột biến có tỉ lệ A: G = 79,28%.


- Đột biến nói trên đã làm cho cấu trúc của gen bị thay đổi như thế nào và thuộc
kiểu nào của đột biến gen?


- Số liên kết H trong gen thay đổi như thế nào?
<b>III. KẾT LUẬN</b>


Trên đây là những mức độ câu hỏi và các dạng bài tập thường gặp liên quan
đến nội dung cơ sở vật chất và cơ chế di truyền ở mức độ phân tử. Tác giả hi vọng
rằng chuyên đề này sẽ giúp ích phần nào cho cơng tác phát hiện và bồi dưỡng HSG
cấp THCS trên địa bàn tỉnh Vĩnh Phúc. Rất mong nhận được sự góp ý chân thành
từ các thầy cơ để chun đề ngày càng hồn thiện hơn.


</div>
<span class='text_page_counter'>(53)</span><div class='page_container' data-page=53>

<b>TÀI LIỆU THAM KHẢO</b>



1. 100 câu hỏi chọn lọc về di truyền và biến dị. Tác giả Lê Đình Trung.


2. Chuyên đề Câu hỏi, bài tập trong dạy học sinh học. Tác giả Lê Đình Trung.
3. Chuyên đề Hoạt động hóa người học. Tác giả Phan Thị Thanh Hội.


4. Chuyên đề kiểm tra, đánh giá trong dạy học. Tác giả Trịnh Nguyên Giao.


5. Phương pháp giải bài tập sinh học – Tác giả : Nguyễn Văn Sang – Nguyễn Thị
Vân.


6. Sách giáo khoa, sách giáo viên Sinh học 9. NXB Giáo dục.
7. Sách giáo khoa, sách giáo viên Sinh học 12. NXB Giáo dục.
8. Sách bài tập sinh học 9. NXB Giáo dục.


9. Sách bài tập Sinh học 12. NXB Giáo dục.


</div>
<span class='text_page_counter'>(54)</span><div class='page_container' data-page=54>

<b>VAI TRỊ CỦA LÃNH TỤ NGUYỄN ÁI QUỐC - HỒ CHÍ MINH ĐỐI VỚI </b>
<b>THẮNG LỢI CỦA CÁCH MẠNG THÁNG TÁM NĂM 1945</b>


<b>Nguyễn Thị Minh Hải – THPT Chuyên Vĩnh Phúc</b>
<b>I. ĐẶT VẤN ĐỀ</b>


Mỗi bước đi và thắng lợi của cách mạng Việt Nam đều gắn liền với tên tuổi và
sự nghiệp của Lãnh tụ Nguyễn Ái Quốc - Hồ Chí Minh. Sau nhiều năm bơn ba tìm
đường cứu nước, Người đã tìm ra con đường cứu nước cho dân tộc. Đầu năm
1941, Người về nước, cùng với Trung ương Đảng trực tiếp chỉ đạo quá trình chuẩn
bị, tiến lên tổng khởi nghĩa giành chính quyền tháng Tám 1945, lập nên nước Việt
Nam Dân chủ Cộng hòa.


Trong chương trình dạy học sinh giỏi lịch sử THCS, phần vai trò của Lãnh tụ


Nguyễn Ái Quốc - Hồ Chí Minh đối với Cách mạng tháng Tám là phần nâng cao.
Giáo viên cần xác định, phân tích rõ cho học sinh hiểu những đóng góp to lớn của
Người đối với Cách mạng Việt Nam. Qua đó kích thích tư duy sáng tạo của học
sinh, tạo điều kiện để các em có thể viết bài đạt kết quả cao nhất.


<b>II. NỘI DUNG</b>


<b>1. Tìm ra con đường cứu nước, sáng lập Đảng Cộng Sản Việt Nam</b>


Nguyễn Ái Quốc sinh ra và lớn lên trong một gia đình nhà nho yêu nước, trên quê
hương giàu truyền thống cách mạng. Lớn lên trong hoàn cảnh nước mất nhà tan, lại
được chứng kiến các cuộc đấu tranh của các bậc tiền bối bị thất bại. Tất cả đã hun
đúc trong lòng Nguyễn Ái Quốc lòng yêu nước nồng nàn và quyết tâm ra đi tìm cứu
nước, cứu dân. Qua nhiều năm bơn ba ở hải ngoại để tìm đường cứu nước, cứu dân.
Đầu tiên, Người đến nước Pháp rồi đi các nước Á, Phi, Mĩ Latinh. Đến năm 1917,
khi Cách mạng tháng Mười Nga thắng lợi, Người từ Luân Đôn (Anh) về Pari (Pháp)
để nghiên cứu, học tập Cách mạng tháng Mười Nga.


</div>
<span class='text_page_counter'>(55)</span><div class='page_container' data-page=55>

Năm 1920, Người đọc Sơ thảo luận cương về vấn đề dân tộc và thuộc địa của
Lênin. Từ đó, Người hồn tồn tin theo Lênin, dứt khốt đứng về Quốc tế thứ ba,
khẳng định muốn cứu nước và giải phóng dân tộc là phải đi theo con đường cách
mạng vô sản. Tại Đại hội của Đảng Xã hội Pháp họp ở Tua, Nguyễn Ái Quốc đã bỏ
phiếu tán thành Quốc tế thứ ba và lập ra Đảng Cộng sản Pháp. Sau đó Người đã
tham gia Đảng Cộng sản Pháp - và là người cộng sản Việt Nam đầu tiên đánh dấu
bước ngoặt trong hoạt động Nguyễn Ái Quốc. Qua thực tiễn cách mạng năm 1924,
Người chỉ ra rằng: "Cuộc đấu tranh giai cấp không diễn ra giống như ở phương
Tây… Chính vì thế, Người khơng rập khn máy móc học thuyết về đấu tranh giai
cấp mà đã áp dụng một cách linh hoạt vào điều kiện lịch sử cụ thể ở thuộc địa.
Năm 1925 tại Quảng Châu Trung Quốc, Người sáng lập Hội Việt Nam Cách mạng
Thanh niên, là tổ chức tiền thân của Đảng cộng sản Việt Nam. Dưới tác động của


Hội phong trào cách mạng Việt Nam ngày càng phát triển, kết thành một làn sóng
dân tộc dân chủ mạnh mẽ.


Cuối năm 1929 sự ra đời liên tiếp của các tổ chức cộng sản trong nước: Đông
Dương Cộng sản đảng (6/1929), An Nam Cộng sản Đảng (8/1929) và Đơng
Dương Cộng sản liên đồn (9/1929), là xu thế khách quan của cuộc vận động giải
phóng dân tộc ở Việt Nam theo khuynh hướng cách mạng vô sản. Tuy nhiên sự ra
đời và cùng hoạt động không tránh khỏi sự riêng rẽ, tranh giành ảnh hưởng của
nhau, cách mạng trong nước đứng trước nguy cơ chia rẽ lớn. Được sự ủy nhiệm
của Quốc tế Cộng sản cùng với uy tín của Nguyễn Ái Quốc, Người đã triệu tập Hội
nghị hợp nhất các tổ chức cộng sản thành lập Đảng Cộng sản Việt Nam. Tại Hội
nghị Người đã thơng qua Chính cương vắn tắt, Điều lệ vắn tắt… coi như Cương
lĩnh chính trị đầu tiên của Đảng, có giá trị thực tiễn và lâu dài đối với cách mạng
Việt Nam.


Cương lĩnh chính trị nhấn mạnh nhiệm vụ hàng đầu của cách mạng Việt Nam là
chống đế quốc, giành độc lập dân tộc: đánh đổ đế quốc chủ nghĩa Pháp và bọn
phong kiến, làm cho nước Nam được hoàn toàn độc lập, dựng ra Chính phủ cơng
nơng binh và tổ chức qn đội cơng nơng; tịch thu tồn bộ các sản nghiệp lớn của
bọn đế quốc giao cho Chính phủ công nông binh; tịch thu ruộng đất của bọn đế
quốc làm của công và chia cho dân cày nghèo.


</div>
<span class='text_page_counter'>(56)</span><div class='page_container' data-page=56>

không lúc nào bằng”. “Pháp – Nhật ngày nay không phải chỉ là kẻ thù của công
nông” mà là kẻ thù chung của cả dân tộc.


Hội nghị khẳng định: "Cần phải thay đổi chiến lược. Sự thay đổi về kinh tế, chính
trị Đơng Dương, lực lượng giai cấp Đông Dương, buộc Đảng ta phải thay đổi
chính sách cách mạng ở Đơng Dương cho hợp với nguyện vọng chung của toàn thể
nhân dân Đông Dương…"Nhiệm vụ đánh Pháp đuổi Nhật không phải riêng của
của giai cấp vô sản và dân cày, mà là nhiệm vụ chung của tồn thể nhân dân Đơng


Dương". Hội nghị chủ trương: “Trong lúc này, quyền lợi của bộ phận, của giai cấp
phải đặt dưới sự sinh tử, tồn vong của quốc gia, của dân tộc. Trong lúc này nếu
không giải quyết được vấn đề giải phóng dân tộc, khơng địi được độc lập tự do
cho tồn thể dân tộc, thì chẳng những tồn thể quốc gia dân tộc còn chịu mãi kiếp
ngựa trâu, mà quyền lợi của bộ phận, giai cấp đến vạn năm cũng khơng địi lại
được". Vì vậy, phải tạm gác khẩu hiệu "đánh đổ địa chủ, chia ruộng đất cho dân
cày". Quyền lợi của nông dân được giải quyết ở một mức độ thích hợp bằng việc
thực hiện các khẩu hiệu: tịch thu ruộng đất của bọn đế quốc và Việt gian chia cho
dân cày nghèo, chia lại ruộng đất công cho công bằng, giảm tô, giảm tức.


Hội nghị chủ trương gải quyết vấn đề dân tộc trong khuôn khổ từng nước ở Đông
Dương, nhằm thực hiện chính sách “dân tộc tự quyết”. “Các dân tộc sống trên cõi
Đông Dương sẽ tùy theo ý muốn, tổ chức thành liên bang cộng hòa dân chủ hay
đứng riêng thành thành một dân tộc quốc gia tùy ý”. Cách giải quyết đó có tác
dụng phát huy sức mạnh của mỗi dân tộc trong cuộc đấu tranh tự giải phóng mình,
đập tan những luận điệu xuyên tạc của kẻ thù về vấn đề dân tộc, đồng thời tạo điều
kiện đoàn kết ba dân tộc Việt Nam, Lào, Campuchia trong cuộc đấu tranh chống
kẻ thù chung.


Về phương pháp cách mạng, Hội nghị đề ra chủ trương khởi nghĩa vũ trang, nhấn
mạnh đây là nhiệm vụ trung tâm của Đảng và của nhân dân ta trong giai đoạn hiện
tại. Nghị quyết hội nghị ghi rõ: "Phải luôn luôn chuẩn bị một lực lượng sẵn sàng,
nhằm vào cơ hội thuận tiện hơn cả mà đánh lại quân thù…". Trong những hoàn
cảnh nhất định "với lực lượng sẵn có ta có thể lãnh đạo một cuộc khởi nghĩa từng
phần trong từng địa phương, cũng có thể giành sự thắng lợi mà mở đường cho một
cuộc tổng khởi nghĩa to lớn.


</div>
<span class='text_page_counter'>(57)</span><div class='page_container' data-page=57>

dân tộc và quan điểm khởi nghĩa dân tộc là sự chuẩn bị về đường lối và phương
pháp cho thắng lợi của Cách mạng tháng Tám sau này.



<b>3. Sáng lập mặt trận Việt Minh và chuẩn bị lực lượng chính trị </b>


Sự nghiệp giải phóng dân tộc bị áp bức trước hết phải dựa vào sức mạnh bên trong
<i>của mỗi dân tộc. Trong tác phẩm Đường kách mệnh, Người cho rằng: "Cách mệnh </i>
là việc chung của cả dân chúng", lực lượng của cách mạng giải phóng dân tộc bao
gồm cả dân tộc: “dân tộc cách mệnh chưa phân giai cấp, nghĩa là sĩ, nơng, cơng,
thương đều nhất trí chống lại cường quyền”, trong đó “cơng nơng là gốc cách
mệnh”, là “chủ cách mệnh”, cịn “học trị, nhà bn nhỏ, điền chủ nhỏ... là bầu bạn
cách mệnh”. Mọi giới đồng bào đều phải đoàn kết dưới ngọn cờ lãnh đạo của Đảng
cách mạng để chống lại kẻ thù của dân tộc.


Hội nghị thành lập Đảng do Nguyễn Ái Quốc chủ trì thơng qua Cương lĩnh chính
trị đầu tiên, nêu rõ: Đảng phải thu phục đại bộ phận công nhân, nơng dân, đồn kết
với tiểu tư sản, trí thức, trung nông, tranh thủ hoặc trung lập phú nông, trung tiểu
<i>địa chủ và tư sản Việt Nam...; đồng thời thông qua chủ trương thành lập Hội phản </i>


<i>đế đồng minh. Chủ trương tập hợp lực lượng cách mạng rộng rãi thể hiện tư tưởng </i>


đại đoàn kết dân tộc của Hồ Chí Minh.


Xây dựng lực lượng chính trị là mối quan tâm hàng đầu của Người vì đó vừa là cơ
sở cho việc xây dựng lực lượng vũ trang, là cơ sở cho đấu tranh quân sự, vừa là lực
lượng trực tiếp đánh địch, đấu tranh từ hình thức thấp đến cao.


Lực lượng chính trị là đạo quân cách mạng vô cùng đông đảo, bao gồm tất cả quần
chúng được giác ngộ và tổ chức. "Muốn có đội quân vũ trang phải có đội quân
tuyên truyền vận động, đội quân chính trị trước đã, nên phải làm ngay, sao cho đội
qn chính trị ngày càng đơng. Phải có quần chúng giác ngộ chính trị tự nguyện
cầm súng thì mới thắng được".



Từ đầu năm 1941, Hồ Chí Minh chỉ đạo xây dựng thí điểm mặt trận Việt Minh ở
Cao Bằng nhằm rút kinh nghiệm cho phong trào cả nước. Hội nghị lần thứ tám của
Ban chấp hành Trung ương Đảng (5-1941) do Người chủ trì xác định chiến thuật
hiện tại của Đảng ta là phải vận dụng “một phương pháp hiệu triệu hết sức thống
thiết, làm sao đánh thức được tinh thần dân tộc xưa nay trong nhân dân”. Vì thế
mặt trận của ta hiện tại “không thể gọi như trước là mặt trận thống nhất dân tộc
phản đề Đông Dương, mà phải đổi ra cái tên khác cho có tính chất dân tộc hơn”,
đó là Việt Nam độc lập đồng minh, gọi tắt là Việt Minh. Các đoàn thể trong Việt
<i>Minh đều mang tên Cứu quốc nhằm đoàn kết mọi người Việt Nam có lịng u </i>
nước thương nòi.


</div>
<span class='text_page_counter'>(58)</span><div class='page_container' data-page=58>

đều phải kề vai gánh vác một phần trách nhiệm: người có tiền góp tiền, người có
của góp của, người có tài năng góp tài năng”.


Hồ Chí Minh kêu gọi quần chúng tham gia Việt Minh nhằm:
“Quyết làm cho nước non này,
Cờ treo độc lập, nền xây bình quyền”.


Trong Chỉ thị thành lập Đội Việt Nam tuyên truyền giải phóng quân (22/12/1944)
Người khẳng định: Cuộc kháng chiến của ta là cuộc kháng chiến toàn dân, phải
động viên toàn dân, vũ trang tồn dân. Khởi nghĩa vũ trang khơng phải là một cuộc
đấu tranh quân sự thuần túy.


Việt Minh là khối đại đồn kết dân tộc Việt Nam, tập hợp đơng đảo các tầng lớp
nhân dân, các đảng phái chính trị và tơn giáo u nước, có tác dụng cơ lập cao độ
kẻ thù là đế quốc và tay sai để chĩa mũi nhọn đấu tranh vào chúng. Đó là nơi tổ
chức, giác ngộ và rèn luyện lực lượng chính trị cho cách mạng, một lực lượng cơ
bản và có ý nghĩa quyết định trong tổng khởi nghĩa giành chính quyền.


<b>4. Chuẩn bị lực lượng vũ trang và xây dựng căn cứ địa cách mạng </b>



Ngay trong thời kỳ vận động thành lập Đảng, Hồ Chí Minh đã quan tâm đào tạo
cán bộ quân sự, nghiên cứu lý luận quân sự, đặt nền móng cho sự ra đời lực lượng
vũ trang cách mạng Việt Nam.


Là một nhà hoạt động quốc tế, Hồ Chí Minh nghiên cứu nhiều vấn đề quân sự ở
các nước trên thế giới, tiếp thu có chọn lọc những nội dung tích cực, phù hợp của
tinh hoa quân sự thế giới. Nghiên cứu về quân đội ở Trung Quốc, Người ghi chép


<i>Những hiểu bết cơ bản về quân sự. Người cũng quan tâm nghiên cứu, phổ biến </i>


kinh nghiệm tổ chức, xây dựng và hoạt động của lực lượng vũ trang ở cơ sở...
Cách mạng phải có nơi đứng chân để giải quyết vấn đề tiềm lực. Đầu năm 1941,
khi mới về nước Người chọn Cao Bằng làm nơi đứng chân đầu tiên vì đó là nơi
"có phong trào tốt từ trước", "có hàng rào quần chúng bảo vệ", lại là nơi có địa thế
hiểm yếu, “tiến có thể đánh, lui có thể giữ”, nhưng từ Cao Bằng cịn phải phát triển
về xi thì cách mạng mới thắng lợi. Trong thời kỳ kháng Nhật cứu nước, khi
vùng giải phóng ở Việt Bắc được mở rộng, bao gồm các tỉnh Cao Bằng, Bắc Cạn,
Lạng Sơn, Hà Giang, Tuyên Quang, Thái Nguyên, Hồ Chí Minh chỉ thị thành lập


<i>Khu giải phóng và củng cố thành căn cứ địa cách mạng cho cả nước.</i>


</div>
<span class='text_page_counter'>(59)</span><div class='page_container' data-page=59>

Nam tuyên truyền giải phóng quân được thành lập, lúc đầu có 34 chiến sĩ, do đồng
chí Võ Nguyên Giáp chỉ huy. Ba ngày sau, Đội Việt Nam tuyên truyền giải phóng
quân đánh thắng hai trận liên tiếp ở Phai Khắt và Nà Ngần. Đó là đội quân đàn
anh. “Tuy lúc đầu quy mô của nó cịn nhỏ, nhưng tiền đồ của nó rất vẻ vang. Nó là
khởi điểm của giải phóng quân, nó có thể đi suốt từ Nam chí Bắc, khắp đất nước
Việt Nam”.


Thực hiện tư tưởng vũ trang quần chúng cách mạng, xây dựng quân đội nhân dân


<i>của Hồ Chí Minh, Việt Nam tun truyền giải phóng qn và Cứu quốc quân được </i>
<i>thống nhất thành Việt Nam giải phóng quân (5/1945), đồng thời lực lượng bán vũ </i>
trang gồm các đội du kích, tự vệ và tự vệ chiến đấu được xây dựng ngày càng rộng
khắp.


Quá trình chuẩn bị lực lượng là một quá trình kết hợp tổ chức và đấu tranh, thông
qua đấu tranh để rèn luyện lực lượng và củng cố tổ chức. Cả hai lực lượng chính trị
và vũ trang đều là chỗ dựa của bạo lực cách mạng, là điều kiện để kết hợp đấu
tranh chính trị với đấu tranh vũ trang.


<b>5. Xác định đúng thời cơ, kiên quyết phát động tổng khởi nghĩa </b>


Để tiến hành khởi nghĩa thắng lợi phải có đủ điều kiện chủ quan, khách quan và
phải nổ ra đúng thời cơ. Cùng với việc chuẩn bị lực lượng chu đáo, Hồ Chí Minh
và Trung ương Đảng hết sức coi trọng vấn đề thời cơ cách mạng. Khi nghe tin về
cuộc khởi nghĩa Nam Kỳ, Người nói: Tình hình chung trên thế giới ngày càng có
lợi cho ta, nhưng thời cơ chưa đến, chưa thể khởi nghĩa được. Song nay khởi nghĩa
đã nổ ra rồi, thì cần rút lui cho khéo để duy trì phong trào. Là bậc thầy về tạo lực,
lập thế, tranh thời, Hồ Chí Minh nhận thức sâu sắc tầm quan trọng của vấn đề thời
cơ. Trong bài thơ Học đánh cờ, Người viết: “Lạc nước hai xe đành bỏ phí


Gặp thời, một tốt cũng thành công”.


Hội nghị lần thứ tám của Ban chấp hành Trung ương Đảng (5/1941) khẳng định:
“Cách mạng Đông Dương phải kết liễu bằng một cuộc khởi nghĩa vũ trang”.
Chúng ta phải “luôn luôn chuẩn bị một lực lượng sẵn sàng, nhằm vào cơ hội thuận
tiện hơn cả mà đánh lại quân thù”. Hồ Chí Minh nêu ba điều kiện của thời cơ khởi
nghĩa:


<i>Một là, chính quyền thực dân đế quốc đã lung lay bối rối đến cao độ, chúng đã cảm</i>



thấy không thể ngồi yên nắm giữ địa vị của chúng như trước.


<i>Hai là, quần chúng đói khổ đã căm thù thực dân đế quốc đến cực điểm, đã thấy cần</i>


</div>
<span class='text_page_counter'>(60)</span><div class='page_container' data-page=60>

<i>Ba là, đã có một chính đảng cách mạng đủ sức tổ chức, lãnh đạo quần chúng nổi </i>


dậy khởi nghĩa theo một đường lối đúng đắn, một kế hoạch phù hợp, đảm bảo
giành thắng lợi cho cuộc khởi nghĩa.


Hội nghị lần thứ tám của Ban chấp hành Trung ương Đảng nêu rõ muốn gây cuộc
khởi nghĩa bằng võ trang phải nhằm vào các điều kiện chủ quan:


“1- Mặt trận cứu quốc đã thống nhất được tồn quốc.


2- Nhân dân khơng thể sống được nữa dưới ách thống trị của Pháp – Nhật, mà đã
sẵn sàng hy sinh bước vào con đường khởi nghĩa.


3- Phe thống trị Đông Dương đã bước vào cuộc khủng hoảng phổ thơng đến cực
điểm về kinh tế, chính trị lẫn quân sự.


4- Những điều kiện khách quan thuận lợi cho cuộc khởi nghĩa Đông Dương như
quân Tàu đại thắng quân Nhật, Cách mạng Pháp hay Cách mạng Nhật nổi dậy, phe
dân chủ đại thắng ở Thái Bình Dương, Liên Xô đại thắng, cách mạng các thuộc địa
Pháp, Nhật sôi nổi và nhất là quân Tàu hay quân Anh – Mỹ tràn vào Đông


Dương”.


Đảng và Hồ Chí Minh ln dự đốn thời cơ, đánh giá chính xác xu thế phát triển
của thời cuộc. Khi Chiến tranh thế giới thứ hai kết thúc, phát xít Nhật đầu hàng


Đồng minh không điều kiện, hàng ngũ chỉ huy của Nhật ở Đông Dương chia rẽ
đến cực điểm; quân lính Nhật tan rã, mất tinh thần; bọn Việt gian thân Nhật hoảng
sợ. “Những điều kiện khởi nghĩa ở Đơng Dương như đã chín muồi”. “Cơ hội rất tốt
cho ta giành quyền độc lập đã tới”.


Tuy nhiên thời cơ này khơng tồn tại vĩnh viễn. Nó sẽ biến mất khi quân Đồng
minh vào nước ta để tước vũ khí phát xít Nhật. Với bản chất thực dân đế quốc,
chúng có thể dựng ra một chính quyền tay sai trái với ý chí và nguyện vọng của
dân tộc ta. Đế quốc Pháp đang lăm le khôi phục lại địa vị cũ ở Đông Dương. Bọn
phản động trong nước cũng đang tìm cách thay thầy đổi chủ. Thời cơ cách mạng
chỉ tồn tại trong khoảng thời gian từ khi phát xít Nhật đầu hàng Đồng minh đến
trước khi quân Đồng minh vào nước ta. Đó là lúc kẻ thù cũ đã ngã gục, nhưng kẻ
thù mới chưa kịp đến, so sánh lực lượng có lợi nhất đối với cách mạng.


Ngày 16-8-1945 Hồ Chí Minh kêu gọi: "Giờ quyết định cho vận mệnh dân tộc ta
đã đến. Toàn quốc đồng bào hãy đứng dậy đem sức ta mà giải phóng cho ta…
Chúng ta không thể chậm trễ".


</div>
<span class='text_page_counter'>(61)</span><div class='page_container' data-page=61>

nhanh chóng chuyển từ khởi nghĩa từng phần trong cao trào kháng Nhật cứu nước
lên tổng khởi nghĩa giành chính quyền trong cả nước.


Nhờ có sự ch̉n bị lực lượng chu đáo, lại nổ ra đúng thời cơ, cách mạng tháng
Tám đã giành được thắng lợi “nhanh, gọn, ít đổ máu”. Đó là một điển hình thành
cơng về nghệ thuật tạo thời cơ, dự đốn thời cơ, nhận định chính xác thời cơ, đồng
thời kiên quyết chớp thời cơ phát động toàn dân nổi dậy tổng khởi nghĩa giành
chính quyền.


<b>6. Sáng lập Nhà nước Việt Nam Dân chủ Cợng hịa </b>


<i>Trong tác phẩm Đường Kách mệnh, Hồ Chí Minh chỉ rõ: “Chúng ta đã hy sinh làm</i>


cách mệnh, thì nên làm cho đến nơi nghĩa là làm sao cách mệnh rồi thì quyền giao
cho dân chúng số nhiều, chớ để trong tay bọn ít người. Thế mới khỏi hy sinh nhiều
lần, thế dân chúng mới được hạnh phúc”.


Người đã xem xét các mơ hình nhà nước ở Mĩ, Pháp, Liên Xơ, Người chủ trương
“dựng ra chính phủ cơng nơng binh”. Đó là chính quyền nhà nước của quảng đại
quần chúng lao động. Đến năm 1941, Người chủ trương thay hình thức chính
quyền cơng nơng bằng hình thức dân cộng hịa dân chủ. “Chính quyền cách mạng
của của nước dân chủ mới ấy không phải thuộc quyền riêng của một giai cấp nào
mà là của chung cả toàn thể dân tộc, chỉ trừ có bọn tay sai của đế quốc Pháp –
Nhật và những bọn phản quốc, những bọn thù, khơng được giữ chính quyền, cịn ai
là người dân sống trên dải đất Việt Nam thảy đều được một phần tham gia giữ
chính quyền, phải có một phần nhiệm vụ giữ lấy và bảo vệ chính quyền ấy”.
“Khơng nên nói cơng nơng liên hiệp và lập chính quyền Xơ viết mà phải nói tồn
thể nhân dân liên hợp và lập chính phủ dân chủ cộng hoà”. “Sau khi đánh đuổi
được đế quốc Pháp, Nhật, sẽ thành lập chính phủ nhân dân của Việt Nam Dân chủ
Cộng hồ... Chính phủ ấy do quốc dân đại hội cử ra".


Từ mơ hình nhà nước cơng nơng binh chuyển sang mơ hình nhà nước đại biểu cho
khối đoàn kết dân tộc là một quyết định sáng suốt của Hồ Chí Minh, phản ánh
được nét đặc thù của thực tiễn dân tộc, phù hợp với sự thay đổi chiến lược và sách
lược của cách mạng Việt Nam.


</div>
<span class='text_page_counter'>(62)</span><div class='page_container' data-page=62>

Thắng lợi của Cách mạng tháng Tám 1945 là thắng lợi của đường lối giải phóng dân
tộc đúng đắn của Đảng và tư tưởng độc lập tự do của Hồ Chí Minh. "Chủ tịch Hồ
Chí Minh là một biểu tượng kiệt xuất về quyết tâm của cả một dân tộc, đã cống hiến
trọn đời mình cho sự nghiệp giải phóng dân tộc của nhân dân Việt Nam, góp phần
vào cuộc đấu tranh chung của các dân tộc vì hồ bình, độc lập dân tộc, dân chủ và
tiến bộ xã hội".



<b>III. MỘT SỐ DẠNG CÂU HỎI</b>


<b>Câu 1: (2 điểm) Nguyên nhân thành công của Hội nghị thành lập Đảng Cộng</b>
<b>sản Việt Nam (1/1930)? Vai trị của Ngũn Ái Q́c trong Hợi nghị đó</b>
<b>như thế nào? </b>


1. Ngun nhân thành cơng của hội nghị thành lập Đảng


- Giữa các đại biểu các tổ chức cộng sản khơng có mâu thuẫn về ý thức hệ, đều có
xu hướng vơ sản, đều tuân theo điều lệ của Quốc tế Cộng sản. Đáp ứng đúng nhu
cầu thực tiễn của Cách mạng Việt Nam lúc đó.


- Do được sự quan tâm của Quốc tế Cộng sản và uy tín cao của lãnh tụ Nguyễn Ái
Quốc.


2. Vai trò của Nguyễn Ái Quốc trong hội nghị


- Trước yêu cầu thống nhất các tổ chức cộng sản, Nguyễn Ái Quốc đã chủ động
đứng ra triệu tập hội nghị hợp nhất các tổ chức cộng sản. Người chủ trì Hội nghị,
chính uy tín năng lực của Người đóng vai trị quan trọng đưa đến sự hợp nhất các tổ
chức cộng sản để thành lập Đảng Cộng sản Việt Nam.


- Người biên soạn Chính cương vắn tắt, sách lược vắn tắt được Hội nghị thơng
qua, văn kiện đó trở thành Cương lĩnh chính trị đầu tiên của Đảng, có giá trị thực
tiễn và lâu dài.


<b>Câu 2: (2 điểm) Trình bày khái quát những sự kiện lịch sử liên quan đến lanh</b>
<b>tụ Hồ Chí Minh trong tiến trình cách mạng Việt Nam từ năm 1941 đến</b>
<b>năm 1945. </b>



- Ngày 28/01/1941, Nguyễn Ái Quốc về nước triệu tập Hội nghị Trung ương
lần thứ VIII (10/5/1941) và thành lập Mặt trận Việt Minh (19/5/1941), trực tiếp
lãnh đạo cuộc vận động cho Cách mạng tháng Tám.


</div>
<span class='text_page_counter'>(63)</span><div class='page_container' data-page=63>

- Tháng 5/1945, Hồ Chí Minh rời Pác Bó (Cao Bằng) về Tun Quang, xây dựng
Tân Trào thành trung tâm chỉ đạo phong trào cách mạng cả nước.


- Ngày 4/6/1945, Hồ Chí Minh chỉ thị thành lập Khu giải phóng Việt Bắc.


- Ngày 16 đến 17 tháng 8/1945, Hồ Chí Minh triệu tập Đại hội quốc dân ở Tân Trào
và được cử làm Chủ tịch Uỷ ban Dân Tộc Giải Phóng Việt Nam.


- Ngày 25/8/1945, Hồ Chí Minh từ Tân Trào về Hà Nội.


- Ngày 28/8/1945, Hồ Chí Minh cải tổ Ủy ban Dân tộc giải phóng Việt Nam thành
Chính phủ lâm thời nước Việt Nam Dân Chủ Cộng Hịa. Trong những ngày này, Hồ
Chí Minh soạn thảo bản Tun ngơn độc lập.


- Ngày 2/9/1945, Hồ Chí Minh đọc Tuyên ngôn độc lập khai sinh nước Việt Nam
dân chủ cộng hòa.


<b>Câu 3: (2 điểm). Nêu vai trị của Ngũn Ái Q́c trong Hợi nghị lần thứ 8</b>
<b>Ban Chấp hành Trung ương Đảng Cộng sản Đông Dương (5 - 1941).</b>
- Sau hơn 30 năm bôn ba ở hải ngoại, ngày 28 - 1 - 1941, lãnh tụ Nguyễn Ái Quốc
trở về nước, cùng với Trung ương Đảng trực tiếp lãnh đạo cách mạng. Người vận
động quần chúng các dân tộc ở Cao Bằng tham gia cách mạng, mở nhiều khóa
huấn luyện chính trị và quân sự cho các bộ và nhân dân; dịch và viết sách về quân
sự, chính trị để làm tài liệu học tập và tuyên truyền; chuẩn bị tiến tới Hội nghị lần
thứ 8 Ban Chấp hành Trung ương Đảng Đông Dương.



- Người tổ chức và chủ trì Hội nghị lần thứ 8 Ban Chấp hành Trung ương Đảng ở
Pác Bó (Cao Bằng) từ ngày 10 đến 19 - 5 - 1941. Hội nghị đã hoàn chỉnh việc chủ
trương lãnh đạo cách mạng Việt Nam trong tình hình mới đã được đề ra ở Hội nghị
Trung ương Đảng lần 6 (11-1939) là: Giương cao hơn nữa ngọn cờ độc lập dân tộc,
đặt nhiệm vụ giải phóng dân tộc lên hàng đầu, giải quyết vấn đề dân tộc trong
phạm vi từng nước ở Đông Dương, mở rộng mặt trận dân tộc thống nhất …
- Vai trị:


+ Là người trực tiếp chủ trì Hội nghị trung ương Đảng lần VIII, Nguyễn Ái Quốc
cùng với Ban Chấp hành Trung ương Đảng có những quyết định đúng đắn, sáng
suốt.


+ Khẳng định nhiệm vụ trước mắt của cách mạng là giải phóng dân tộc. Đề
xuất việc chuẩn bị về lực lượng chính trị: thành lập Mặt trận Việt Minh, đoàn kết
toàn dân tiến hành đấu tranh chống Pháp - Nhật giành độc lập tự do.


</div>
<span class='text_page_counter'>(64)</span><div class='page_container' data-page=64>

- Những quyết định của Hội nghị có tác dụng quan trọng trong việc vận động toàn
Đảng, toàn dân chuẩn bị tiến tới Cách mạng tháng Tám 1945.


<b>Câu 4: (2 điểm) Bằng những sự kiện lịch sử có chọn lọc từ năm 1941 đến năm</b>
<b>1945, hay làm sáng tỏ vai trò của Hồ Chí Minh đới với thắng lợi của</b>
<b>Cách mạng tháng Tám 1945. </b>


<i>- Xác định đường lối và phương pháp cách mạng: Nguyễn Ái Quốc chủ trì Hội </i>
nghị lần thứ 8 Ban Chấp hành Trung ương Đảng (5 - 1941), giương cao hơn nữa
ngọn cờ giải phóng dân tộc, đồng thời đề ra chủ trương khởi nghĩa vũ trang.


- Sáng lập Mặt trận Việt Minh (19/5/1941), với các hội "cứu quốc", xây dựng khối đại
đoàn kết dân tộc; tập hợp và rèn luyện lực lượng chính trị quần chúng.



- Xây dựng lực lượng vũ trang và căn cứ địa cách mạng: Chỉ thị thành lập Đội Việt
Nam tuyên truyền giải phóng quân (22/12/1944); chọn Cao Bằng để xây dựng căn cứ
<i>địa, chỉ thị thành lập Khu giải phóng Việt Bắc (6/1945). </i>


- Cùng với Trung ương Đảng đánh giá chính xác thời cơ, chớp đúng thời cơ, kiên
quyết phát động và lãnh đạo tồn dân nổi dậy tổng khởi nghĩa giành chính quyền
<i>thắng lợi; lập nên nước Việt Nam Dân chủ Cộng hoà (2 - 9 - 1945). </i>


<b>Câu 5: Chứng minh sự lanh đạo kịp thời và sáng tạo của Đảng Cợng sản Đơng</b>
<b>Dương và lanh tụ Hồ Chí Minh trong Cách mạng tháng Tám (1945). </b>
<i>a) Xác định thời cơ của cuộc Tổng khởi nghĩa tháng Tám 1945: </i>


- Đến đầu tháng 8/1945, sau khi trải qua một thời gian trực tiếp chuẩn bị chu đáo
về tất cả các mặt từ 1939 - 1945, những điều kiện chủ quan của một cuộc tổng khởi
nghĩa giành chính quyền đã chín muồi.


- Cao trào kháng Nhật cứu nước phát triển đến đỉnh cao, chiến tranh du kích kết
hợp khởi nghĩa từng phần và đấu tranh chính trị diễn ra sôi nổi ở nông thôn và thành
thị. Tháng 8/1945 khơng khí cách mạng đã tràn ngập trong cả nước,qn chúng
nhân dân sôi sục chuẩn bị khởi nghĩa, và sẵn sàng đứng lên Tổng khởi nghĩa giành
chính quyền khi có lệnh của Đảng.


- Trưa 15/8/1945, Nhật hồng tuyên bố đầu hàng phe Đồng minh. Điều kiện khách
quan cho Tổng khởi nghĩa đã đến. Quân đội Nhật ở Đơng Dương rệu rã. Chính phủ
thân Nhật Trần Trọng Kim hoang mang, tê liệt. Lực lượng trung gian đã ngả hẳn
về phía cách mạng. Những sự kiện trên làm cho những điều kiện (thời cơ) của cuộc
Tổng khởi nghĩa chín muồi trong tồn quốc.


</div>
<span class='text_page_counter'>(65)</span><div class='page_container' data-page=65>

- Ngay từ ngày 13/8/1945, khi được những thông tin về việc Nhật sắp đầu hàng,
Trung ương Đảng và Tổng bộ Việt Minh lập tức thành lập Ủy ban khởi nghĩa toàn


quốc. 23 giờ cùng ngày, Ủy ban khởi nghĩa tồn quốc ra “Qn lệnh số 1”, chính
thức phát động Tổng khởi nghĩa trong cả nước.


- Tiếp đó, ngày 14 và 15/8/1945, Hội nghị tồn quốc của Đảng đã thơng qua kế
hoạch lãnh đạo tồn dân Tổng khởi nghĩa giành chính quyền. Từ ngày 16 đến
17/8/1945, Quốc dân Đại hội được triệu tập tại Tân Trào, tán thành chủ trương
Tổng khởi nghĩa của Đảng, thơng qua 10 chính sách của Mặt trận Việt Minh, cử ra
Ủy ban Dân tộc Giải phóng Việt Nam do Hồ Chí Minh làm Chủ tịch.


- Từ ngày 14 đến ngày 28 tháng 8/1945, Tổng khởi nghĩa thắng lợi trong toàn quốc.
Đảng ta đứng đầu là Chủ tịch Hồ Chí Minh đã kịp thời phát động tồn dân Tổng
khởi nghĩa giành chính quyền đồng thời đề ra nhiều chủ trương, biện pháp đúng
đắn và sáng tạo. Nhờ đó, cuộc Tổng khởi nghĩa tháng Tám 1945 đã giành được
thắng lợi nhanh chóng trong 15 ngày.


</div>
<span class='text_page_counter'>(66)</span><div class='page_container' data-page=66>

<b>CÁC DẠNG CÂU HỎI LÝ THUYẾT CHỦ YẾU TRONG ĐỀ THI HỌC </b>
<b>SINH GIỎI LỚP 9 MƠN ĐỊA LÍ.</b>


<b>Lê Thị Hải ́n – THPT Chun Vĩnh Phúc</b>
<b>A. MỞ ĐẦU</b>


- Tác giả chuyên đề: Lê Thị Hải Yến - GV Địa lí Trường THPT Chuyên
Vĩnh Phúc.


- Đối tượng học sinh bồi dưỡng: Học sinh khá, giỏi lớp 9.
- Giới hạn: Địa lí kinh tế xã hội Việt Nam (chương trình lớp 9)


- Hệ thống kiến thức sử dụng trong chuyên đề: Kiến thức cơ bản trong
Sách giáo khoa và kiến thức nâng cao dùng trong kỳ thi học sinh giỏi cấp
trường, cấp huyện –thị và cấp tỉnh.



- Hệ thống phương pháp cơ bản, đặc trưng để giải các dạng bài tập trong
chuyên đề: Phương pháp thuyết trình để giảng phần lý thuyết cơ bản, phương
pháp đàm thoại - vấn đáp, phương pháp hoạt động theo nhóm.


- Trong chuyên đề gồm có: lý thuyết cơ bản, hệ thống các ví dụ, bài tập
đặc trưng để giải các dạng bài tập trong chuyên đề; các bài tập HS tự giải.
- Sự cần thiết của chuyên đề: Nhiều học sinh vẫn nghĩ Địa lí là mơn khoa


học xã hội chỉ cần học thuộc lòng và nhớ thật nhiều số liệu là đủ. Thực tế,
nhiều học sinh có kiến thức nhưng điểm thi học sinh giỏi lại không cao. Tỉ lệ
bài đạt 7-8 điểm mơn Địa lí trong các kì thi học sinh giỏi các cấp không nhiều.
Một trong những nguyên nhân quan trọng dẫn đến tình trạng này là do thí sinh
cịn lúng túng trong việc nhận dạng câu hỏi và cách làm bài thi. Tác giả thông
qua việc tổng kết các đề thi học sinh giỏi các cấp, nhất là ở lớp 9 những năm
qua để khái quát hóa thành các dạng câu hỏi lí thuyết chủ yếu với hướng dẫn
cách giải cụ thể hy vọng phần nào khắc phục được thực trạng trên.


</div>
<span class='text_page_counter'>(67)</span><div class='page_container' data-page=67>

<b>B. NỘI DUNG</b>


<b>I. Dạng câu hỏi trình bày</b>
<b>1) Nhận biết</b>


- Dựa vào hình thức câu hỏi để xác định. Nếu trong câu hỏi xuất hiện các
từ và cụm từ như “trình bày”, “nêu”, “phân tích”, “hãy cho biết”, “thế nào”,
“gì”, “như thế nào”…


<b>Ví dụ:</b>


VD1: Nêu đặc điểm của các loại hình quần cư ở nước ta.



VD2: Phân tích mối quan hệ giữa tỉ lệ gia tăng dân số tự nhiên và cơ cấu dân số
theo độ tuổi ở nước ta


VD3: Các nhân tố tự nhiên ảnh hưởng như thế nào đến sự phát triển và phân bố
nơng nghiệp nước ta?


VD4:Trình bày sự chuyển dịch cơ cấu kinh tế ở nước ta.


VD5:Hãy cho biết những khó khăn do thiên nhiên gây ra cho ngành thủy sản?
- Lưu ý: có những câu hỏi về hình thức học sinh hay nhầm lẫn là dạng câu hỏi trình
bày nhưng thực chất lại thuộc dạng câu hỏi khác.


VD:Phân tích những điểm khác biệt giữa quần cư nơng thơn và thành thị ở nước ta.
Về hình thức học sinh dễ nhầm lẫn sang dạng trình bày nhưng thực chất đây là
dạng so sánh (so sánh khác biệt)


<b>2) Cách giải</b>


Dạng câu hỏi trình bày là dạng câu hỏi dễ nhất trong số các dạng câu hỏi thường
gặp trong các đề thi HSG mơn Địa lí. Qui trình chung để trả lời dạng câu hỏi này
nên theo các bước cơ bản sau:


- Đọc kĩ yêu cầu đề bài và nhận biết đúng dạng câu hỏi
- Tái hiện kiến thức đã học để trả lời câu hỏi.


</div>
<span class='text_page_counter'>(68)</span><div class='page_container' data-page=68>

- Căn cứ vào câu hỏi, thí sinh cần phải sắp xếp các kiến thức cơ bản sao cho phù
hợp.


(Gợi ý: sắp xếp các ý từ khái quát đến cụ thể, từ quan trọng đến ít quan trọng; Các


yếu tố tự nhiên trình bày trước sau đó trình bày đến các yếu tố kinh tế - xã hội,
thuận lợi trình bày trước khó khăn…)


*Lưu ý: khi trình bày bài làm u cầu HS trả lời thẳng vào yêu cầu câu hỏi, tránh
tình trạng trình bày lan man, dài dịng. u cầu HS tách bạch các ý, khơng bỏ sót
ý.


<b>3)Phân loại câu hỏi</b>


Các câu hỏi thuộc dạng trình bày rất đa dạng về nội dung. Khi cần kiểm tra kiến
thức cơ bản của thí sinh, người ta có thể đặt câu hỏi ở bất cứ nội dung nào trong
sách giáo khoa Địa lí 9. Do đó, khơng đặt ra việc phân loại các dạng câu hỏi.
<b>4) Luyện tập</b>


Các câu hỏi thuộc dạng trình bày rất đa dạng về nội dung nhưng thường yêu cầu
trình bày về các nhân tố ảnh hưởng hoặc mối liên hệ. Các nội dung này chiếm một
tần suất khá lớn.


<i><b>Một số câu hỏi về các nhân tố ảnh hưởng.</b></i>


„Các nhân tố ảnh hưởng“ có thể được dùng bằng các cụm từ khác gần nghĩa như
„điều kiện“, „nguồn lực“, „thế mạnh và hạn chế“, „thuận lợi và khó khăn“…


VD1:Phân tích các nhân tố ảnh hưởng đến sự phát triển và phân bố nông nghiệp
nước ta.


VD2: Các nhân tố kinh tế - xã hội ảnh hưởng đến sự phát triển và phân bố nông
nghiệp nước ta như thế nào?


VD3: Các nhân tố tự nhiên ảnh hưởng đến sự phát triển và phân bố nông nghiệp


nước ta như thế nào?


</div>
<span class='text_page_counter'>(69)</span><div class='page_container' data-page=69>

VD5: Phân tích những thuận lợi và khó khăn về mặt tự nhiên để phát triển ngành
nuôi trồng thủy sản nước ta.


VD6: Phân tích những thuận lợi và khó khăn về mặt tự nhiên để phát triển ngành
khai thác thủy sản nước ta.


VD7:Phân tích các nhân tố ảnh hưởng đến sự phát triển và phân bố công nghiệp
nước ta.


Gợi ý:


- Các VD1,VD2,VD3 đều hỏi về các nhân tố ảnh hưởng đến ngành nông
nghiệp nhưng phạm vi yêu cầu trả lời khác nhau (VD1 yêu cầu trả lời tất cả
các yếu tố, còn VD2 và VD3 chỉ là 1 phần trong VD1). Nếu học sinh làm
thừa hoặc thiếu các yếu tố đều bị trừ điểm.


- Cần thấy được sự khác nhau trong các nhân tố ảnh hưởng đến sự phát
triển và phân bố ngành nông nghiệp và công nghiệp (VD1,VD7). Các nhân tố
tự nhiên tác động đến nông nghiệp là tài ngun đất, khí hậu, nước và sinh vật
cịn với CN quan trọng hàng đầu trong các yếu tố tự nhiên là khống sản sau
đó mới đến thủy năng và đất, khí hậu, rừng…Cơ sở hạ tầng và cơ sở vật chất
kĩ thuật trong nông nghiệp HS phải chỉ ra được cụ thể là giao thông nông
thôn, mạng lưới điện, hệ thống thủy lợi, cơ sở chế biến, dịch vụ trồng trọt,
chăn nuôi. Cơ sở hạ tầng, cơ sở vật chất kĩ thuật trong công nghiệp lại là giao
thông vận tải nói chung, các cơ sở cơng nghiệp, nhà máy, trung tâm công
nghiệp…


Các nhân tố


ảnh hưởng


Nông nghiệp Công nghiệp


ĐKTN và
TNTN


-Đất(các loại,diện tích, phân bố)
 ảnh hưởng cơ cấu cây trồng.
-KH (đặcc điểm nhiệt,ẩm, phân
hóa)→cơ cấu cây trồng, cơ cấu
mùa vụ; biện pháp canh tác,
năng suất…


</div>
<span class='text_page_counter'>(70)</span><div class='page_container' data-page=70>

-Nước (mặt+ngầm)phát triển
thủy lợi


-SV tự nhiênthuần dưỡng, lai tạo
giống


- Thủy năng sông suối  pt
CN năng lượng.


- TN đất, nước, KH, rừng,
Sv biển…PT N-L-NPt
CNCB.


- Sự phân bố tài nguyên trên
lãnh thổ tạo thế mạnh khác
nhau giữa các vùng.



ĐK kinh tế
-xã hội


-Dân cư và lao động nông thôn
(dân số nông thôn, LĐ nông
thôn, kinh nghiệm sx).


-CSHT,CSVCKT:giao thông,
mạng lưới điện nông thôn; hệ
thống thủy lợi, cơ sở CB, dịch vụ
trồng trọt, chăn nuôi…


- Chính sách PT NN:khoán
ruộng đất, pt kinh tế trang trại,
hộ gia đình, XD vùng chuyên
canh…


- Thị trường trong và ngoài nước
-Khác (vốn…)


-Dân cư-LĐ(số dân, khả
năng tiếp thu KHKT)→ cơ
cấu ngành CN, khả năng thu
hút đầu tư.


- CSHT,CSVCKT:


GTVT,TTLL, điện; các nhà
máy, TTCN…



- Chính sách PTCN: C/s
CNH, thu hút đầu tư, đa
đạng hóa các thành phần
kinh tế…


- Thị trường trong và ngoài
nước


-Khác (l/s khai thác lãnh
thổ…)


<i><b>Một số câu hỏi trình bày về mối liên hệ.</b></i>


</div>
<span class='text_page_counter'>(71)</span><div class='page_container' data-page=71>

VD2: Giữa dân số với lao động và việc làm có mối quan hệ với nhau như thế nào?
VD3: Phân tích ảnh hưởng của chuyển dịch cơ cấu kinh tế tới vấn đề sử dụng lao


động và vấn đề việc làm ở nước ta.
<b>II. Dạng câu hỏi chứng minh.</b>


<b>1) Yêu cầu</b>


- Nhận dạng câu hỏi dựa vào các từ và cụm từ „Chứng minhh rằng“, „chứng
minh“, „lấy ví dụ“…


- Nắm vững kiến thức cơ bản. Học sinh nhớ số liệu liên quan đến yêu cầu của câu
hỏi.


- Biết sàng lọc, lựa chọn kiến thức cũng như số liệu cần thiết để đưa ra bằng
chứng.



<b>2) Phân loại các câu hỏi</b>


- Loại câu hỏi chứng minh hiện trạng


Bao gồm tất cả các hiện tượng tự nhiên, kinh tế - xã hội đang tồn tại.
- Loại câu hỏi chứng minh tiềm năng


<b>3) Hướng dẫn cách giải</b>


<b>a)Loại câu hỏi chứng minh hiện trạng</b>


- Chứng minh hiện trạng về địa lí dân cư và các nội dung liên quan


Rất nhiều nội dung có thể được sử dụng để đặt câu hỏi dưới dạng chứng minh như:
đặc điểm chung dân cư cả nước, đặc điểm lao động, vấn đề lao động và việc làm...
- Chứng minh hiện trạng về địa lí kinh tế


Về phương diện địa lí kinh tế các câu hỏi dạng chứng minh thường có liên quan
đến các ngành, các vùng lãnh thổ hoặc nội dung kinh tế của vùng.


Quy trình giải loại câu hỏi chứng minh hiện trạng nên tiến hành theo các bước sau
đây:


</div>
<span class='text_page_counter'>(72)</span><div class='page_container' data-page=72>

+ Hệ thống hoá kiến thức và số liệu có liên quan


Về số liệu nên lưu ý đến những số liệu cơ bản nhất, đặc biệt tại mốc thời gian quan
trọng có liên quan đến việc chứng minh. Số liệu không cần nhớ quá nhiều nhưng
nhất thiết không được quên các mốc thời gian quan trọng.



+ Dùng kiến thức cơ bản và số liệu đã chọn lọc để đưa ra các bằng chứng có tính
thuyết phục cao theo u cầu câu hỏi.


<b>b) Loại câu hỏi chứng minh tiềm năng</b>


Loại câu hỏi chứng minh tiềm năng nhìn chung là dễ và tương đối đơn giản. Nó
gần như chỉ có một cách hỏi, liên quan đến tiềm năng ( thế mạnh hay hạn chế) của
một ngành hay một vùng lãnh thổ.


Các tiềm năng của một ngành hoặc một vùng lãnh thổ thường được thể hiện ở các
mặt:


+ Vị trí địa lí


+ Điều kiện tự nhiên và tài nguyên thiên nhiên
+ Điều kiện kinh tế - xã hội


<b>4) Luyện tập</b>


<b>a)VD1:Chứng minh rằng nước ta có tiềm năng lớn về lao động, nhưng chưa được</b>
sử dụng hợp lý.


* Tiềm năng lớn về lao động .


- Số lượng: nguồn lao động nước ta dồi dào và tăng nhanh (d/c)
- Chất lượng:


+ Người lao động nước ta có nhiều kinh nghiệm sản xuất, cần cù, ham học hỏi, có
khả năng tiếp thu khoa học kĩ thuật.



+ Chất lượng lao động tăng lên (d/c)
- Phân bố lao động:


+ Ở các vùng kinh tế phát triển tập trung nhiều lao động, nhất là lao động có
chun mơn kỹ thuật.


</div>
<span class='text_page_counter'>(73)</span><div class='page_container' data-page=73>

* Sử dụng lao động chưa hợp lí.


- Trong các ngành kinh tế: Tỉ lệ lao động nơng nghiệp vẫn cịn khá cao, chuyển
dịch cơ cấu lao động theo ngành diễn ra chậm (d/c)


-Tỉ lệ thất nghiệp và thiếu việc làm của nước ta còn cao (d/c)


<b>b)VD2:Chứng minh rằng sự phát triển ngành thuỷ sản nước ta đạt được nhiều</b>
thành tựu trong những năm gần đây.


- Sản lượng thuỷ sản tăng nhanh (dc).


- Trong đó sản lượng thủy sản nuôI trồng và đánh bắt đều tăng nhưng sản lượng
thủy sản nuôi trồng tăng nhanh hơn (dc)


- Trong cơ cấu ngành thuỷ sản, nuôi trồng thuỷ sản chiếm % ngày càng cao


- Xuất khẩu thuỷ sản phát triển vượt bậc (d/c), đây là đòn bẩy tác động đến tồn bộ
các khâu khai thác, ni trồng và chế biến thuỷ sản.


- Hình thành được các vùng trọng điểm sản xuất thuỷ sản (kể)...


<b>c)VD3: Chứng minh rằng tài nguyên thiên nhiên nước ta đa dạng tạo điều kiện để</b>
phát triển cơ cấu cơng nghiệp đa ngành.



<i><b>* Khống sản</b></i>


- Nhiên liệu: than, dầu khí… phát triển cơng nghiệp năng lượng, hóa chất


- Kim loại: sắt, mangan, crơm, thiếc, chì, kẽm, đồng, vàng… phát triển công
nghiệp luyện kim đen, luyện kim màu…


- Phi kim loại: apatit, pirit, photphorit…phát triển cơng nghiệp hóa chất…


- Vật liệu xây dựng: sét, đá vôi, cát thủy tinh...phát triển công nghiệp vật liệu xây
dựng...


</div>
<span class='text_page_counter'>(74)</span><div class='page_container' data-page=74>

<i><b>* Tài nguyên đất, nước, khí hậu, rừng, thủy sản... Phát triển công nghiệp chế</b></i>
biến nông, lâm, thủy sản


<b>III. Dạng câu hỏi giải thích</b>
<b>1)Nhận biết</b>


-Trong yêu cầu đề bài xuất hiện các từ và cụm từ “hãy giải thích”, “tại sao”, “vì
sao”, “giải thích vì sao”, “giải thích nguyên nhân”…


- Tránh nhầm lẫn với các dạng câu hỏi khác.


+ VD: Tại sao nói ngành thủy sản nước ta trong những năm gần đây phát triển
nhanh vượt bậc.


Về hình thức câu hỏi xuất hiện cụm từ “Tại sao nói” HS dễ nhầm sang dạng trình
bày nhưng thực chất đây là câu hỏi yêu cầu chứng minh. Cụ thể với câu hỏi này
HS phải chứng minh hiện trạng phát triển ngành thủy sản (giá trị sản xuất, sản


lượng TS, sản lượng TS khai thác, sản lượng TS nuôi trồng; khả năng xuất
khẩu…). Nếu trả lời theo dạng giải thích dựa vào các thế mạnh để phát triển thủy
sản bao gồm điều kiện TN và KT-XH là lạc đề.


<b>2)Hướng dẫn cách giải</b>


- Đọc kĩ câu hỏi, xác định đúng vấn đề cần giải thích
- Tái hiện kiến thức cơ bản


Yêu cầu HS phải nắm chắc kiến thức cơ bản không chỉ của 1 bài mà nhiều khi là 1
chương, thậm chí là cả chương trình. HS khơng chỉ thuộc mà địi hỏi phảihiểu
được bản chất của kiến thức và biết vận dụng những kiến thức đó để giải thích 1
hiện tượng. Đây là điều kiện cần nhưng chưa đủ.


-Tìm mối liên hệ, đặc biệt mối liên hệ nhân quả giữa các hiện tượng địa lí.


- Khái qt hố các kiến thức có liên quan đến câu hỏi và mối liên hệ của chúng để
tìm ra nguyên nhân.


</div>
<span class='text_page_counter'>(75)</span><div class='page_container' data-page=75>

<b>3)Phân loại</b>


Nếu căn cứ vào cách giải thì có thể phân thành hai loại:
- Loại câu hỏi có cách giải theo mẫu tương đối cố định.


+ Các dạng câu hỏi yêu cầu cách giải chủ yếu dựa vào nguồn lực.


+Các câu hỏi yêu cầu cách giải dựa trên cơ sở khái niệm đã có trong SGK.
+ Các câu hỏi yêu cầu cách giải dựa vào vai trị.


- Loại câu hỏi có cách giải không theo một mẫu nhất định. Đối với câu hỏi này thí


sinh phải vận dụng các kiến thức đã có, tìm mối liên hệ để phát hiện ra ngun
nhân.


<b>a) Loại câu hỏi có cách giải theo mẫu dựa vào việc phân tích nguồn lực.</b>


*Cách giải: Về lí thuyết, nguồn lực để phát triển kinh tế - xã hội bao gồm những
nội dung chính sau đây:


+ Vị trí địa lí


Chú ý: Vị trí địa lý với nguồn lực tự nhiên là hai nguồn lực khác nhau nên học sinh
không được xếp vị trí địa lý vào nguồn lực tự nhiên


+ Nguồn lực tự nhiên: Địa hình, đất, khí hậu, thuỷ văn, sinh vật, khoáng sản.


+ Nguồn lực kinh tế - xã hội: Dân cư và nguồn lao động, kết cấu hạ tầng, cơ sở vật
chất - kỹ thuật, thị trường, đường lối chính sách, các nguồn lực khác.


Việc vận dụng các nguồn lực để giải thích theo yêu cầu của câu hỏi cần phải linh
hoạt, tránh sự rập khuân, máy móc. Đối với mỗi đối tượng địa lý cần xác định các
yếu tố nào là cơ bản có liên quan, nếu khơng ảnh hưởng thì loại bỏ như khi phân
tích nhân tố ảnh hưởng tự nhiên đến nơng nghiệp thì tài ngun khống sản cần
loại bỏ.


Ngồi ra, về lí thuyết khi đề cập tới nguồn lực là bao hàm cả thế mạnh ( thuận lợi)
và hạn chế ( khó khăn).


</div>
<span class='text_page_counter'>(76)</span><div class='page_container' data-page=76>

<b>VD1:Vì sao ĐBCL là vùng trọng điểm lúa lớn nhất cả nước?</b>


- Diện tích lớn nhất cả nước: 4 triệu ha lớn hơn diện tích ĐBSH và DHMT cộng lại


- Thế mạnh về tự nhiên đặc biệt thuận lợi.


- Thế mạnh về kinh tế - xã hội (thị trường trong và ngồi nước, dân cư – lao động.
chính sách, csvckt - ht...)


<b>VD2: Vì sao ĐBSH có mật độ dân số lớn nhất cả nước?</b>


- Trình độ phát triển kinh tế, tính chất nền kinh tế: trình độ phát triển là vùng kinh
tế năng động t2 cả nước, CN-DV phát triển; tính chất thâm canh lúa nước.


- Thuận lợi về tự nhiên (địa hình, đất, nước)
- Lịch sử khai thác lãnh thổ lâu đời


<b>VD3:Tại sao ngành thủy sản gần đây phát triển nhanh ?</b>


<b>VD4: Giải thích tại sao trong thời gian qua sản xuất lương thực nước ta đạt nhiều</b>
thành tựu to lớn.


<b>VD5: Vì sao chăn ni cịn chiếm tỷ trọng thấp trong cơ cấu giá trị sản xuất nơng</b>


nghiệp của nước ta?


Vì nước ta thiếu những điều kiện thuận lợi cho phát triển chăn nuôi:


+ Cơ sở thức ăn chưa ổn định cững chắc (Diện tích đồng cỏ nhỏ, phân tán, có
nhiều cỏ tạp khó cải tạo, năng suất đồng có thấp, thức ăn CN CB chưa nhiều...)
+ CSHT, csvckt phục vụ chăn ni cịn thiếu; CNCB thức ăn chăn nuôi và công
tác dịch vụ thú y cịn hạn chế


+ Mơi trường khí hậu nhiệt đới ẩm dễ phát sinh dịch bệnh cho vật ni


+ Hình thức chăn nuôi chủ yếu theo lối quảng canh, lạc hậu


+ Giống gia súc, gia cầm năng suất thấp, chất lượng chưa cao, chưa đáp ứng nhu
cầu xuất khẩu.


</div>
<span class='text_page_counter'>(77)</span><div class='page_container' data-page=77>

gắng sắp xếp các nhân tố quan trọng lên trước, điều này tùy thuộc vào từng đối
tượng. Đây chính là trường hợp ví dụ1,ví dụ 2, ví dụ 3 và ví dụ 4.


Giải thích cho các đối tượng phát triển chậm, không ổn định, tỷ trọng thấp hoặc
giảm ta nên dựa chủ yếu vào những hạn chế, khó khăn (Ví dụ 5).


<b>b) Câu hỏi giải thích có cách giải dựa vào mẫu khái niệm.</b>


VD1: Giải thích tại sao CNCB lương thực, thực phẩm là ngành công nghiệp trọng
điểm ở nước ta hiện nay?


VD2: Giải thích tại sao CN sản xuất hàng tiêu dùng là ngành công nghiệp trọng
điểm ở nước ta hiện nay?


VD3: Giải thích tại sao CN điện lực là ngành công nghiệp trọng điểm ở nước ta
hiện nay?


Gợi ý:


- Trước hết phải nêu khái niệm về ngành CN trọng điểm.
- Dựa vào khái niệm để triển khai các ý tiếp theo.


+ Là ngành có thế mạnh lâu dài
+ Đem lại hiệu quả cao



+ Tác động đến các ngành kinh tế khác.


CNCB LT_TP CNSXHTD CN điện lực


THẾ MẠNH LÂU
DÀI


- Nguyên liệu tại chỗ
phong phú


+ Từ trồng trọt
+ Từ chăn nuôi
+ Từ thuỷ sản


- Thị trường(trong và


THẾ MẠNH LÂU DÀI


- Thị trường (ngoài
nước --> trong nước)
- lao động (dồi dào,
khéo tay, rẻ)


- Khác: nguyên liệu tại
chỗ, chính sách...


THẾ MẠNH LÂU DÀI


- Thị trường (đời sống, sx)
- Nguyên, nhiên liệu:


+ Than


+ Dầu khí
+ Thuỷ năng


+ NL khác: gió, MT...


</div>
<span class='text_page_counter'>(78)</span><div class='page_container' data-page=78>

ngồi nước)


- Khác:Lao động,
chính sách...


HIỆU QUẢ CAO
- Kinh tế


+ Vốn đầu tư ít, thời gian quay vòng vốn
nhanh, thu lợi nhuận tương đối dễ...


+ Đẩy nhanh tốc độ tăng trưởng, chuyển dịch
cơ cấu kinh tế.


+ Đóng góp các mặt hàng xuất khẩu chủ lực.
-XH:


+ Giải quyết việc làm cho 1 bộ phận lao động
+ Nâng cao CLCS


+ Giảm sự chênh lệch về trình độ phát triển
giữa các vùng trong nước.



HIỆU QUẢ CAO


-KT


+ Nâng cao năng suất lao
động


+ Nâng cao hiệu quả kinh tế
+ Đẩy nhanh tốc độ tăng
trưởng kinh tế, chuyển dịch
cơ cấu KT.


-XH:


+Nâng cao CLCS


+Giải quyết việc làm cho 1 bộ
phận lao động


+Giảm sự chênh lệch về trình
độ phát triển giữa các vùng
trong nước.


TÁC ĐỘNG MẠNH MẼ ĐẾN CÁC NGÀNH KT KHÁC
<b>-</b> Bản thân các ngành CN


<b>-</b> Các ngành cung cấp nguyên liệu


<b>-</b> Khác: GTVT, TM, tài chính, ngân hàng...



<b>c) Câu hỏi giải thích có cách giải dựa vào vai trị</b>


VD1: Tại sao thuỷ lợi là biện pháp hàng đầu trong thâm canh nông nghiệp ở nước
ta?.


VD2: Tại sao yếu tố chính sách được coi là “đòn bẩy” đối với sự phát triển và phân
bố nơng nghiệp nước ta?


VD3: Vì sao phải gắn các vùng chuyên canh CCN với các cơ sở CNCB?
<b>d) Câu hỏi giải thích khơng theo mẫu chung.</b>


* Qui trình:


- Đọc kĩ câu hỏi --> Định hướng trả lời


</div>
<span class='text_page_counter'>(79)</span><div class='page_container' data-page=79>

- Tìm ngun nhân, lí do
<b>*Ví dụ :</b>


<b>VD1:Vì sao dân số là 1 trong những vấn đề đang được quan tâm hàng đầu ở nước</b>
ta hiện nay?


- Định hướng: chủ yếu dựa vào thách thức (bài dân số, phân bố dân cư)
Trả lời:


-Con người là 1 trong những nguồn lực kinh tế- xã hội quan trọng nhất để phát
triển kinh tế (vừa là nguồn lao động, vừa là thị trường tiêu thụ)


- Đặc điểm dân số nước ta
+ Qui mô dân số đông (d/c)
+ Gia tăng dân số nhanh (d/c)



+ Cơ cấu trẻ nhưng đang có nhiều thay đổi (d/c)
+ Phân bố dân cư không đồng đều (d/c)


-Đặc điểm dân số như trên gây nhiều hậu quả về kinh tế - xã hội và môi trường
(diễn giải)


<b>VD2: Tại sao việc làm là vấn đề kinh tế- xã hội gay gắt ở nước ta hiện nay?</b>
<b>VD3: Tại sao nước ta phải quan tâm đến vấn đề đơ thị hóa?</b>


<b>VD4: Tại sao lương thực là mối quan tâm thường xuyên của nước ta?</b>
<b>IV. Dạng câu hỏi so sánh</b>


<b>1)Yêu cầu</b>


- Trước hết phải nắm vững kiến thức cơ bản.


- Tiếp theo cần hệ thống hoá, phân loại và sắp xếp kiến thức theo từng nhóm riêng
biệt để dễ dàng cho việc so sánh.


- Biết cách khái quát hoá kiến thức đã có để tìm ra các tiêu chí so sánh.
<b>2) Phân loại các câu hỏi</b>


</div>
<span class='text_page_counter'>(80)</span><div class='page_container' data-page=80>

Chỉnh thể được hiểu là một đối tượng hay hiện tượng địa lí kinh tế - xã hội tương
đối hồn chỉnh, ví dụ như vùng kinh tế hay ngành kinh tế.


Ví dụ1: So sánh vùng chuyên canh cây công nghiệp lâu năm Tây Nguyên với
Trung du miền núi Bắc Bộ.


Ví dụ 2: So sánh vùng trọng điểm sản xuất lúa Đồng bằng Cửu Long với Đồng


bằng sông Hồng.


- Loại câu hỏi yêu cầu phải so sánh chỉ một khía cạnh nào đó của hai hay nhiều
chỉnh thể.


Ví dụ1: So sánh thế mạnh tự nhiên để phát triển cây công nghiệp lâu năm giữa Tây
Nguyên với Trung du miền núi Bắc Bộ.


Ví dụ 2: So sánh thế mạnh phát triển sản xuất lúa giữa Đồng bằng Cửu Long với
Đồng bằng sơng Hồng.


Ví dụ 3: Phân tích sự khác biệt về qui mơ và cơ cấu ngành giữa 2 trung tâm công
nghiệp Hà Nội và TP Hồ Chí Minh.


<b>3) Hướng dẫn cách giải.</b>
<b>a)Hướng dẫn chung:</b>


- Tìm ra sự giống và khác nhau giữa các đối tượng cần phải so sánh.
- Xác định các tiêu chí để so sánh.


(Để xác định được tiêu chí phải khái quát hóa kiến thức, phân loại và sắp xếp kiến
thức theo từng nhóm riêng biệt )


- So sánh theo các tiêu chí bằng kiến thức cơ bản đã được chọn lọc.
<b>b)Hướng dẫn cụ thể:</b>


*Loại câu hỏi so sánh hai hay nhiều chỉnh thể với nhau.


Khi so sánh hai hay nhiều ngành với nhau, có thể lựa chọn tiêu chí dựa theo mẫu
dưới đây:



</div>
<span class='text_page_counter'>(81)</span><div class='page_container' data-page=81>

- Nguồn lực để phát triển
- Tình hình phát triển
- Cơ cấu ngành


- Tình hình phân bố
- Hướng phát trỉên


Các nội dung theo mẫu về ngành và về vùng có nhiều điểm tương đồng. Khi so
sánh giữa các vùng lãnh thổ, có thể xác định tiêu chí căn cứ vào mẫu sau đây:
- Vị trí địa lí, vai trị và quy mơ của vùng


- Các nguồn lực để phát triển
- Hướng chun mơn hố


- Tình hình phát triển và phân bố các ngành kinh tế trong vùng.
- Hướng phát triển


Chú ý, hai mẫu trên đưa ra các nội dung ở mức tối đa. Trên thực tế, khơng phải lúc
nào cũng sử dụng tồn bộ các tiêu chí này để so sánh. Phụ thuộc vào câu hỏi cụ
thể, học sinh đưa ra các tiêu chí thích hợp nhất.


*Loại câu hỏi so sánh một khía cạnh nào đó của hai hay nhiều chỉnh thể.


Loại câu hỏi này đỏi hỏi so sánh không phải các đối tượng địa lí kinh tế - xã hội
với tư cách như các chỉnh thể, mà chỉ là một khía cạnh ( một phần) của chúng mà
thơi.


Có rất nhiều khía cạnh có thể được đưa ra để thiết kế câu hỏi dạng so sánh. Có thể
ví dụ một vài khía cạnh chủ yếu sau đây:



- So sánh về thế mạnh/ nguồn lực
- So sánh về tình hình phát triển
- So sánh về cơ cấu


</div>
<span class='text_page_counter'>(82)</span><div class='page_container' data-page=82>

Đối với câu hỏi so sánh về tình hình phát triển, các tiêu chí để so sánh có thể là:
- Giai đoạn ( thời kì) phát triển


- Nhịp độ phát triển
- Sản phẩm tiêu biểu


Đối với câu hỏi so sánh về cơ cấu, các tiêu chí để so sánh có thể là:
- Giai đoạn và sự chuyển dịch


- Cơ cấu theo ngành
- Cơ cấu theo lãnh thổ


Đối với những câu hỏi so sánh về phân bố, các tiêu chí có thể so sánh là:
- Đặc điểm phân bố


- Sự phân bố theo giai đoạn


- Mức độ hợp lí ( hay chưa hợp lí)...
<b>4) Luyện tập</b>


<b>a)VD1: So sánh thế mạnh để phát triển sản xuất lúa ở Đồng bằng sông Cửu Long</b>
và Đồng bằng sông Hồng.


<i>Giống nhau.</i>



*Vai trị và quy mơ:


- Cả hai đồng bằng đều là châu thổ rộng nhất, nằm ở hạ lưu hai hệ thống sông lớn
nhất của nước ta.


- Đây là hai vùng trọng điểm sản xuất lúa quan trọng nhất của nước ta, là hai vùng
có vai trị quyết định trong việc đảm bảo nhu cầu lương thực trong nước.


* Điều kiện tự nhiên và tài nguyên thiên nhiên:


- Địa hình tương đối bằng phẳng thuận lợi cho phát triển cây lúa (vận chuyển vật
tư và sản phẩm nông nghiệp, canh tác...).


</div>
<span class='text_page_counter'>(83)</span><div class='page_container' data-page=83>

- Khí hậu nhiệt đới ẩm, nắng lắm, mưa nhiều tạo điều kiện thuận lợi cho cây lúa
sinh trưởng và phát triển quanh năm.


- Có các hệ thống sông lớn với lưu lượng nước phong phú, thuận lợi phát triển thủy
lợi.


*Điều kiện kinh tế - xã hội:


- Hai vùng có số dân đơng, mật độ dân số cao; nguồn lao động dồi dào với kinh
nghiệm trồng lúa nước.


- Có nhiều cơ sở chế biến nguyên liệu từ nơng nghiệp; hình thành và phát triển hệ
thống đơ thị, trong đó có những đơ thị vào loại lớn của cả nước (Hà Nội, Hải
Phòng, Cần Thơ...).


- Nhà nước quan tâm đầu tư, thị trường mở rộng…



<i>Khác nhau.</i>


* Vai trị và quy mơ:


- ĐBSCL là vùng trọng điểm số 1 về sản xuất lúa, ĐBSH là vùng trọng điểm sản
xuất lúa đứng thứ 2.


- Xét về một số chỉ tiêu, ĐBSCL có quy mơ lớn hơn ĐBSH (diện tích tự nhiên,
diện tích gieo trồng lúa, sản lượng lúa, bình quân sản lượng lúa trên đầu người, khả
năng xuất khẩu).


* Điều kiện tự nhiên và tài nguyên thiên nhiên:
- Địa hình và đất đai:


+ ĐBSCL có diện tích lớn hơn, địa hình bàng phẳng, thấp hơn và khơng có hệ thống đê
nên hằng năm vẫn được bồi đắp phù sa thuận lợi cho sản xuất nơng nghiệp; ĐBSCL
cịn nhiều diện tích đất hoang hóa nên khả năng mở rộng diện tích đất canh tác lớn hơn.
+ĐBSH diện tích tự nhiên nhỏ hơn, có đê ngăn lũ, đất khơng được bồi phù sa hàng
năm…


- Khí hậu:


+ĐBSCL có khí hậu cận xích đạo, nóng quanh năm, lượng mưa lớn, khí hậu ổn định
thuận lợi cho cây lúa phát triển hơn, khả năng tăng vụ lớn 2-3 vụ lúa/năm.


+ĐBSH có khí hậu nhiệt đới gió mùa có một mùa đơng lạnh có thể trồng tối đa 2 vụ
lúa, có vụ đơng xn.


</div>
<span class='text_page_counter'>(84)</span><div class='page_container' data-page=84>

* Điều kiện kinh tế - xã hội:
- Dân cư – lao động



+ĐBSH dân cư đông đúc hơn, lao động có nhiều kinh nghiệm thâm canh lúa nước
cao hơn ĐBSCL. Vì vậy, năng suất lúa ở đây đứng hàng đầu cả nước.


+ĐBCL mật độ dân số thấp hơn nhưng người dân có kinh nghiệm sản xuất lúa
hàng hóa.


- ĐBSH có hệ thống cơ sở hạ tầng –cơ sở vật chất kĩ thuật hoàn thiện hơn của
ĐBSCl (d/c).


- Các điều kiện khác: lịch sử khai thác lãnh thổ, vốn đầu tư…
<b>b)VD2: Phân biệt quần cư nông thôn và thành thị ở nước ta.</b>


Tiêu chí Quần cư nơng thơn Quần cư thành thị
Chức năng


- Sản xuất nơng nghiệp là chính


- Nhiều chức năng (Các TP là
những trung tâm kinh tế, chính trị,
văn hóa, khoa học kĩ thuật quan
trọng)


- Sản xuất CN-DV là chính
Phân bố Phân tán, trải rộng theo không


gian.


Tập trung, mật độ dân số cao



Cách thức
tổ chức
sinh sống


Các điểm quần cư là làng, ấp,
bản…


-Thành phố, thị xã, thị trấn


- Kiểu “nhà ống” san sát nhau,
chung cư cao tầng được xây dựng
nhiều. Ngồi ra có kiểu nhà biệt
thư, nhà vườn…


<b>C. KẾT LUẬN</b>


</div>
<span class='text_page_counter'>(85)</span><div class='page_container' data-page=85>

<b>Examining the plausibility of Extensive Reading as an approach to learning </b>
<b>English at a secondary school context.</b>


<b>Thuy Tran Thi – Vinhphuc Specialized High School</b>
Her students’ increasing need for exposure to more authentic language input,
together with the writer’s wish to boost those students’ autonomy has led to this
theoretical examination of the plausibility of introducing Extensive Reading into
her teaching context- a tracking school for the gifted in Vinh Yen, Vinh Phuc.
<b>Part 1: Introduction</b>


<b>Reasons for choosing the study</b>


Various attempts at different levels have been made to improve the quality of
teaching and learning English at school. In fact, the Ministry of Education and


Training has recently suggested that by the year 2020, at least half of the young
should be able to use English fluently, and this is a goal for teachers of English and
education authorities to work towards. The main challenges lay, however, in the
lack of human resources to cater for such need and the lack of motivation to learn
English from the students.


In a workshop in 2011, the writer was exposed to Extensive Reading, an “aid
to language learning” which has been reported to work in many an Asian countries.
By doing a great amount of reading under the language teacher’s instruction and
facilitation, students are claimed to make steady progress in all language


components and language skills. It is therefore believed that extensive reading can
be the key to language improvement which may be appropriate in our educational
setting, since it provides an alternative to the more common teacher-centred mode
in Vietnam and can hence deal with the two aforementioned issues effectively.
Implementing a programme of extensive reading in a particular school, however,
needs elaborate preponderance. This study is therefore to serve as a theoretical
basis for the official formation of an extensive reading club within a senior
secondary school for the gifted in Vinh Phuc.


<b>Aims and significance of the study</b>


</div>
<span class='text_page_counter'>(86)</span><div class='page_container' data-page=86>

<b>Subjects of the study</b>


This being a theoretical review, the primary subject of the study is the vast amount
of literature on extensive reading. Besides, there are 20 students participating in the
informal discussions on their study needs and study preferences.


<b>Limits of the study</b>



Should there be sufficient resources for an actual pilot implementation of the
programme to support the review, this study could be considerably improved.
<b>Methodology</b>


The study involves reviewing and summarising the literature on extensive reading.
This is followed by observation of and informal conversations with 20 students in
an advanced English class in the school.


<b>Research time</b>


March 2014 –February 2015
<b>Part 2. The Study</b>


<b>1. Extensive reading: A definition</b>


Extensive reading is understood as an approach to language learning/teaching
which is “generally associated with reading large amounts with the aim of getting
an overall understanding of the material. Readers are more concerned with the
meaning of the text than the meaning of individual words or sentences.” (Day
&Bramford, 2002).


<b>2. The benefits of extensive reading</b>


Bell (1998) summarised a number of advantages of extensive reading, from
positive psychological aspects to real language enhancement, of which the most
important are listed below:


<i><b>a. The provision of authentic, 'comprehensible input' </b></i>


It has always been proposed that that extensive reading will lead to language


acquisition, provided that certain preconditions are met, including adequate


exposure to the language, interesting material, and a learner-friendly environment.


<i><b>b. The enhancement of learners' general language competence </b></i>


</div>
<span class='text_page_counter'>(87)</span><div class='page_container' data-page=87>

<i><b>c. The increase of the students' exposure to the language </b></i>


It is a fact that the quality of exposure to language that learners receive is of great
importance to their ability to acquire knowledge, whether in terms of grammar or
lexico, from the input. Elley’s (1991:404) research subjects in an extensive reading
programme showed rapid growth in language development compared with learners
in regular language programs. He claimed to have seen a "spread of effect from
reading competence to other language skills - writing, speaking and control over
syntax,"


<i><b>d. Increased knowledge of vocabulary </b></i>


It is suggested that only a small percentage of vocabulary learning is due to direct
vocabulary instruction, the remainder is acquisition of words from reading.


Traditional approaches to the teaching of vocabulary, in which the number of new
words taught in each class was carefully controlled (words often being presented in
related sets), is therefore not sufficient in promoting vocabulary growth. Exposing
leaners to reading text can fill in the gap.


<i><b>e. Improvement in writing </b></i>


With such regular peruse of “comprehensible input”, there have been reported
significant improvements in subjects' written work. Extensive reading is


accordingly an effective means of fostering improvements in students writing.


<i><b>f. Increased motivation for learners to read </b></i>


As Bell (1998) put it “reading material selected for extensive reading programs
should address students' needs, tastes and interests, so as to energize and motivate
them to read the books”. With some need analysis in advance, the


organiser/teacher can easily identify the appropriate genre(s) for their students to
read, for example by means of informal discussion or the delivery of a need
analysis questionnaire. In many cases, familiar material and popular titles


reflecting the local culture, or, paradoxically, stories of faraway land and strange
customs and practices could be the most popular among school children and even
teenagers.


<i><b>g. The consolidation of previously learned language </b></i>


The authentic, or near-authentic choice of books to read also mean that by reading
students can experience what is termed the “recycling” of language. This is by
itself a form of reinforcement which helps to ensure that new input is “retained and
made available for spoken and written production” (Bell, 1998).


</div>
<span class='text_page_counter'>(88)</span><div class='page_container' data-page=88>

It is the case that all classroom reading work in our course books are traditionally
focused on the exploitation of shorts texts. Gradually the average student could be
accustomed to reading those “simple-looking” texts that he/she would back up
when asked to read longer, more complex ones for academic purposes. Extensive
reading can therefore provide a “fun”, stress-free approach to the acclimatisation of
students with these longer texts.



<b>3. Principles of an extensive reading programme</b>


Drawing on their experience with different extensive reading programmes,


researchers (Day, 2002; Bramford, 1998; Maley, 2008) have proposed a number of
criteria on which to develop a standard programme which could be useful for
future reference. Some of the most widely known and well supported ones are as
follows:


<i><b>a. Maximize Learner Involvement </b></i>


This is meant to reduce the feeling of obligation which may impede students’
automaticity both during the whole programme and during their individual act of
reading. Particularly, students should be encouraged discuss with the teacher to
identify their own interest in reading, for instance, whether it is books about the
environment/space or fiction books. For students in Nguyen Tat Thanh senior
secondary school for the gifted, this involvement is highly desirable. From a
psychological point of view, most of the students here, being teenagers, have very
strong face-saving needs and the teacher’s failure to give them a sense of


“ownership” of their reading project may cause feelings of compulsion and later
dissatisfaction.


<i><b>b. Provide a wide range of texts </b></i>


It is strongly believed that some variety in terms of genre and length of texts can
minimize the student’s chance of feeling bored. For the success of any lengthy
project, there should be abundant reading materials available in a range of genre
and length. The teacher should note, however, that whatever type of material, the
language presented must be within the language competence of the reader-student.



<i><b>c. Avoid the Use of Tests </b></i>


As testing in our context is often associated with rote learning, memorization and
lack of freedom, extensive reading done at home or during an agreed period of
time assigned should be totally detached from testing.


<i><b>d. Discourage the Over-Use of Dictionaries </b></i>


</div>
<span class='text_page_counter'>(89)</span><div class='page_container' data-page=89>

that, for the extensive reader-learner "Use of the dictionary was sparing, with the
main focus on meaning". Similarly, Day (2002) suggests no dictionary use during
reading as most of the chosen material should be “comprehensible input”


<i><b>e. The teacher’s role</b></i>


There is a shift in the roles that the teacher plays in an extensive reading


programme with those she/he does in a normal classroom setting. He/she no longer
directs the students’ learning. Rather, she/he facilitates it. Besides, Bramford
(1998) points out that in order to administer the resources efficiently, and to trace
students' developing reading habits and interests, effective monitoring from the
teacher is fundamental.


<i><b>f. Maintain the Entertainment.</b></i>


This is in fact the key to any extensive reading programme. Only when students
are delighted with what they are doing can they continue with their work. Once the
fun is kept, automaticity and self-adjustment along the way can be more feasible.
Entertainment can be achieved by different ways yet the vast amount of



responsibility is on the teacher as a facilitator. Her/his creativity, suitable
follow-up activities, good management skills may help.


<b>4. A model of an extensive reading club in Vinh Phuc senior secondary school </b>
<b>for the gifted</b>


The above analysis of the literature on extensive reading has made a case for an
extensive reading club to be set up in the next school year. Below is a step-by-step
proposal for such a project.


<i><b>Membership intake</b></i>


It is suggested that membership should be open to everyone, but should primarily
target at English-majored 10 graders who may have more motivation to persevere
with the project. My observation and informal conversation with students from
different grades has confirmed me that for 11 or 12 graders the amount of time
required by the project may be a source of pressure for them as they are


</div>
<span class='text_page_counter'>(90)</span><div class='page_container' data-page=90>

<i><b>Raising fund </b></i>


For the project to work, the teacher has to make sure that the “comprehensible
input” students get exposed to is abundant and validated. This means at least 200
books need purchasing from reliable publishers. The finance for the buying and
locating of those books may come from different sources, namely:


PTA: A sensitive issue, yet, most feasible in our school context where many
parents can be willing to support such a programme, especially those whose
children are members of the club.


ERF: The extensive reading foundation also offers some financial support to


certain reading programme. A proposal on the programme, including the detailed
budget should be submitted to the grant committee for consideration.


<i><b>Need and Competence analysis</b></i>


An informal meeting among the members and teacher could easily help identify
the kinds of books to be purchased. There is a wide range of choice covering
almost every genre, from poems to adapted novels to suit the learner-reader’s
needs.


However, the members’ competence has to be examined carefully by means of
testing their speed of reading comprehension. With students being at around the
same grade, it is supposed that the levels of their competence are more or less the
same. Again, automaticity should be encouraged. In other words, students are
given freedom to decide for themselves which level of difficulty is the most
suitable for him. The analysis has implications for the purchase of materials. All
extensive reading materials are graded according to their levels of difficulty (see
the sample attached on page 14). If, for instance, the students are comfortable with
reading a text at level 4, then she/he should buy only those from level 4 and up.
Prior to the discussion and competence analysis, the teacher should prepare a form
to note down all those needs (see below)


<b>Name</b> <b>Genre 1</b> <b>Genre 2</b> <b>Level</b>


<i><b>Purchasing/ Accumulating material </b></i>


</div>
<span class='text_page_counter'>(91)</span><div class='page_container' data-page=91>

for the purpose can be downloaded online for free. Please find attached on page 14
a sample provided.


<i><b>e. Other arrangements: time, location, regulations</b></i>



These should be talked through and agreed upon among the members with the
teacher playing the role of a key member willing to offer help and a more “global”
vision.


<b>f. Extensive reading: 1st cycle</b>


The club will last for all the three years of the student’s senior secondary school.
After each year, reflection is necessary and more books can be bought to keep up
with the improvement in the students’ language competence. During the time, the
following should be noted.


<i><b>Monitoring from the teacher</b></i>


The previously mentioned no-test rule does not mean that follow-up activities are
excluded from the project. On the contrary, they are crucial in the teacher’s effort
to monitor the students’ performance and progress.


Below are some highly recommended activities by Bramford (1998) and Day
(2002)


<i><b>The Reader Interview </b></i>


Regular conferencing between teacher and student on what they have read and on
their opinion on the book.


<i><b>Read Aloud to the Class/ Use of audio books </b></i>


The teacher may need to read some favourite extracts outloud to the class and then
share his/her thinking on the parts. This act provides a model of how to correctly


pronounce a word and how to read with emotion. Students then can follow. In
cases when the teacher is not comfortable with reading out loud, the use of an
audio book may help.


<i><b>Student Presentations </b></i>


Short presentations on books mean more opportunities to practise speaking skills
in a highly authentic context. IN similar projects, it has been observed that most of
the book choices made by students resulted from recommendations made by


</div>
<span class='text_page_counter'>(92)</span><div class='page_container' data-page=92>

<i><b>Follow-up written reports/ blurbs/ reviews </b></i>


Contests can be organised in which students choose on their own to write a report
on a book they have read or an alternative blurb to the one used by the publisher.
<b>Part 3. Conclusion</b>


The study seeks to provide some insights into extensive reading, focussing on
highlighting the benefit and relevance of extensive reading in our educational
setting. From this a plan has been suggested for the next academic year.


</div>
<span class='text_page_counter'>(93)</span><div class='page_container' data-page=93>

<b>REFERENCES</b>


[1]. Bell, T, (1998) Extensive Reading: Why? and How? The IESL Journal, 12 (4)
[2]. Day, Richard, R. (2002) ‘Top Ten Principles for teaching extensive reading.’


Reading in a Foreign Language. 14(2)


[3]. Day, Richard, R and Bamford, Julian.(1998) Extensive Reading in the
Second Language Classroom. Cambridge: Cambridge University Press.
[4]. Elley, W. B., & Manghubai, F. (1983). 'The effect of reading on second



language learning.' Reading Research Quarterly, 19/1, (pp 53-67).


</div>
<span class='text_page_counter'>(94)</span><div class='page_container' data-page=94>

<b>Appendix 1: Sample of a reading text. Genre: Literature. Grade 5</b>


<i><b>The Devoted Widow</b></i>


A widow weeping on her husband's grave was approached by an Engaging
Gentleman who, in a respectful manner, assured her that he had long entertained
for her the most tender feelings.


"Wretch!" cried the Widow. "Leave me this instant! Is this a time to talk to me
of love?"


"I assure you, madam, that I had not intended to disclose my affection," the
Engaging Gentleman humbly explained, "but the power of your beauty has
overcome my discretion."


"You should see me when I have not been crying," said the Widow.


</div>
<span class='text_page_counter'>(95)</span><div class='page_container' data-page=95>

<b>GIẢI BÀI TẬP CO2</b>


<b>Nguyễn Đắc Tứ - THPT Chuyên Vĩnh Phúc</b>
Bài tập của CO2 ở đây không chỉ là bài tập của CO2 mà là các dạng bài tập có cùng


cấu trúc với bài tập của CO2.


<b>Bài 1.</b>


Viết phương trình phản ứng xảy ra và nêu hiện tượng.


1. Cho từ từ CO2 vào dung dịch Ca(OH)2 cho tới dư


2. cho từ từ NaOH vào dung dịch AlCl3


3. cho từ từ HCl vào dung dịch NaAlO2


4. sục CO2 vào dung dịch NaAlO2


5. cho từ từ Na2CO3 vào dung dịch HCl


6. Cho SO2 vào dung dịch chứa hỗn hợp Ba(OH)2 và NaOH


<b>Bài giải</b>
1. Phương trình phản ứng:


CO2 + Ca(OH)2   CaCO3 + H2O


CO2 + CaCO3 + H2O   Ca(HCO3)2


Có kết tủa trắng CaCO3 sau đó kết tủa tan khi CO2 dư.


2. 3NaOH + AlCl3   3NaCl + Al(OH)3


NaOH + Al(OH)3   NaAlO2 + 2H2O


Có kết tủa trắng keo (Al(OH)3), kết tủa tan khi NaOH dư.


3. HCl + Na2CO3   NaCl + NaHCO3


HCl + NaHCO3   NaCl + CO2 + H2O



Giai đoạn đầu khơng thấy hiện tượng, sau đó có bọt khí thốt ra.
4. CO2 + NaAlO2 + 2H2O   NaHCO3 + Al(OH)3


</div>
<span class='text_page_counter'>(96)</span><div class='page_container' data-page=96>

5. 2HCl + Na2CO3   2NaCl + CO2 + H2O


Có bọt khí thốt ra.


6. SO2 + Ba(OH)2   BaSO3 + H2O


SO2 + 2NaOH   Na2SO3 + H2O


SO2 + Na2SO3 + H2O   2NaHSO3


SO2 + BaSO3 + H2O   Ba(HSO3)2


Có kết tủa trắng BaSO3 sau đó kết tủa không đổi, rồi kết tủa lại tan dần khi SO2 dư.


<b>Bài 2.</b>


Cho a mol CO2 vào dung dịch có chứa b mol NaOH thu được dung dịch X. Trong


X có chứa chất gì?


<b>Bài giải</b>
CO2 + NaOH   NaHCO3


CO2 + 2NaOH   Na2CO3 + H2O


Nếu đặt : 2



<i>NaOH</i>
<i>CO</i>


<i>n</i>


<i>t</i>


<i>n</i> 


Ta có:


1 2 t


NaHCO3 Na2CO3


NaHCO<sub>3</sub>


Na2CO3


Na<sub>2</sub>CO<sub>3</sub>


NaOH(du)


<b>Bài 3.</b>


Cho CO2 vào 500 ml dung dịch NaOH 1M, sau phản ứng cô cạn cẩn thận dung


dịch thu được 25,2 gam chất rắn khan.
1. Xác định thể tích khí CO2 đã dùng (đktc)



2. Tính CM các chất trong dung dịch sau phản ứng ( thể tích dung dịch khơng thay


đổi).


</div>
<span class='text_page_counter'>(97)</span><div class='page_container' data-page=97>

Ta có:


nNaOH = 0,5.1 = 0,5 (mol)


giả sử phản ứng CO2 với NaOH vừa đủ để tạo thành Na2CO3


CO2 + 2NaOH   Na2CO3 + H2O


=> 2 3


1


0, 25( )
2


<i>Na CO</i> <i>NaOH</i>


<i>n</i>  <i>n</i>  <i>mol</i>


=> m = 0,25.106 = 26,5 (gam) > 25,2 (gam)


=> NaOH dư hay trong dung dịch sau phản ứng có Na2CO3 và NaOH dư


Gọi x là số mol CO2



=> 106x + 40(0,5 – 2x) = 25,2
=> x = 0,2 (mol)


=> V (CO2) = 0,2.22,4 = 4,48 lít


2. CM(NaOH) = 0,2M và của Na2CO3 = 0,4M


<b>Bài tập tương tự</b>
<b>Bài 4.</b>


Cho CO2 vào 700 ml dung dịch NaOH 0,5M sau phản ứng cô cạn cẩn thận dung


dịch thu được 23,2 gam chất rắn khan.
1. Tính thể tích CO2 (đktc) đã dùng


2. Tính CM các chất trong dung dịch sau phản ứng. (Coi V dung dịch không đổi)


<b>Bài 5.</b>


Cho 19,2 gam kim loại M tan hoàn toàn vào dung dịch H2SO4 đặc, nóng dư, lấy


tồn bộ lượng khí SO2 sinh ra cho vào 700 ml dung dịch NaOH 1M. Sau phản ứng


cô cạn cẩn thận dung dịch thu được 41,8 gam chất rắn khan. Xác định kim loại M.
<b>Bài 6.</b>


Cho CO2 vào dung dịch có a mol Ca(OH)2 thu được kết tủa. Vẽ đồ thị biểu diễn sự


phụ thuộc của số mol kết tủa vào số mol CO2.



<b>Bài giải</b>


</div>
<span class='text_page_counter'>(98)</span><div class='page_container' data-page=98>

CO2 + CaCO3 + H2O   Ca(HCO3)2 (2)


TH1: nếu Ca(OH)2 dư hay <i>nCO</i>2 <i>a</i>


(1) => <i>n</i> <i>nCO</i>2


TH2: nếu xảy ra cả hai phản ứng hay <i>a n</i> <i>CO</i>2 2<i>a</i>


(1), (2) => <i>n</i> 2<i>a n</i> <i>CO</i>2


TH3: nếu <i>nCO</i>2 2<i>a</i>


 khơng có kết tủa


ta có đồ thị:


a


a 2a


nCO2
CaCO3


<b>Bài tập Vận dụng</b>
<b>Bài 7.</b>


Hịa tan 11,2 gam CaO vào H2O được dung dịch A.



1. Cho CO2 vào dung dịch A thu được 2,5 gam kết tủa. Xác định thể tích CO2


(đktc) đã dùng?


2. Cho 28,1 gam hỗn hợp B gồm MgCO3 và BaCO3 a% theo khối lượng tác dụng


với dung dịch HCl dư, thu tồn bộ khí CO2 vào dung dịch A thu được kết tủa. Xác


định a để


a) kết tủa thu được là lớn nhất
b) Kết tủa thu được là nhỏ nhất.
<b>Bài 8.</b>


Cho 7,2 gam hỗn hợp hai muối cácbonat của hai kim loại thuộc phân nhóm chính
nhóm II và thuộc hai chu kì kế tiếp phản ứng hồn tồn với dung dịch H2SO4


lỗng, dư, tồn bộ lượng khí thốt ra cho phản ứng hoàn toàn với 450 ml dung dịch
Ba(OH)2 0,2M. Sau phản ứng thu được 15,76 gam kết tủa.


</div>
<span class='text_page_counter'>(99)</span><div class='page_container' data-page=99>

<b>Bài 9.</b>


Sục từ từ CO2 vào dung dịch chứa a mol NaOH và b mol Ba(OH)2 thu được kết


tủa. Vẽ đồ thị biểu diễn sự phụ thuộc của số mol kết tủa vào số mol CO2


<b>Bài giải</b>


CO2 + Ba(OH)2   BaCO3 + H2O (1)



CO2 + 2NaOH   Na2CO3 + H2O (2)


CO2 + Na2SO3 + H2O   2NaHCO3 (3)


CO2 + BaSO3 + H2O   Ba(HCO3)2 (4)


TH1: <i>nCO</i>2 <i>b</i>


(1) => <i>n</i> <i>nCO</i>2


TH2: <i>b n</i> <i>CO</i>2  <i>a b</i>


(1),(2), (3) => <i>n</i> <i>nBa OH</i>( )2


TH3: <i>a b n</i>  <i>CO</i>2  <i>a</i> 2<i>b</i>


=> <i>n</i> 2<i>b a n</i>  <i>CO</i>2


TH4: <i>nCO</i>2  <i>a</i> 2<i>b</i>


 khơng có kết tủa
 đồ thị:


nCO2


BaCO3


<b>Bài tập vận dụng</b>
<b>Bài 10.</b>



Cho V lít CO2 (đktc) vào dung dịch hỗn hợp NaOH 0,15 (mol), Ca(OH)2 0,1 (mol)


</div>
<span class='text_page_counter'>(100)</span><div class='page_container' data-page=100>

1. V = 1,972 lít
2. 3,36 lít
3. 6,72 lít
<b>Bài 11.</b>


Cho CO2 vào dung dịch A chứa 0,1 mol NaOH và 0,15 mol Ba(OH)2 , sau phản


ứng thu được m gam kết tủa. Xác định thể tích CO2 (đktc) đã dùng trong các


trường hợp sau
1. m = 19,7
2. m = 29,55
<b>Bài 12.</b>


Hịa tan hồn tồn 16 gam hỗn hợp A gồm Na và Ba vào H2O thu được dung dịch


B và khối lượng dung dịch tăng 15,7 gam.
1. Xác định % khối lượng của hỗn hợp đầu?


2. Cho CO2 vào dung dịch B thu được 17,73 gam kết tủa. Xác định số mol CO2 đã


dùng


3. Nếu lấy lượng CO2 ở trên cho phản ứng với 1 lít dung dịch Ca(OH)2 1,5M thì


lượng kết tủa thu được là bao nhiêu?
<b>Bài 13.</b>



Dung dịch X chứa a mol HCl, dung dịch Y chứa b mol Na2CO3.


1. Cho từ từ X vào Y thu được dung dịch Z. Xác định các chất trong Z và số mol
mỗi chất theo a, b


2. Cho từ từ Y vào X thu được dung dịch T. Xác định các chất trong T và số mol
mỗi chất theo a, b


<b>Bài giải</b>
1) Cho từ từ HCl vào Na2CO3


HCl + Na2CO3   NaCl + NaHCO3


HCl + NaHCO3   NaCl + CO2 + H2O


TH1: a <b


</div>
<span class='text_page_counter'>(101)</span><div class='page_container' data-page=101>

TH2: a = b


 trong dung dịch có NaCl, NaHCO3


TH3: b< a< 2b


 trong dung dịch có NaCl, NaHCO3


TH4: a = 2b


 trong dung dịch có NaCl


TH5: a > 2b



 trong dung dịch có NaCl, HCl


2) Cho từ từ Na2CO3 vào HCl


Na2CO3 + 2HCl   2NaCl + CO2 + H2O


TH1: a < 2


<i>b</i>


 trong dung dịch có NaCl, HCl


TH2: a = 2


<i>b</i>


 trong dung dịch có NaCl


TH3: a > 2


<i>b</i>


 trong dung dịch có NaCl, Na2CO3


<b>Bài tập vận dụng:</b>
<b>Bài 14.</b>


Cho dung dịch A chứa 0,3 mol HCl
Dung dịch B chứa 0,2 mol Na2CO3



1. Cho từ từ A vào B thu được V1 lít khí (đktc)


2. Cho từ từ B vào A thu được V2 lít khí (đktc)


3. Đổ A vào B thu được V3 lít khí (đktc)


Xác định V1, V2, V3


<b>Bài 15.</b>


Cho từ từ 1 mol HCl vào 0,8 lít dung dịch A chứa Na2CO3 1M và NaHCO3 0,5M. Tính thể tích khí thu


được (đktc).


<b>Bài 16.</b>


</div>
<span class='text_page_counter'>(102)</span><div class='page_container' data-page=102>

Cho từ từ HCl 0,1M vào dung dịch A đến khi bắt đầu có khí thốt ra thì dùng hết 0,8 lít dung dịch HCl
0,1M.


Nếu cho Ba(OH)2 dư vào A thu được 23,64 gam kết tủa.


Xác định số mol mỗi chất trong A.


<b>Bài 17.</b>


Dung dịch A chứa NaHCO3 0,5M và Na2CO3 0,6M. Dung dịch B chứa K2CO3 0,5M và KHCO3 0,4M


Trộn 200 ml dung dịch A với 300 ml dung dịch B thu được dung dịch D.
1. Cho từ từ 0,4 mol HCl vào dung dịch D. Tính thể tích khí thu được (đktc).


2. Nếu cho BaCl2 dư vào dung dịch D thì thu được bao nhiêu gam kết tủa.


3. Nếu cho Ca(OH)2 dư vào dung dịch D thì thu được bao nhiêu gam kết tủa.


<b>Bài 18.</b>


có hai dung dịch A và B đựng trong hai ống nghiệm mất nhãn chứa HCl và Na2CO3. Người ta tiến hành


các thí nghiệm sau


Nếu cho từ từ A vào B, sau phản ứng thu được 1,12 lít khí (đktc)
Nếu cho từ từ B vào A , sau phản ứng thu được 3,36 lít khí (đktc)
1. Xác định xem dung dịch A, B là dung dịch nào?


2. Tính số mol mỗi chất trong A, B.


<b>Bài 19.</b>


Cho CO2 vào 500 ml dung dịch NaOH thu được dung dịch A.


Cho từ từ HCl vào dung dịch A đến khi bắt đầu có khí thấy cần dùng hết 500 ml dung dịch HCl 0,2M.
Nếu cho Ba(OH)2 dư vào dung dịch A thu được 11,82 gam kết tủa.


Xác định thể tích CO2 (đktc) đã dùng và nồng độ mol/l của dung dịch NaOH ban đầu.


<b>Bài 20.</b>


Cho CO2 vào 500 ml dung dịch NaOH thu được dung dịch A.


Cho từ từ HCl vào dung dịch A đến khi bắt đầu có khí thấy cần dùng hết 500 ml dung dịch HCl 0,2M.


Nếu cho Ba(OH)2 dư vào dung dịch A thu được 29,55 gam kết tủa.


</div>
<span class='text_page_counter'>(103)</span><div class='page_container' data-page=103>

<b>PHƯƠNG PHÁP GIẢI BÀI TOÁN DỊNG ĐIỆN KHƠNG ĐỔI</b>


<b>Đồ Văn T́n – THPT Chun Vĩnh Phúc</b>
<b>Phương pháp giải toán</b>


Khi gặp các bài toán về dịng điện khơng đổi chúng ta có thể sử dụng nhiều
phương pháp để làm trong đó hay sử dụng nhất là dùng định luật Ơm cho tồn
mạch và cho từng loại đoạn mạch, ngồi ra chúng ta cũng có thể sử dụng định luật
Kiêsxốp, phương pháp nguồn tương đương hoặc phương pháp chồng chất. Trong
bài này tôi xin lấy một vài ví dụ để sử dụng các phương pháp này:


<b>VD 1: Cho mạch điện như hình vẽ: R</b>1 = 2


Ω, R2 = 1 Ω, R3 = 3 Ω, E = 6 V, r = 1 Ω. RA


= 0.


a. Cho R4 = 1 Ω. Tìm số chỉ của ampe kế.


b. Biết am pe kế chỉ 0,5 A và cực dương
của nó mắc vào điểm C. Tìm giá trị của R4.


<i><b>Lời giải:</b></i>


<i><b>a. Đây là một bài tốn tḥn nên ta có thể</b></i>


<i>sử dụng định luật Ôm cho toàn mạch để</i>



<i>làm (</i>


E
I


r R


 ).


- Do ampe kế có điện trở khơng đáng kể nên điểm C và D chập lại với nhau  cấu
tạo của mạch ngoài là: (R1//R2) nt (R3//R4)


 điện trở mạch ngoài:


A


R1 R3


R2 R4


A B


</div>
<span class='text_page_counter'>(104)</span><div class='page_container' data-page=104>

R =


3 4
1 2


1 2 3 4



R R


R R 2.1 3.1


R R R R 2 1 3 1   
17
12 <sub>Ω</sub><sub>.</sub>


 cường độ dịng điện mạch chính là (tính theo định luật Ơm cho tồn mạch):
I =


E
r R =


72


29<sub> A</sub>


Suy ra cường độ dòng điện qua R1 và R3 lần lượt là (đoạn mạch mắc song song thì


cường độ dịng điện tỷ lệ nghịch với điện trở):


2
1
1 2
I.R
I
R R

 <sub> = </sub>


24
29<sub> A </sub>


4
3
3 4
I.R
I
R R

 <sub> = </sub>
18
29<sub> A </sub>


Ta thấy I1 > I3 nên dòng điện qua ampe kế có chiều chạy từ D đến C và số chỉ của


nó là:


IA = I1 - I3 =


6


29<sub> A </sub> 0,2 A.


<i><b>b. Đây là một bài tốn ngược nên có hai cách để làm:</b></i>
<i><b>Cách 1: Dùng định luật Ôm cho toàn mạch:</b></i>


Tương tự như phần a nhưng ta đặt R4 = x.


 R =




2.1 3.x 6 11x


2 1 3 x 3 3 x




 


  


Cường độ dịng điện mạch chính: I =





18 3 x


E 6


6 11x


r R <sub>1</sub> 15 14x


3 3 x



 

 




Cường độ dòng điện qua R1 và R3 là:




2
1


1 2


6 3 x


I.R I


I


R R 3 15 14x




  


</div>
<span class='text_page_counter'>(105)</span><div class='page_container' data-page=105>



4
3


3 4



18 3 x


I.R I.x x 18x


I


R R 3 x 15 14x 3 x 15 14x




   


    


Theo giả thiết cực dương của ampe kế mắc vào C nên: I3 = I1 + IA


Hay:


18x
15 14x <sub>= </sub>




6 3 x
15 14x




 <sub>+ 0,5  x = 5,1 </sub><sub>Ω</sub><sub>.</sub>



<i><b>Cách 2: Dùng định luật Ôm cho các loại đoạn mạch (I</b></i>MN<b> = </b>


MN
MN
U E
R

<b>):</b>
Tại các nút C và D ta có:


I3 = I1 + IA = I1 + 0,5 (1)


I2 = IA + I4  I4 = I2 – 0,5 (2)


Ta sẽ thiết lập hệ hai phương trình với hai ẩn là I1 và I3.


Cường độ dịng điện qua R1 là:


2
1


1 2


I.R I


I


R R 3


 



 <sub> dịng điện mạch chính I = 3I</sub>


1.


Hiệu điện thế hai đầu nguồn (định luật Ôm cho đoạn mạch chứa nguồn):
UAB = E – Ir = 6 – I.1 = 6 – 3I1 (3)


Mặt khác: UAB = UAD + UDB = I1.R1 + I3.R3 = 2I1 + 3I3 (4).


So sánh (3) và (4) ta được: 6 = 5I1 + 3I3.


Kết hợp với (1) ta được một hệ hai phương trình với hai ẩn là I1 và I3. Giải hai


phương trình này ta được:


I1 =


9


16<sub> A và I</sub><sub>3</sub><sub> = </sub>
17
16<sub> A </sub>


Mặt khác: I2 = I – I1 = 2I1 =


9


8<sub> A . Thay vào (2) ta có I</sub><sub>4</sub><sub> = </sub>
5


8<sub> A </sub>


Ngồi ra UCB = UDB = I3.R3 =


51
16<sub> V  </sub>


CB
4
4
U
R
I


</div>
<span class='text_page_counter'>(106)</span><div class='page_container' data-page=106>

<b>Nhận xét: Trong hai cách làm này thì cách 1 là</b>
các làm tổng quát có thể áp dụng cho tất cả các
bài và trong cách này ta chỉ có một phương trình
với một ẩn. Tuy nhiên cách này thường dài và rất
hay bị nhầm lẫn. Đối với cách 2 có thể có 2 hay
nhiều ẩn hơn nhưng đổi lại các phương trình
chứa ẩn này thường dễ giải và ít bị nhầm lẫn.


Tuy nhiên cách này đòi hỏi học sinh phải tư duy để chọn ra các ẩn cho phù hợp.
<b>VD 2: Cho mạch điện như hình vẽ: E</b>1 = 6V, r1 = 1Ω, E2 = 3V, r2 = 3Ω.


a. Cho R = 3Ω. Tìm cường độ dịng điện qua R.
b. Tìm R để công suất tiêu thụ trên R cực đại.
<i><b>Lời giải:</b></i>



<i><b>a. Trong phần này có rất nhiều cách để làm:</b></i>
<i><b>- Cách 1: Dùng định luật Ôm cho đoạn mạch: </b></i>


Giả sử chiều của các cường độ dòng điện qua các nguồn và qua điện trở R như
hình vẽ.


Áp dụng định luật Ơm cho đoạn mạch AB có chứa nguồn E1.


BA 1 BA


1


1


U E U 6


I


r 1


 


 


;


Đoạn mạch AB chứa nguồn E2.


BA 2 BA



2


2


U E U 3


I


r 3


 


 


;


Đoạn mạch AB chứa điện trở R:


AB AB
U U
I
R 3
 
;


Mặt khác tại nút A ta có: I = I1 + I2 


AB BA BA


U U 6 U 3



3 1 3


 


 


 UAB = 4,2 V  I = 1,4 A.


</div>
<span class='text_page_counter'>(107)</span><div class='page_container' data-page=107>

<i><b>- Cách 2: Dùng định luật Kiêtsxốp: (Trong một mạch điện kín có n dịng điện,</b></i>


<i>trong đó có x nút và y mắt mạng. Khi đó ta sẽ xây dựng x – 1 phương trình nút (</i>


vào ra


I  I


<i><sub>) và n – x + 1 phương trình mặt mạng (</sub></i>

Ei 

I Ri i <i>với E</i>


<i>i > 0 nếu</i>


<i>chiều dương chọn đi vào cực âm và đi ra cực dương của nguồn, Ii > 0 nếu dòng</i>


<i><b>điện cùng chiều dương)).</b></i>


Trong bài này ta có 1 phương trình nút: I = I1 + I2 (1).


Và hai phương trình cho mắt mạng (giả sử chiều dương được chọn ngược chiều
kim đồng hồ):



Mắt mạng chứa E1 và R: E1 = I1.r1 + I.R hay 6 = I1 + 3.I (2).


Mắt mạng chứa E2 và R: E2 = I2.r2 + I.R hay 3 = 3.I2 + 3.I (3).


Từ (1), (2), (3) ta được I = 1,4 A.


<i><b>- Cách 3: Phương pháp chồng chất (I</b></i>R = IRchỉ do E1 gây ra + I<b>Rchỉ do E2 gây ra )</b>


+ Để tính cường độ dịng điện qua R chỉ do E1 gây ra ta cho E2 bằng 0. Khi đó


mạch điện gồm một nguồn E1, r1 nối với mạch ngoài là r2 // R.


 điện trở mạch ngoài: R1 =


2
2


r R


r R<sub> = 1,5 </sub><sub>Ω</sub><sub>  cường độ dịng điện mạch ngồi: I</sub>


n1 =


1


1 1


E


r R <sub>= 2,4 A</sub>



 cường độ dòng điện qua R là: IR1 =


n1 2
2


I r


r R<sub>= 1,2 A.</sub>


+ Tương tự để tính cường độ dịng điện qua R chỉ do E2 gây ra ta cho E1 bằng 0.


Khi đó mạch điện gồm một nguồn E2, r2 nối với mạch ngồi là r1 // R. Tính tốn ta


được: IR2 = 0,2 A


+ Vậy cường độ dòng điện tổng cộng qua R là (chú ý chiều của các dòng IR1 và IR2


</div>
<span class='text_page_counter'>(108)</span><div class='page_container' data-page=108>

<i><b>- Cách 4: Phương pháp thay thế: (ta thay thế toàn bộ mạch điện không chứa điện</b></i>


<i>trở R thành một nguồn điện duy nhất E, r, trong bài này thì phép thay thế là phép</i>
<i>xác định nguồn tương đương của hai nguồn mắc song song nhưng không giống</i>


<i><b>nhau: </b></i> td 1 2


1 1 1


r  r r <b><sub>; </sub></b>


td 1 2



td 1 2


E E E


r r  r <b><sub>). </sub></b>


Dùng hai công thức trên ta dễ dàng tìm được: rtd = 0,75 Ω, Etd =


21


4 <sub>V.</sub>


 cường độ dòng điện qua R là: IR =


td
td


E


R r <sub> = 1,4 A.</sub>


<i><b>b. Để làm phần b ta nên sử dụng phương pháp thay thế (cách 4). </b></i>


Dễ dàng suy ra để công suất trên R cực đại thì R = rtd = 0,75 Ω và Pmax =


2
td


td



E
4r <sub> = </sub>


147
16


 9,2 W.


<b>Bài tập tự luyện</b>


<b>Bài tập 1: Cho mạch điện như hình vẽ. Nguồn điện có E = 12 V, r = 2 </b>Ω. R1 = 2 Ω,


R3 = 3 Ω, RA = 0. Biết rằng khi K đóng và khi K mở số chỉ của ampe kế là 2 A và 3


</div>
<span class='text_page_counter'>(109)</span><div class='page_container' data-page=109>

<b>Bài tập 2: Cho mạch điện như hình vẽ. ξ</b>1 = 4,5V; r1 = 1Ω, ξ2 = 1,8V, RAB = 8Ω,


RA = 0, R = 3Ω. Tìm giá trị của điện trở đoạn


AC để ampe kế chỉ số không:


<b>Bài tập 3: Cho mạch điện như hình vẽ. </b>
E1 = 6 V, r1 = 1 Ω, E2 = 3 V, r2 = 2 Ω,


R1 = R3 = 3 Ω, RV = .


a. Cho R2 = 2 Ω. Tìm số chỉ của vơn kế.


b. Tìm R2 để UCD = 4 V.



c. Tìm R2 để vơn kế chỉ 0 V.


d. Tìm R2 để vôn kế chỉ 1 V.


R
1


R
2
R
3
R
4


A
E,


r


K


R
A
ξ1 , r1


ξ2 , r2


A C B


V


E1,
r1


E2,
r2


R1 R2


R3


A B


C


</div>
<span class='text_page_counter'>(110)</span><div class='page_container' data-page=110>

<b>RÈN LUYỆN KĨ NĂNG LÀM KIỂU BÀI NGHỊ LUẬN XÃ HỘI CHO HỌC</b>
<b>SINH GIỎI MÔN NGỮ VĂN THCS</b>


<b>Hoàng Văn Quyết – THPT Chuyên Vĩnh Phúc</b>


<b>A. ĐẶT VẤN ĐỀ</b>


Định hướng chung, cái đích của đổi mới phương pháp dạy học hiện nay là phát
huy được tính chủ động, tích cực, sáng tạo của học sinh trong quá trình lĩnh hội kiến
thức, rèn luyện tồn diện về tư duy, tình cảm, tâm hồn. Điều này đồng nghĩa với việc
học sinh phải được coi là ngọn lửa cần thắp sáng chứ không phải là cái bình chứa kiến
thức. Những năm gần đây, nhằm đáp ứng yêu cầu đổi mới giáo dục và phát triển toàn
diện cho học sinh về kiến thức cũng như kĩ năng, đề thi mơn văn có rât nhiều đổi mới.
Đề bài thường có 2 phần: Nghị luận văn học và nghị luận xã hội. Đây là khó khăn cho
học sinh vì nghị luận xã hội địi hỏi lối viết sắc sảo, tư duy xã hội sâu rộng, thiên về lý
trí còn nghị luận văn học đòi hỏi vốn văn học chắc chắn, lối viết thiên về cảm xúc.


Học sinh sẽ phải tự dung hòa hai cách viết, hai lối tư duy khác nhau trong cùng một
bài viết.


</div>
<span class='text_page_counter'>(111)</span><div class='page_container' data-page=111>

Qua thực tế giảng dạy, chúng tơi nhận thấy có một hạn chế phổ biến ở học
sinh hiện nay là bài văn nghị luận thiếu chất văn. Bài văn nghị luận có thể mạch
lạc, rõ ràng, đáp ứng yêu cầu về nội dung tư tưởng nhưng nhạt về tình cảm, nghèo
ngôn ngữ, thiếu tâm huyết hoặc vụng về trong diễn đạt, nghĩa là chất văn còn hạn
chế. Bởi vậy, RÈN LUYỆN KĨ NĂNG LÀM KIỂU BÀI NGHỊ LUẬN XÃ HỘI CHO HỌC
<b>SINH GIỎI MÔN NGỮ VĂN THCS rất cần thiết, là định hướng đúng đắn cho học sinh.</b>


<i><b>Phần nội dung chuyên đề có cấu trúc như sau:</b></i>
I. Nhận diện đặc điểm của văn nghị luận xã hội.


<b>II. Kĩ năng làm kiểu bài nghị luận xã hội đúng, hay và giàu chất văn.</b>
III. Hướng dẫn học sinh ôn luyện kiểu bài NLXH.


<b>B. PHẦN NỘI DUNG</b>


<b>I. NHẬN DIỆN ĐẶC ĐIỂM KIỂU BÀI NGHỊ LUẬN XÃ HỘI.</b>


<b>1. Khái niệm.</b>


<b>Nghị luận : nghị (xem xét, trao đổi; luận: bàn bạc, đánh giá) dùng lý lẽ, dẫn</b>
chứng và cách thức lập luận để phân tích, bàn luận, đánh giá về một (các) vấn đề
nào đó.


<b>Xa hợi : các vấn đề của đời sống con người (triết học, lịch sử, kinh tế, đạo</b>
<b>đức, văn học nghệ thuật, lối sống, cách ứng xử…). </b>


<b>Nghị luận xa hội là những bài văn nghị luận bàn về các vấn đề xã hội (thực</b>


trạng xã hội, các hiện tượng đời sống, vấn đề về lối sống của con người, các mối
quan hệ của con người trong xã hội…) nhằm thể hiện suy nghĩ, thái độ, tiếng nói
chủ quan của người viết về vấn đề đặt ra, góp phần tạo những tác động tích cực tới
con người, bồi đắp những giá trị nhân văn và thúc đẩy sự tiến bộ chung của xã hội.
<b>2. Các kiểu bài nghị luận xa hội. </b>


</div>
<span class='text_page_counter'>(112)</span><div class='page_container' data-page=112>

Dạng đề nghị luận về một tư tưởng, đạo lí thường nhân một câu danh ngơn,
một nhận định, đánh giá nào đó để yêu cầu người viết bàn luận và thể hiện tư
tưởng, quan điểm, thái độ của mình.


Dạng đề nghị luận về một hiện tượng đời sống thường nêu lên một hiện
tượng, một vấn đề có tính thời sự, được dư luận xã hội trong nước cũng như cộng
đồng quốc tế quan tâm.


Dạng đề nghị luận về một vấn đề xã hội đặt ra trong tác phẩm văn học kết
hợp kiểm tra được cả về năng lực đọc – hiểu tác phẩm văn học, cả về kiến thức xã
hội và khả năng nghị luận với hai hình thức sau: Từ một tác phẩm đã học, đề yêu
cầu người viết bàn về một ý nghĩa xã hội nào đó. Cũng có thể từ một tác phẩm
chưa được học, thường là câu chuyện nhỏ (truyện mi ni), đề yêu cầu bàn về ý
nghĩa xã hội đặt ra trong đó.


<b>3. Yêu cầu của bài nghị luận xa hội.</b>


<b>* Đảm bảo kiến thức mang màu sắc chính trị - xa hợi:</b>
<b>- Những hiểu biết về chính trị, pháp luật;</b>


<b>- Kiến thức về lịch sử, văn hóa, đạo đức, tâm lý xã hội…</b>
<b>- Những tin tức thời sự cập nhật.</b>


<b>* Đảm bảo kĩ năng nghị luận:</b>



<b>- Tập trung hướng tới luận đề để bài viết không tản mạn.</b>
<b>- Có ý thức triển khai thành các </b>lu ậ n đi ể m chặt chẽ.
<b>- Dẫn chứng xác đáng, tiêu biểu.</b>


<b>-</b> L ậ p lu ậ n chặt chẽ, thuyết phục.


<b>-</b> Ngôn ng ữ : trong sáng, có màu sắc luận lí vừa có sắc thái mĩ cảm.


</div>
<span class='text_page_counter'>(113)</span><div class='page_container' data-page=113>

<b>II. KĨ NĂNG LÀM KIỂU BÀI NGHỊ LUẬN XÃ HỘI ĐÚNG, HAY VÀ GIÀU CHẤT VĂN.</b>


<b>1. Những kinh nghiệm cần có để viết được bài văn nghị luận xa hội đúng, hay</b>
<b>và giàu chất văn.</b>


<i><b>-</b></i> <i>Người viết cần xác định được yêu cầu của đề, nắm bắt tinh thần của đề bài.</i>


Phải xác định trúng, nắm bắt chính xác u cầu của đề bài thì người viết mới
có thể có được định hướng suy nghĩ đúng đắn, khoanh vùng kiến thức, phạm vi
dẫn chứng để biện giải vấn đề.


<b>-</b> Để bài viết có những tác động tích cực tới nhận thức và tình cảm con người,
<i>người viết cần xác định cho mình lập trường và thái độ đúng đắn trên cơ sở</i>
những hiểu biết về chuẩn mực đạo đức cũng như các chuẩn đánh giá chung của
xã hội. Có vậy sự biện luận mới đúng, sắc và thuyết phục người đọc.


<b>-</b> Tuy nhiên, người viết phải thể hiện được cái nhìn, đánh giá của riêng mình
về cuộc đời, con người, về mục đích, lối sống… Những điều đó khơng có trong
sách vở mà cần sự trải nghiệm của chính chủ thể.


<b>-</b> Phải thể hiện được thái độ, tình cảm, nhiệt tình của người viết. Những cảm


xúc chân thành chính là rung động của tâm hồn khi chạm vào cuộc sống, khiến
bài văn không phải là bài thuyết giáo cho một tư tưởng đạo lí khơ khan mà bài
viết là sự chia sẻ chân thành của người viết về những gì mình trải qua, mình
chiêm nghiệm. Người viết cần tạo cho mình tâm thế của người trong cuộc, đặt
mình trong hồn cảnh, tình huống của vấn đề. Khi đó, người viết sẽ có những
suy ngẫm, đánh giá bằng chính những trải nghiệm của bản thân, điều này sẽ chi
phối thái độ, cảm xúc, suy nghĩ, giọng điệu của người viết. Đọc những bài văn
này, người đọc sẽ có cảm giác như đang được đối thoại trực tiếp với người viết,
chất sống, “chất xã hội” sẽ hiện lên một cách tự nhiên mà sống động. Tuy nhiên,
nếu chỉ đặt mình là người trong cuộc, sử dụng điểm nhìn từ bên trong thì
những suy ngẫm mang tính chủ quan, những đánh giá dễ mang tính cực đoan,
một chiều, hoặc là ngợi ca đề cao quá mức, hoặc là phê phán lên án quá độ.


</div>
<span class='text_page_counter'>(114)</span><div class='page_container' data-page=114>

<i>cần xác định cho mình điểm nhìn khách quan, xem xét vấn đề từ nhiều phương</i>
<i>diện, khía cạnh khác nhau. Khi đó, bài văn nghị luận xã hội dễ tìm được sự</i>


đồng cảm, đồng tình ở người đọc, thuyết phục người đọc.


<b>-</b> Để giúp người đọc hiểu vấn đề, thuyết phục người đọc đồng tình với quan
điểm của người viết, bài văn nghị luận xã hội không chỉ cần luận điểm mạch
lạc, lập luận chặt chẽ, lí lẽ sắc bén, dẫn chứng tiêu biểu, phù hợp mà còn cần có
<i><b>chất văn hấp dẫn về hình thức diễn đạt. </b></i>


+ Sử dụng linh hoạt các kiểu câu; phong phú, chính xác, tinh tế, độc đáo ngôn ngữ;
viết lời dẫn, lời chuyển ý sao cho khéo léo, uyển chuyển, mượt mà… và viết văn
<i>có hình ảnh để nâng cao chất văn cho bài viết.</i>


+ Sau bước giải thích từ ngữ quan trọng, nên lấy từ 2 – 3 ví dụ cụ thể hoá khái
niệm (nghĩa của từ ngữ quan trọng). Trên thực tế, bước này đã khơi dậy
được không chỉ tâm hồn, cảm xúc mà còn cả lối hành văn rất hình ảnh. Có


<i>thể lấy ví dụ cụ thể hố nghĩa của từ hưởng thụ: Buổi sáng thức giấc, bạn</i>


<i>bật tung cửa sổ để đón nhận khí trời và nắng mai. Đó là hưởng thụ, bạn</i>
<i>đang hưởng thụ từ Mẹ Trái Đất. Hay lấy hình ảnh gà mái và con suối nhỏ</i>


<i>trong cuốn Đaghextan của tơi – Raxun Gamzatốp làm ví dụ cụ thể hố cho</i>
<i>việc con người ta khơng tự biết mình là ai: gà mái mơ thấy mình là chim</i>


<i>ưng, nó bay khỏi vách đá và ngã gãy cánh. Con suối nhỏ mơ thấy mình là</i>
<i>dịng sơng lớn, nó tràn vào bãi cát và lập tức bị hút khô...; hoặc so sánh</i>
<i><b>người lạc quan với kẻ bi quan: nếu người lạc quan nói sẽ có ánh sáng ở</b></i>


</div>
<span class='text_page_counter'>(115)</span><div class='page_container' data-page=115>

chỉ đem lại cảm giác cân đối, mạch lạc, mà nó cũng nói nên rằng người
viết thực sự có trường độ tư duy (biết nhìn vấn đề ở nhiều mức, nhiều cấp,
nhiều mặt ngay trong một đơn vị rất ngắn là câu….)


+ Nên viết các kiểu câu có nội dung hai hoặc ba vế vừa phát triển vừa như
đối nghịch nhau để gây ấn tượng (Tạm gọi đó là những câu chứa nghịch
lí). Ví dụ “Cuộc sống hiện đại của chúng ta đang nảy sinh quá nhiều
<i>nghịch lí. Chúng ta đã xây được nhiều nhà to hơn, vững chãi hơn, nhưng</i>
<i>gia đình thì nhỏ lại, hạnh phúc gia đình thì mong manh hơn. Chúng ta tạo</i>
<i>ra nhiều máy tính để có được nhiều thơng tin, nhiều kết nối, nhiều bản sao</i>
<i>hơn, nhưng lại càng ít đi những giao tiếp giữa người với người. Chúng ta</i>
<i>có thể bay lên mặt trăng rồi quay về trái đất, nhưng chúng ta lại ngại rẽ</i>
<i>qua con phố để sang nhà hàng xóm. Nhiều khi chúng ta khơng chỉ khổ vì</i>
<i>nghèo mà cịn khổ vì q giàu có. Đa số những vấn đề chưa giải quyết</i>
<i>được của nhân loại ngày nay lại không phải do khách quan tự nhiên đem</i>
<i>lại mà do chính chúng ta gieo ra…”. Kiểu câu như vậy cho thấy rõ nhất</i>
<i>một cái nhìn có tính chất phát hiện đời sống của người viết. </i>



+ Dùng liên tiếp các câu có chung một kiểu cấu trúc ngữ pháp, thậm chí có
chung chủ ngữ để tạo sự trùng điệp - biện pháp lặp cấu trúc, góp phần nhấn
mạnh ý. Cách diễn đạt này cũng sẽ rất thuyết phục nếu chúng ta biết kết
hợp với cách nói lớp lang.


<i>Trong bài viết “Làm thế nào để biết được giá trị của thời gian?”, tác giả của</i>
bài viết đã vận dụng thành công lối diễn đạt này:


<i>-</i> <i>Muốn biết giá trị thật sự của một năm, hãy hỏi một học sinh thi rớt đại</i>
<i>học</i>


</div>
<span class='text_page_counter'>(116)</span><div class='page_container' data-page=116>

<i>-</i> <i>Muốn biết giá trị thật sự của một tuần, hãy hỏi biên tập viên của một</i>
<i>tạp chí ra hàng tuần</i>


<i>-</i> <i>Muốn biết giá trị thật sự của một giờ, hãy hỏi những người đang yêu</i>
<i>chờ đợi để được gặp nhau</i>


<i>-</i> <i>Muốn biết giá trị thật sự của một phút, hãy hỏi người vừa nhỡ chuyến</i>
<i>tàu</i>


<i>-</i> <i>Muốn biết giá trị thật sự của một giây, hãy hỏi người vừa thoát khỏi tai</i>
<i>nạn hiểm nghèo</i>


<i>-</i> <i>Muốn biết giá trị thật sự của một phần trăm giây, hãy hỏi người vừa</i>
<i>đoạt huy chương bạc Olympics</i>


<i>-</i> <i>…..</i>


<i>-</i> <i>Một giây là thời gian, mà thời gian là vịng xốy bất tận, một giây của</i>
<i>hôm nay không như một giây của hôm qua và càng không giống một giây</i>


<i>của ngày mai. Hãy sống để không bao giờ hối tiếc dù chỉ một giây ngắn</i>
<i>ngủi. Có thể chỉ một giây sẽ thay đổi cuộc đời con người …</i>


Cách diễn đạt này vừa xoáy sâu được ý muốn làm nổi bật, vừa thể hiện
được kiến thức phong phú của người viết, tạo ra được nét đặc biệt trong một đoạn
văn bản. Vừa nghị luận một cách tập trung vừa tạo ra điểm mới trong diễn đạt,
khiến người đọc không thể bỏ qua.


</div>
<span class='text_page_counter'>(117)</span><div class='page_container' data-page=117>

<i>ngắn khác nhau, sự phối hợp các âm “Bất kì đàn ơng, đàn bà, bất kì người già,</i>
<i>người trẻ…”. (Hồ Chí Minh)</i>


<b>2. Cách viết mở bài, kết bài trong văn nghị luận xa hội.</b>
<b>2.1. Mở bài </b>


<i><b>*Thế nào là một mở bài hay ?</b></i>


- Là mở bài đúng : có phần dẫn dắt vào đề và nêu vấn đề nghị luận.
- Là mở bài ngắn gọn, độc đáo, ấn tượng, sáng tạo.


<i><b>* Một số “mẹo” mở bài hay :</b></i>


- Nhập đề bằng câu chuyện ngắn trong thực tiễn đời sống hay trong văn học.
- Nhập đề bằng danh ngôn.


- Nhập đề bằng thơ.


- Nhập đề bằng lời bài hát.
- ….


<b>2.2. Kết bài </b>



<i><b>* Thế nào là một kết bài hay ?</b></i>


- Là kết bài đúng : thể hiện đúng quan điểm đã trình bày ở phần thân bài; chỉ
nêu những ý khái qt, có tính tổng kết, khơng lan man, lặp lại những gì đã trình
bày.


- Là kết bài độc đáo, sáng tạo, tự nhiên và để lại dư vị.
<i><b>* Một số “mẹo” kết bài hay :</b></i>


</div>
<span class='text_page_counter'>(118)</span><div class='page_container' data-page=118>

- Kết bài bằng danh ngơn, hoặc câu nói có tính triết lí.


<b>3. Cách triển khai phần thân bài đối với từng kiểu bài nghị luận xa hội. </b>
<b>3.1. Nghị luận về một tư tưởng, đạo lý</b>


<i><b>* Cách tư duy và xác định luận điểm:</b></i>
+ Tư tưởng, đạo lý ấy là gì ?


+ Vì sao tư tưởng, đạo lý ấy lại như thế ?


+ Nó biểu hiện trong đời sống và văn học thế nào ?


+ Nó có ý nghĩa gì với cuộc sống, con người và bản thân anh (chị) ?
<i><b>* Các bước triển khai luận điểm, luận cứ :</b></i>


<i><b>+ Bước 1 : Giải thích</b></i>


<i>Có 3 cấp độ giải thích : </i>


<i>- Giải thích từ ngữ, hình ảnh, khái niệm.</i>



- Giải thích các cụm từ, các vế trong câu.
- Giải nghĩa của cả câu.


<i>+ Bước 2 : Bàn luận (Phân tích, lý giải)</i>


- Bộc lộ ý kiến về câu nói : đúng - sai, hợp lý - chưa hợp lý, hoàn toàn
đúng – đúng một phần...


<b>- Đưa ra các lí lẽ và dẫn chứng để làm sáng tỏ vấn đề NL theo quan</b>
điểm đánh giá của người viết (tự đặt ra và tìm các ý trả lời cho câu hỏi Vì
sao?)


</div>
<span class='text_page_counter'>(119)</span><div class='page_container' data-page=119>

- Đánh giá vấn đề được đưa ra bàn luận đã là bài học xử
thế hay chưa, nó có giá trị như thế nào trong việc hình thành nhân cách
của con người và sự tiến bộ của xã hội.


- Phản đề: nêu những hiện tượng trái chiều; đặt vấn đề
vào những tình huống phức tạp của cuộc sống để bàn luận với cái nhìn
nhiều chiều, thậm chí lật ngược vấn đề.


<i>+ Bước 4: Bài học nhận thức và hành động</i>


<b>Ví dụ minh họa</b>


<i>Đề: Đại văn hào người Nga M.Goorki từng tâm niệm: Nơi lạnh nhất không</i>


<i>phải là Bắc Cực mà là nơi khơng có tình thương.</i>


<i><b>Suy nghĩ của anh (chị) về nhận định trên. (Đề HSG lớp 9 – năm 2012-2013)</b></i>



<i><b>1. Giải thích</b></i>


- Bắc Cực nằm ở cực Bắc của trái đất, quanh năm băng tuyết bao phủ. Sự
sống ở nơi đây thật khó khăn, khắc nghiệt. Cái lạnh nơi Bắc Cực là sự giá lạnh của
thời tiết, của thiên nhiên do vị trí địa lí gây ra. Cái lạnh ở Bắc Cực không ngăn cản
được sự sống của sự vật và niềm say mê khám phá những vùng đất lạ của con
người.


- Tình thương là tình cảm yêu thương, sự chia ngọt, sẻ bùi tự nguyện giữa
con người với con người trong cuộc sống. Nơi thiếu tình thương là nơi giữa con
người và con người không tồn tại tình người, khơng có sự cảm thơng, thấu hiểu và
chia sẻ. Cái lạnh ở nơi khơng có tình thương là cái lạnh trong lòng người, là sự
băng giá của trái tim.


</div>
<span class='text_page_counter'>(120)</span><div class='page_container' data-page=120>

lạnh hơn ở Bắc Cực. Cách so sánh giàu ý nghĩa biểu cảm cho chúng ta thấy được
<b>tầm quan trọng và ý nghĩa của tình thương trong cuộc sống.</b>


<i><b>2. Luận bàn về câu nói</b></i>


- Đây là một nhận định hồn tồn đúng đắn.


- Tình thương chính là sự đồng cảm, sẻ chia với mọi nỗi khổ đau, bất hạnh
của con người. Nhờ có tình thương, con người biết quan tâm, giúp đỡ người khác,
nhờ có tình thương con người sống gần gũi với nhau hơn. <i>Tình thương sẽ cứu</i>
<i>chuộc thế giới. ( First new )… ( Dẫn chứng minh họa).</i>


<i>-</i> Nếu khơng có tình thương, con người sẽ trở nên lạnh lùng, thờ ơ, vô cảm
trước nỗi khổ đau của người khác, khi đó cuộc sống con người trở nên lạnh giá hơn ở
Bắc Cực. Con người sẽ thu mình trong vỏ bọc cơ đơn, sẽ khơng có gia đình, khơng


có cộng đồng, khơng có nhân loại, khơng có sự sống…( Dẫn chứng minh họa).


<i><b>3. Mở rộng, nâng cao </b></i>


- Khẳng định câu nói của M. Goorki là bài học cuộc sống sâu sắc, có ý nghĩa
với mọi thời đại. Con người ta khơng thể sống mà thiếu tình thương.


- Trong cuộc sống hiện đại càng cần đến tình thương, sự đồng cảm và chia
sẻ. Những biểu hiện của tình thương giữa con người và con người trong cuộc sống
hôm nay: Xây dựng những mơi trường thân thiện, nhân ái, chung tay góp sức ủng
hộ quỹ vì người nghèo, xây dựng nhà tình nghĩa, những ngôi nhà mơ ước...


- Phê phán những người sống thiếu tình thương, khơng biết đồng cảm, sẻ
chia với đau khổ, bất hạnh của con người.


<i><b>4. Bài học nhận thức và hành động</b></i>


</div>
<span class='text_page_counter'>(121)</span><div class='page_container' data-page=121>

<i><b>- Cần sống nhân hậu, yêu thương mọi người để cuộc sống thêm ý nghĩa.</b></i>
<b>3.2. Nghị luận về một hiện tượng đời sống.</b>


<b>* Cách tư duy và xác định luận điểm:</b>


+ Thực trạng/hiện tượng đời sống ấy cụ thể thế nào ?


+ Nguyên nhân, hậu quả (kết quả) của thực trạng/hiện tượng ấy ?
+ Biện pháp nào để khắc phục ?


<b>* Các bước triển khai luận điểm, luận cứ:</b>


<i>+ Bước 1: Nêu khái niệm hoặc nhận thức về hiện tượng.</i>


<i>+Bước 2: Thực trạng của hiện tượng đời sống, bao gồm :</i>


+ Các biểu hiện
+ Các dạng tồn tại
+ Các số liệu


<i>+ Bước 3: Phân tích, bình luận nguyên nhân: chủ quan, khách quan</i>


<i>+ Bước 4: Phân tích, bình luận kết quả (hậu quả). Hậu quả : cần xem xét ở</i>
các khía cạnh cá nhân – cộng đồng; hiện tại – tương lai…


<i>+ Bước 5: Đề xuất giải pháp. Giải pháp nên bắt nguồn từ nguyên nhân, căn</i>
cứ vào nguyên nhân mà xác định giải pháp.


<i><b>+ Bước 6: Bài học nhận thức và hành động của bản thân</b></i>
<b>Ví dụ minh họa</b>


</div>
<span class='text_page_counter'>(122)</span><div class='page_container' data-page=122>

<i>- Bạo lực học đường là những hành vi thô bạo, tàn nhẫn, bất chấp công lý,</i>


đạo lý, xúc phạm, trấn áp người khác gây nên những tổn thương cho con người
trong phạm vi trường học.


<i>- Bạo lực học đường diễn ra dưới nhiều hình thức: bạo lực thể xác và bạo lực</i>
tinh thần.


<i><b>2. Thực trạng: </b></i>


- Bạo lực học đường hiện nay có xu hướng gia tăng nhanh chóng, phát triển
phức tạp, diễn ra ở nhiều nơi, do đó đang trở thành một vấn nạn của xã hội.



- Bạo lực học đường diễn ra dưới nhiều biểu hiện phức tạp:


+ Xúc phạm, lăng mạ, sỉ nhục, chà đạp nhân phẩm, làm tổn thương về mặt
tinh thần con người thơng qua lời nói.


+ Đánh đập, hành hạ, làm tổn hại về sức khỏe, xâm phạm cơ thể con người
thông qua những hành vi bạo lực.


+ Một bộ phận thanh niên coi đó là thú vui…
<i><b>3. Hậu quả:</b></i>


- Với nạn nhân: tổn thương về thể xác, tinh thần, gây tâm lý nặng nề, ảnh
hưởng đến cuộc sống, học tập.


- Làm biến thái môi trường giáo dục.


- Với xã hội: tạo tâm lý bất ổn, lo lắng, hoang mang.


- Với người gây ra hành vi bạo lực: con người phát triển khơng tồn diện;
mầm mống của tội ác; làm hỏng tương lai của chính mình; bị mọi người lên án, xa
lánh, căm ghét.


</div>
<span class='text_page_counter'>(123)</span><div class='page_container' data-page=123>

- Sự phát triển thiếu toàn diện, thiếu hụt về nhân cách, thiếu khả năng kiểm
soát hành vi ứng xử của bản thân, thiếu kỹ năng sống, sai lệch trong quan điểm
sống.


- Có những căn bệnh tâm lý.


- Do ảnh hưởng của môi trường văn hóa bạo lực từ cuộc sống và phim ảnh.
- Thiếu sự quan tâm của gia đình.



- Sự giáo dục trong nhà trường: nặng về dạy kiến thức văn hóa, chưa thật
chú trọng dạy kỹ năng sống cho học sinh.


- Xã hội chưa có sự quan tâm đúng mức, chưa có những giải pháp thiết thực,
đồng bộ, triệt để.


<i><b>5. Giải pháp:</b></i>


- Xã hội cần có những giải pháp đồng bộ. Cần phối hợp chặt chẽ giữa gia
đình, nhà trường và xã hội trong việc giáo dục học sinh.


- Tăng cường giáo dục đạo đức, dạy kỹ năng sống, vươn tới những giá trị
chân, thiện, mỹ.


- Có những biện pháp quyết liệt để giáo dục, răn đe, làm gương cho người
khác.


<i><b>6. Bài học nhận thức và hành động: </b></i>


- Có quan điểm nhận thức, hành động đúng đắn, hình thành những quan
niệm sống tốt đẹp.


- Đấu tranh, tố cáo những hành vi bạo lực học đường.


</div>
<span class='text_page_counter'>(124)</span><div class='page_container' data-page=124>

<i><b>* Cách tư duy và xác định luận điểm:</b></i>


+ Vấn đề xã hội đặt ra trong tác phẩm/đoạn tin là gì ?


+ Nó biểu hiện cụ thể thế nào trong tác phẩm/đoạn tin ấy và trong đời sống ?


+ Ý nghĩa mà nó đặt ra đối với đời sống nói chung và mỗi người nói riêng.
<i><b>* Cách triển khai luận điểm, luận cứ:</b></i>


- Tùy theo vấn đề xã hội đặt ra trong tác phẩm là vấn đề đạo lý hay hiện tượng
xã hội mà có cách triển khai tương ứng. (Xem ở phần trước)


<b>Ví dụ minh họa 1</b>
<b>Người đi săn và con vượn</b>


<i>Có một người rất tài săn bắn. Nếu con thú rừng nào không may gặp bác ta</i>
<i>thì hơm ấy coi như ngày tận số.</i>


<i>Một hôm, người đi săn xách nỏ vào rừng. Bác thấy một con vượn lông xám</i>
<i>đang ngồi ôm con trên tảng đá. Bác nhẹ nhàng rút mũi tên bắn trúng tim con</i>
<i>vượn mẹ.</i>


<i>Vượn mẹ giật mình, hết nhìn mũi tên lại nhìn về phía người đi săn bằng đơi</i>
<i>mắt căm giận, tay không rời con. Máu ở vết thương từ từ rỉ ra loang khắp mũi</i>
<i>tên.</i>


<i>Người đi săn đứng im chờ kết quả...</i>


<i>Bỗng vượn mẹ nhẹ nhàng đặt con nằm xuống, vơ vội nắm bùi nhùi gối lên</i>
<i>đầu con, rồi nó hái cái lá to, vắt sữa vào và đặt lên miệng con. Sau đó, vượn</i>
<i>mẹ nghiến răng giật phắt mũi tên ra, rú lên một tiếng ghê rợn rồi từ từ gục</i>
<i>xuống.</i>


<i>Người đi săn đứng lặng. Hai giọt nước mắt từ từ lăn trên má. Bác mắm môi</i>
<i>bẻ gãy cung nỏ và lẳng lặng quay gót ra về.</i>



<i>Từ đấy, bác không bao giờ đi săn nữa. </i>


</div>
<span class='text_page_counter'>(125)</span><div class='page_container' data-page=125>

Câu chuyện trên gợi cho anh (chị) những suy nghĩ gì?


<i><b>1.</b></i> <i><b>Vấn đề cốt lõi đặt ra trong câu chuyện của Lep Tôn- xtôi là: </b></i>
- Sự thiêng liêng và sức mạnh kì diệu của tình mẫu tử:


+ Vượn mẹ trước khi chết vẫn một lòng lo cho sự sống của con: “tay
không rời con”, “nhẹ nhàng đặt con nằm xuống”, “ vơ vội nắm bùi nhùi
gối lên đầu con”, “hái lá to” “vắt sữa” “để lên miệng con” cho dù “máu ở vết
thương từ từ rỉ ra loang khắp mũi tên”.


+ Chính tình mẫu tử cao đẹp đã làm thay đổi tình cảm, nhận thức của
nhân vật bác thợ săn: “đứng lặng”, “giọt nước mắt từ từ lăn trên má” “bẻ gãy
cung nỏ” “quay gót ra về”. Tình thương con, sự quên mình của vượn mẹ đã
cho người thợ săn một bài học sâu sắc.


- Hành động lỗi lầm và sự hối hận, phục thiện của con người: bác thợ săn
“không bao giờ đi săn nữa”. Kết thúc câu chuyện, tác giả gợi mở một niềm tin,
<i>niềm hi vọng vào sự hướng thiện của con người. (ý này chỉ là nền tảng để từ đó</i>


<i>học sinh trình bày suy nghĩ của mình, do đó cần viết ngắn gọn)</i>


<i><b>2.</b></i> <i><b>Từ vấn đề mà nhà văn đặt ra trong tác phẩm, câu chuyện gợi mở nhiều</b></i>
<i><b>suy nghĩ về các vấn đề trong cuộc sống: </b></i>


- Tình mẫu tử là tình cảm thiêng liêng và cao đẹp. Trong cuộc đời này cũng
đã có biết bao nhiêu người mẹ yêu con, hi sinh cho con đến qn mình.


- Con người có thể gây ra lỗi lầm thậm chí là những hành động tội ác nhưng


con người cũng sẽ biết thức tỉnh nếu tâm hồn được thanh lọc, soi sáng bởi những
tình cảm cao đẹp đầy nhân tính.


- Mối quan hệ và cách ứng xử của con người với môi trường tự nhiên. Phải
chăng sự vơ tình của con người đã gây ra những hậu quả đau lịng? Con người cần
có nhận thức và hành động như thế nào để bảo vệ môi trường tự nhiên và cũng là
môi trường sống của chính mình?


</div>
<span class='text_page_counter'>(126)</span><div class='page_container' data-page=126>

<b>Ví dụ minh họa 2 (Đề HSG lớp 9 tỉnh Vĩnh Phúc năm 2011-2013)</b>
<b>Đọc đoạn tin sau:</b>


<i>Cô là người con gái thứ 20 trong một gia đình có 22 người con. Cơ sinh</i>
<i>thiếu tháng nên mọi người nghĩ cơ khó mà sống được.</i>


<i> Nhưng cơ vẫn sống khỏe mạnh. Năm lên 4 tuổi, cô bị viêm phổi và sốt phát</i>
<i>ban. Sau trận ốm đó, cơ bị liệt chân trái và phải chống gậy khi di chuyển. Năm 9</i>
<i>tuổi, cô bỏ gậy và bắt đầu tự đi. Đến năm 13 tuổi cơ đã có thể đi lại một cách bình</i>
<i>thường và cơ quyết định trở thành một vận động viên điền kinh. Cô tham gia vào</i>
<i>một cuộc thi chạy và về cuối cùng. Những năm sau đó cơ đều tham dự tất cả các</i>
<i>cuộc thi điền kinh, nhưng cũng đều về cuối. Mọi người nói cơ nên từ bỏ nhưng cô</i>
<i>vẫn tiếp tục theo đuổi ước mơ trở thành một vận động viên điền kinh. Và rồi cơ đã</i>
<i>chiến thắng trong một cuộc thi. Từ đó trở đi cô luôn chiến thắng trong tất cả các</i>
<i>cuộc thi mà cơ tham gia. Sau đó cơ đã giành được ba huy chương vàng Olimpic.</i>
<i>Cô là Wilma Rudolph. (Wilma Rudolph là nữ vận động viên người Mỹ).</i>


Em có suy nghĩ gì từ ý nghĩa đoạn tin trên?
<i><b>1. Giải thích ý nghĩa của đoạn tin.</b></i>


- Đoạn tin là một câu chuyện kì diệu về một nữ vận động viên nổi tiếng của Mỹ có
tên là Wilma Rudolph. Từ một đứa trẻ kém may mắn: sức khỏe yếu vì sinh thiếu


tháng, lên 4 tuổi bị liệt chân trái vì bệnh tật, Wilma Rudolph đã kiên trì tập luyện
để có thể đi lại bình thường. Lên 9 tuổi cơ đã đi lại được và có ước mơ trở thành
vận động viên điền kinh. Sau nhiều lần thất bại (về cuối trong các cuộc thi) cô vẫn
không nản lịng. Sau nhiều năm cố gắng cơ đã chiến thắng và giành được ba huy
chương vàng Olimpic.


</div>
<span class='text_page_counter'>(127)</span><div class='page_container' data-page=127>

bản thân khơng chỉ để trở thành con người bình thường mà còn trở thành con người
xuất chúng.


<i><b>2. Bàn luận, mở rộng vấn đề.</b></i>


- Trong cuộc sống, có khơng ít người gặp phải hoàn cảnh bất hạnh (do bẩm sinh,
do tai nạn, bệnh tật…). Nhiều người trong số đó đã vươn lên khơng ngừng, tự
khẳng định mình “tàn nhưng khơng phế”.


- Câu chuyện của Wilma Rudolph và nhiều người khác gợi suy nghĩ:


+ Sự khâm phục, ngưỡng mộ với những con người giàu ý chí, nghị lực
trong cuộc sống.


+ Khơng có khó khăn nào mà con người khơng thể vượt qua, điều quan
trọng là cần phải có ý chí nghị lực, có hồi bão ước mơ, có tình u với cuộc sống.


- Trách nhiệm của mỗi người và toàn xã hội với họ:


+ Cảm thông, tôn trọng chứ không xa lánh, ghẻ lạnh họ.
+ Động viên, giúp đỡ, tạo điều kiện để họ phát huy khả năng.


- Phê phán một bộ phận không nhỏ (nhất là thanh niên) sống khơng có nghị lực, ý
chí, ước mơ hồi bão.



<i><b>3. Liên hệ bản thân và rút ra bài học.</b></i>


<b>III. HƯỚNG DẪN HỌC SINH ÔN LUYỆN KIỂU BÀI NGHỊ LUẬN XÃ HỘI.</b>


<b>1. Mợt sớ chủ đề ơn lụn, tích lũy vớn sớng</b>
<b>1.1. Về tư tưởng, đạo lý</b>


- Chủ đề 1 : Thành công và thất bại
- Chủ đề 2 : Tài và đức


</div>
<span class='text_page_counter'>(128)</span><div class='page_container' data-page=128>

- Chủ đề 4 : Tình bạn và tình yêu
- Chủ đề 5 : Học và tự học


- Chủ đề 6 : Lí tưởng và lối sống
- Chủ đề 7 : Tiền bạc và hạnh phúc
- Chủ đề 8 : Nghề nghiệp và con người
- Chủ đề 9 : Ước mơ và thực tế


- Chủ để 10 : Cá nhân và tập thể
<b>1.2. Về hiện tượng đời sống</b>


- Chủ đề 1 : Thiên nhiên, môi trường
- Chủ đề 2 : Giao thông


- Chủ đề 3 : Văn hóa giao tiếp, ứng xử
- Chủ đề 4 : Văn hóa ăn mặc


- Chủ đề 5 : Văn hóa đọc
- Chủ đề 6 : Bệnh vơ cảm



- Chủ đề 7 : Những tấm gương vượt lên số phận
- Chủ đề 8 : Những “tấm lòng vàng”


- Chủ đề 9 : Những anh hùng lao động, những tấm gương hi sinh vì cộng đồng…
- Chủ để 10 : Những người trẻ tài năng


- Chủ đề 11 : Con đường vào đời


</div>
<span class='text_page_counter'>(129)</span><div class='page_container' data-page=129>

<b>2. Phương pháp ôn luyện. </b>


- Thường xuyên tập viết các bài nghị luận xã hội.


- Học tập cách viết văn nghị luận xã hội của các cây bút lớn như Hồ Chí Minh,
Trường Chinh, Phạm Văn Đồng, Tố Hữu, Nguyễn Đình Thi, Hữu Thọ.


- Tham khảo bài viết của các bạn học sinh giỏi, nhất là những bạn đạt giải quốc
gia.


- Tìm đọc và sưu tầm các bài nghị luận xã hội trên sách báo.


- Thường xuyên cập nhật và ghi chép vào sổ tay các thông tin về những vấn đề
nghị luận.


- Ghi lại những câu danh ngơn hay và học thuộc.
<b>3. Góp ý về cách chọn đề luyện tập</b>


- Đề mẫu hoặc đề luyện phải có tính vấn đề, gây được khơng khí tranh luận. VD :


<i>+ Học ở trường và học trong cuộc sống, cách học nào quan trọng</i>


<i>hơn ? </i>


<i>+ Sành điệu có phải là hư hỏng ? </i>


<i>+ Game online tốn thời gian và vô bổ, bạn nghĩ sao ? </i>


<i>+ Phải chăng người Việt trẻ hiện nay khơng có lịng u nước ? </i>
<i>+ Chỉ có tiền tài và địa vị mới có hạnh phúc ?</i>


<i>- Đề mẫu hoặc đề luyện phải có tính thời sự, thậm chí cập nhật trong những</i>


khoảng thời gian gần nhất. VD :


</div>
<span class='text_page_counter'>(130)</span><div class='page_container' data-page=130>

<i>của mình để giúp đỡ cho những số phận tật nguyền khác (Tuổi Trẻ, ngày </i>
30-11-2011). "Hai câu chuyện trên gợi cho anh/chị suy nghĩ gì về lối sống của một trái
tim không tật nguyền?..."


+ Hay đề bàn về “ th ầ n tư ợ ng” .


- Đề mẫu hay đề luyện phải khơi gợi được sự tị mị, kích thích sự tìm hiểu của HS
à muốn thế đề phải có tính lạ hóa và gây khó khăn một chút cho HS. VD :


<i>- Một bà mẹ khuyên con : con ơi con phải luôn ghi nhớ : tay trái của</i>


<i>người là tay phải của mình. </i>


<i>- Nhạc sĩ Pháp là Gu-nơ có lần nói : Hồi tơi 20 tuổi tơi chỉ thừa nhận</i>
<i>tơi có tài. 30 tuổi, tơi nói : “Tơi và Mơ-da”. 40 tuổi, tơi đã nói : “Mơ-da và tơi”.</i>
<i>Cịn bây giờ, tơi chỉ nói : “Mơ-da” </i>



<i><b>- Hay: Đọc truyện Ba câu hỏi sau và viết bài theo yêu cầu : </b></i>


“Ngày nọ có một người đến gặp nhà triết học Sơ-cơ-rát và nói: Ơng có
muốn biết những gì tơi mới nghe được về người bạn của ông không ? – Chờ một
chút, Sô-cơ-rat trả lời – Trước khi kể về bạn tôi, anh hãy trả lời tơi ba câu hỏi. Thứ
nhất, anh có chắc chắn hoàn toàn rằng những điều anh sắp kể là đúng sự thật
không ? Ồ không – người kia nói – thật ra tơi chỉ nghe nói về điều đó thơi.
-Được rồi – Sơ-cơ-rat nói tiếp – Bây giờ là điều thứ 2 : có phải điều anh sắp nói là
những điều tốt đẹp về bạn tơi khơng ? – Không, mà ngược lại là… - Thế à,
Sô-cơ-rát tiếp tục câu hỏi cuối cùng : Tất cả những điều sắp nói về bạn tơi thật sự cần
thiết cho tơi chứ ? – Khơng. Cũng khơng hồn tồn như vậy. – Vậy đấy, -
Sô-cơ-rat quay sang người khách và nói : “………”.


</div>
<span class='text_page_counter'>(131)</span><div class='page_container' data-page=131>

- Cần đưa các vấn đề NLXH vào những tình huống của đời sống để các em
thấy nếu thiếu hiểu biết và kĩ năng nghị luận trong những trường hợp như thế thì sẽ
thiệt thịi, kém cỏi, mất thể diện hay bị chê cười thế nào. Vd :


+ Sau khi học tập căng thẳng, em xin bố mẹ đi chơi game trong giây
<i>lát nhưng bố mẹ khơng cho vì nghĩ rằng Game online tốn thời gian và vơ bổ, lúc</i>
đó nếu khơng có khả năng “nghị luận” về vấn đề này thì mãi mãi khơng bao giờ có
cơ hội được chơi game.


+ Hay khi ăn mặc sành điệu, em bị người nhà và người khác cho là hư
hỏng trong khi bạn khơng hư hỏng. Lúc đó nếu khơng “nghị luận” cho họ hiểu thì
bạn sẽ ln mang tiếng là khơng ngoan.


- Nên ứng dụng CNTT trong một số trường hợp để thu hút sự chú ý, kích thích
hứng thú của HS. Chẳng hạn : Cung cấp thêm tư liệu ảnh, các video clip để HS
hình dung rõ hơn về vấn đề nghị luận (VD : tai nạn giao thông, ô nhiễm mơi
trường, lịng nhân ái…). Hoặc sử dụng chính những hình ảnh, băng hình, clip trên


Web, Facebook, Youtube để minh họa hay làm đề bài.


<b>4. Hệ thống đề và đáp án gợi ý ôn luyện các dạng bài nghị luận xa hội thường</b>
<b>gặp.</b>


<b>Đề 1</b><i><b> : R.Ta-go, nhà thơ Ấn Độ cho rằng: Thà làm một bông hoa sen nở khi</b></i>


<i>thấy mặt trời rồi mất hết tinh nhụy, cịn hơn giữ ngun hình nụ búp trong sương</i>
<i>lạnh vĩnh cửu của mùa đông.</i>


Suy nghĩ của anh (chị) về nhận định trên.


<b>1. Giải thích ý nghĩa của lời nhận định</b>


</div>
<span class='text_page_counter'>(132)</span><div class='page_container' data-page=132>

<i>- Mặt trời: Đó là ánh sáng vĩnh cửu đem lại sự sống cho vạn vật. Mặt trời</i>
tượng trưng cho sức sống mạnh mẽ, sự huy hoàng.


<i>- Nụ búp: ẩn dụ cho cái non nớt, nhút nhát, e sợ của con người. </i>


<i>- Sương lạnh vĩnh cửu: là mơi trường lạnh giá, khắc nghiệt, ở đó vạn vật</i>


phải ẩn mình, thu mình, khơng thể sinh sơi phát triển. Vì thế nó tượng trưng cho
những khó khăn, thử thách trong cuộc sống.


<i>=> Ý nghĩa câu nói: Ý kiến của Ta- go là một triết lí sống mạnh mẽ, tích cực</i>
và tiến bộ. Trong cuộc sống có rất nhiều khó khăn thử thách, nhưng nếu biết sống
và cống hiến hết mình ta sẽ nhận được thành quả xứng đáng. Nếu sống nhút nhát,
thụ động thì cuộc đời thật nhạt nhẽo, vô nghĩa.


<b>2. Bàn luận, mở rộng vấn đề</b>



<i>a. Tại sao nên chọn cách sống như “bông hoa sen”?</i>


- Cuộc sống rất quý giá nhưng lại ngắn ngủi, và chỉ đến duy nhất một lần. Ta
phải sống thế nào cho xứng đáng, phải sống sao cho khỏi xót xa, ân hận vì những
tháng năm đã sống hồi, sống phí. Ta cần có một trái tim đầy nhiệt huyết để sống
hết mình, để cảm nhận từng hơi thở trong khoảnh khắc của đời mình.


- Đã là con người thì cần phải có ước mơ, lý tưởng và khát khao thực hiện
những điều đó. Tuy nhiên, cuộc sống ln ẩn chứa những khó khăn, thử thách và
những điều tốt đẹp khơng bỗng dưng mà có. Thay vì để khó khăn đánh bại, ta hãy
đón nhận chúng như một cơ hội để rèn luyện bản lĩnh của chính mình, để ta thêm
trưởng thành.


</div>
<span class='text_page_counter'>(133)</span><div class='page_container' data-page=133>

sống mang lại. Đó cũng chính là cách khiến cuộc sống của ta thêm ý nghĩa và trở
nên có ích. Đó mới là cuộc sống đích thực của con người.


<i>b. Tại sao không nên chọn cách sống như “nụ búp”?</i>


<b>- Nếu ta không dám đối mặt trước những khó khăn, thử thách của cuộc sống</b>
vì ta sợ sai lầm, sợ sẽ thất bại, sợ bị cười chê… để rồi mãi mãi ta sống trong vỏ bọc
hèn nhát của mình. Đó là lối sống mịn, sống thừa, sống vơ ích mà khơng được ai
biết đến. Một “cuộc sống đang mòn ra, đang rỉ đi, đang nổi váng.”


- Cuộc sống khơng mục tiêu, ước mơ, hồi bão thật vơ vị. Sống như thế thực
chất chỉ là tồn sự tại mà thôi, là chết ngay cả khi đang sống.


<i>c. Nâng cao</i>


- Liệu có phải lúc nào ta cũng sống hết mình? Nếu cứ hết mình như thế sẽ có


lúc ta kiệt sức. Vậy ta cần phải biết lượng sức mình, khơng phải lúc nào cũng nên
lao về phía trước. Để đối mặt với mọi thử thách trên đường đời trước tiên ta phải
trân trọng chính bản thân ta. Đừng nơn nóng theo đuổi mục đích mà qn mất bản
thân mình.


- Có những phút giây ta nên thu mình lại khi đã cảm thấy mỏi mệt. Khi ấy
không phải ta đang hèn nhát, chỉ là ta đang tìm kiếm chút bình n cho tâm hồn,
tìm được lại ý chí, lịng quyết tâm để tiếp tục tiến lên phía trước.


<b>3. Bài học nhận thức và hành động</b>


- Phê phán lối sống yếu mền, thụ động, chỉ biết ngồi chờ vận may và sự
thuận lợi.


</div>
<span class='text_page_counter'>(134)</span><div class='page_container' data-page=134>

<i><b>Đề 2: Có người cho rằng: Ta hãy học theo cách của dịng sơng, nhìn thấy</b></i>


<i>núi thì đi đường vịng, nhưng người khác lại quan niệm: Trong rừng có nhiều lối</i>
<i>đi, ta chọn lối đi khơng có dấu chân người.</i>


Anh/chị có suy nghĩ gì về những ý kiến trên?
<b>1. Giải thích ý kiến: (2,0 điểm)</b>


<i>- Dịng sơng khi gặp núi thì đi đường vịng: khi gặp khó khăn, trở ngại thì</i>
<i>nên tìm hướng đi khác dễ dàng hơn, nhưng phải mất thời gian hơn. Học theo cách</i>


<i>của dịng sơng: là học kinh nghiệm, là kế thừa những cách thức đi đến thành công</i>


của người đi trước.


<i>- Chọn lối đi khơng có dấu chân người: là tìm lối đi mới, là sáng tạo và dũng</i>


cảm, mạo hiểm khi đối đầu với thử thách.


=> Bằng cách nói hình ảnh, hai ý kiến nêu lên những bài học về lẽ sống: một
cách sống tiếp thu kinh nghiệm, kế thừa, một cách sống sáng tạo, dũng cảm, mạo
hiểm.


<b>2. Bình luận: (4,0 điểm)</b>


Đúng là trong cuộc sống, có lúc ta phải học theo cách của dịng sơng, có lúc
ta phải chọn lối đi "khơng có dấu chân người": (0,5 điểm)


- Gặp khó khăn lớn, vượt q khả năng của mình, cứ đâm đầu vào đá ta sẽ
chuốc lấy thất bại. Còn học theo cách của người đi trước dù mất thêm thời gian,
<i>cơng sức nhưng đến đích một cách an toàn (dẫn chứng chứng minh). (1,5 điểm)</i>


</div>
<span class='text_page_counter'>(135)</span><div class='page_container' data-page=135>

<i>bại cũng là bài học quý cho thành công tiếp theo (dẫn chứng chứng minh). (1,5</i>
điểm).


=> Hai ý kiến không hề đối lập mà chỉ là những cách thức khác nhau để
giúp chúng ta đạt được thành công trong cuộc sống (0,5 điểm).


<b>3. Liên hệ bản thân và rút ra bài học: (2,0 điểm)</b>


- Liên hệ bản thân: những trải nghiệm của chính bản thân (1,0 điểm).


- Rút ra bài học: Trong cuộc sống ta phải linh hoạt, mềm dẻo: có lúc ta nên
học tập, kế thừa người đi trước, nhưng có lúc cần sáng tạo, dũng cảm mạo hiểm.
Nếu thế sẽ biến ước mơ thành hiện thực. Tuy nhiên, cẩn trọng nhưng không ỉ lại,
lười suy nghĩ; sáng tạo, mạo hiểm, dũng cảm khơng có nghĩa là liều lĩnh (1,0
điểm).



<b>Đề 3: </b>


<i><b>Đối thủ đáng sợ nhất</b></i>


<i>Khi Abraham Lincoln ra tranh cử tổng thống, một người bạn đã hỏi ông:</i>
<i>- Anh thấy mình có hy vọng gì khơng? Ai là đối thủ đáng sợ nhất của anh?</i>
<i>Và Abraham Lincoln đã đưa ra một câu trả lời tuy hài hước nhưng rất thật:</i>
<i>- Tơi khơng ngại Breckingridge vì ơng ta là người miền Nam nên người dân</i>
<i>miền Bắc sẽ không ủng hộ ơng ta. Tơi cũng khơng ngại Douglas vì ơng ta là người</i>
<i>miền Bắc nên người dân ở miền Nam sẽ khơng nhiệt tình bỏ phiếu cho ơng ta.</i>
<i>Nhưng có một đối thủ mà tôi rất sợ, ông ta là người duy nhất có thể khiến tơi thất</i>
<i>cử…</i>


<i>Người bạn liền vội ngắt lời:</i>
<i>- Ai vậy?</i>


</div>
<span class='text_page_counter'>(136)</span><div class='page_container' data-page=136>

<i>- Nếu lần này tơi khơng được bầu làm tổng thống thì anh hãy biết rằng đó</i>
<i>chính là lỗi của ơng ta. Ơng ta chính là Abraham Lincoln!</i>


<i><b> (Những tấm lòng cao cả - NXB Trẻ, 2004 - trang 76)</b></i>


Câu chuyện trên gợi cho anh/chị suy nghĩ gì?
<i><b>1. Nhận thức về nội dung, ý nghĩa của câu chuyện.</b></i>


Thí sinh cần đọc hiểu văn bản, từ đó rút ra nội dung ý nghĩa của câu chuyện.
Mỗi người có thể tìm thấy ở câu chuyện đó một hay nhiều ý nghĩa khác nhau
nhưng phải trên cơ sở hợp lí và có sức thuyết phục. Tuy nhiên, cần làm nổi bật
<i><b>được ý nghĩa cơ bản: Đối thủ đáng sợ nhất của mỗi người khơng phải ai khác mà</b></i>
chính là bản thân mình. Vượt qua, chiến thắng được chính mình là điều khó khăn


<i>nhất và cũng là “chiến thắng hiển hách nhất”(Flatơng).</i>


<i><b>2. Phát biểu suy nghĩ của người viết về ý nghĩa của câu chuyện.</b></i>


- Câu chuyện đã cho chúng ta bài học, lời khuyên rất đúng đắn và sâu sắc: Ý
<i>nghĩa của câu chuyện có gặp gỡ với một lời răn trong Kinh Phật “Kẻ thù lớn nhất</i>


<i>của đời người là chính mình” </i>


- Vì sao đối thủ của mỗi người là chính mình?


<i>+ Con người sinh ra trên đời khơng có ai hồn thiện (Bản thân từ “con người” đã</i>
nói lên điều này). Nhưng bản chất của con người là hướng thiện, ln muốn tự
hồn thiện mình, vì thế phải đấu tranh, đấu tranh để chế ngự bản thân, chế ngự
phần bản năng, phần xấu trong con người mình.


+ Sống ở trên đời mỗi người có một hồn cảnh, một điều kiện, một số phận. Có
những số phận may mắn, cũng có những số phận thiệt thịi… Con người muốn tồn
tại, muốn sống cho ra con người thì càng phải đấu tranh để vượt lên chính mình.
(Học sinh lấy dẫn chứng chứng minh).


</div>
<span class='text_page_counter'>(137)</span><div class='page_container' data-page=137>

<i> Cuộc sống là một “trường tranh đấu”, con người phải đấu tranh với những thế</i>
lực bên ngoài (thiên nhiên, các thế lực thù địch, các đối thủ…) và đấu tranh với
bản thân (như đã nói trên) nhưng tính chất của hai cuộc đấu tranh này không giống
nhau:


+ Khi cuộc sống đặt ai đó trong một cuộc tranh giành (tranh giành sức mạnh, tranh
tài…) thì cuộc đấu ấy có đối thủ rõ ràng, có thế trận, có tương quan lực lượng bày
ra trước mắt mọi người. Mỗi đối thủ trong cuộc chơi phải vận động tất cả sức lực,
tài trí của mình để giành chiến thắng.



+ Cịn cuộc đấu với bản thân? Nó âm thầm, lặng lẽ, một mình mình biết, một
mình mình hay…Thêm nữa, trong cuộc sống, con người ta thường nghiêm khắc,
xét nét người khác nhưng lại dễ dãi với bản thân, hay thỏa hiệp, khoan nhượng với
<i>mình… Vì thế dễ đi đến thất bại. (Có thể dẫn ra những câu chuyện, những ví dụ về</i>


<i>những con người khơng vượt qua được chính mình, hoặc những trải nghiệm của</i>
<i>chính bản thân).</i>


<i><b>4.</b></i> <i><b>Liên hệ, rút ra bài học cho bản thân.</b></i>


<b>Đề 4: Suy nghĩ của anh (chị) về ý nghĩa của bài thơ sau:</b>


<i>Ví khơng có cảnh đơng tàn</i>


<i>Thì khơng có cảnh huy hoàng ngày xn</i>
<i>Nghĩ mình trong bước gian truân</i>
<i>Tai ương rèn luyện tinh thần thêm hăng.</i>


</div>
<span class='text_page_counter'>(138)</span><div class='page_container' data-page=138>

- Bài thơ đề cập đến hiện tượng tự nhiên: Nếu khơng có cảnh mùa đơng tàn thì
cũng khơng có được cảnh huy hồng của mùa xn. Đơng qua rồi mới đến xn,
đó là qui luật tất yếu của tự nhiên.


- Từ qui luật tự nhiên, bài thơ liên tưởng đến con người: Trong khó khăn gian khổ,
nếu con người chịu đựng được, vượt qua được những khó khăn thử thách thì sẽ đến
được với cảnh huy hoàng của cuộc sống.


- Những bước gian truân, tai ương gặp phải là những thử thách, rèn luyện làm cho
tinh thần thêm hăng. Bài thơ thể hiện tinh thần lạc quan của người chiến sĩ cách
mạng.



<i><b>2. Bàn luận, mở rộng vấn đề.</b></i>


- Khẳng định quan niệm của Bác trong bài thơ là hoàn toàn đúng:


+ Trong cuộc sống, khơng mấy ai khơng gặp khó khăn, gian khổ. Trước
những trở ngại không được bi quan, chán nản mà phải giữ vững niềm tin vào lí
tưởng, mục đích cuộc sống của mình.


+ Những vất vả, khó khăn gặp phải như cơn gió lạnh mùa đơng. Nếu chịu
đựng và vượt qua được mùa đơng lạnh lẽo thì sẽ được sống trong cảnh huy hồng
của ngày xn. Điều đó có nghĩa vượt qua gian khổ sẽ đến được với thành cơng.
Niềm tin đó sẽ giúp chúng ta vươn lên trong cuộc sống. Chính trong gian khổ con
người sẽ vững vàng hơn. Tai ương gặp trong cuộc đời sẽ sẽ giúp tinh thần thêm
hăng hái. HS có thể lấy dẫn chứng (cuộc đời Bác Hồ và các chiến sĩ cách mạng
trong đấu tranh là dẫn chứng hùng hồn về sự kiên trì, nhẫn nại, về quyết tâm vượt
qua những thử thách, về niềm tin và tinh thần lạc quan cách mạng).


- Phê phán những kẻ sợ khó khăn gian khổ, hay nản chí ngã lịng, bi quan trước
những khó khăn thử thách trong cuộc sống.


</div>
<span class='text_page_counter'>(139)</span><div class='page_container' data-page=139>

- Sống ở trên đời, khi đã xác định được mục đích đúng đắn, muốn đi đến thành
cơng thì phải trải qua những gian nan thử thách. Nếu vượt qua được chắc chắn sẽ
đạt được điều ta mong muốn.


- Chấp nhận đương đầu với khó khăn thử thách, đem hết khả năng của mình góp
phần vào sự nghiệp cách mạng, sự nghiệp xây dựng và đổi mới đất nước.


- Bài thơ giúp ta hiểu được qui luật tất yếu của cuộc sống, từ đó hăng hái học tập
và rèn luyện.



<b>Đề 5 : Viết một văn bản nghị luận ngắn (khoảng một trang tờ giấy thi)</b>
<i><b>bàn về vấn đề: Giờ Trái đất – hành động nhỏ, hiệu quả lớn. </b></i>


<b>1. Giải thích</b>


- Giờ Trái đất (EH), do WWF Quỹ quốc tế về Bảo tồn Thiên nhiên khởi xướng, là
một sáng kiến toàn cầu nhằm nâng cao ý thức của người dân về tiết kiệm năng
lượng và biến đổi khí hậu. Sáng kiến này kêu gọi các cá nhân và doanh nghiệp trên
toàn thế giới tắt điện một tiếng đồng hồ vào tối thứ 7 cuối cùng của tháng 3 hàng
năm. Mục tiêu của chiến dịch nhằm khẳng định mỗi một hành động cá nhân khi
được nhân lên trên diện rộng có thể giúp thay đổi môi trường sống tốt hơn.


<b>2. Thực trạng</b>


- Giờ Trái đất được tổ chức lần đầu tiên ở Sydney năm 2007, với 2.2 triệu người
tham gia bằng cách tắt tất cả các ánh sáng đèn không cần thiết. Năm 2009, Giờ
Trái đất đã lan rộng ra 88 quốc gia và Việt Nam tự hào tham gia cùng với 6 thành
phố chính thức là Hà Nội, Thành phố Hồ Chí Minh, Huế, Hội An, Nha Trang và
Cần Thơ. Giờ Trái đất năm 2010 đánh dấu một hành động tự nguyện lớn nhất vì
mơi trường trong lịch sử với sự tham gia của 128 quốc gia với gần 5.000 thành phố
trên toàn thế giới.


</div>
<span class='text_page_counter'>(140)</span><div class='page_container' data-page=140>

- Giờ Trái Đất là hành động tích cực của con người cho thấy nhận thức đúng đắn
về hiện tượng biến đổi khí hậu. Trên thực tế càng ngày chúng ta càng thấy rõ tác
động của biến đổi khí hậu đối với cuộc sống của con người. Tắt đi ánh sáng và các
thiết bị điện không cần thiết trong 1 giờ là hành động dù rất nhỏ và đơn giản cũng
góp phần tiết kiệm năng lượng và hạn chế sự nóng lên tồn cầu – là hành động bảo
vệ bà mẹ Trái Đất u q của lồi người.



- Sự lớn mạnh nhanh chóng của Giờ Trái đất trong bốn năm qua đã chứng tỏ rằng
biến đổi khí hậu khơng cịn là vấn đề của một vùng, một miền, một quốc gia mà là
vấn đề toàn cầu. Hàng trăm triệu người muốn nỗ lực hơn nữa để bảo vệ hành tinh
của chúng ta bằng hành động biểu trưng là tắt đi ánh sáng và những thiết bị không
cần thiết trong 1 giờ đồng hồ. Tham gia hưởng ứng Giờ Trái đất hàng trăm triệu
người trên khắp các châu lục muốn gửi thông điệp về sự cấp thiết phải cùng nhau
hành động chống lại biến đổi khí hậu, cùng nhau gắn kết để bảo vệ hành tinh này.
<b>3. Nhận thức và hành động. </b>


- Thông qua hành động biểu tượng tắt đèn trong một giờ đồng hồ cộng đồng trên
toàn thế giới muốn mỗi con người thay đổi hành vi của mình, có những hành động
thiết thực cùng chống lại biến đổi toàn cầu. Trước hết là hành vi sử dụng điện của
bạn ngay sau đó và được tiếp nối bằng những hành động yêu trái đất trong suốt
thời gian tiếp theo.


- Mỗi chúng ta hãy tích cực hưởng ứng và tham gia giờ trái đất. Hãy nhớ rằng môi
trường sống liên quan đến vận mệnh của mỗi con người, chúng ta hãy đối xử tử tế
với “Mẹ trái đất”.


<b>Đề 6 : Viết một bài văn nghị luận có nhan đề: CON LẬT ĐẬT</b>
<b>1. Giải thích.</b>


</div>
<span class='text_page_counter'>(141)</span><div class='page_container' data-page=141>

- Đặc điểm nổi bật của lật đật là có bộ phận giữ thăng bằng rất tốt nên dù có bị
tác động thế nào cũng ln tự mình trở lại tư thế thẳng đứng.


<b>2. Phân tích, bình ḷn.</b>


Dù chỉ là một món đồ chơi nhưng lật đật lại gợi cho chúng ta nhiều bài học sâu
sắc:



- Con lật đật ln tự mình đứng thẳng dù có bị lật qua lật lại cho ta bài học về ý
<i><b>chí mạnh mẽ, vươn lên. Điều này cũng vô cùng quan trọng bởi trong cuộc sống</b></i>
con người luôn phải đối mặt với những khó khăn, thất bại, ln mạnh mẽ, khơng
cúi đầu trước bất kì hồn cảnh nào sẽ giúp ta đi đến thành cơng.


- Con lật đật dù có bị tác động thế nào cũng luôn tự đứng vững nhờ bộ phận giữ
<i><b>thăng bằng giúp ta nhận ra trong cuộc sống này mỗi người cần rèn luyện để có bản</b></i>
<i><b>lĩnh sống vững vàng. Khi có bản lĩnh sống vững vàng con người sẽ vượt qua mọi</b></i>
cám dỗ, thử thách để luôn được là chính mình.


<i><b>(Học sinh cần kết hợp lí lẽ, dẫn chứng để làm sáng tỏ các bài học)</b></i>
<i><b>3. Liên hệ, rút ra bài học cho mọi người.</b></i>


- Hãy luôn mạnh mẽ, sẵn sàng đối diện với khó khăn thử thách, biết chấp nhận thất
bại để đi đến thành công.


- Mỗi người cần rèn luyện cho mình bản lĩnh sống vững vàng để có thể tự tin, chủ
động trong cuộc sống vốn có nhiều khó khăn, bất trắc.


- Những bài học từ chú lật đật khơng chỉ có ý nghĩa với mỗi cá nhân mà cịn có ý
nghĩa cho cả cộng đồng, dân tộc.


<b>C. KẾT LUẬN</b>


</div>
<span class='text_page_counter'>(142)</span><div class='page_container' data-page=142>

phục được trái tim người đọc thì tác phẩm có giá trị lâu bền, giá trị nhân văn sâu
sắc.


Đây là một chuyên đề có ý nghĩa thiết thực đối với cơng tác dạy học Ngữ
Văn nói chung và bồi dưỡng học sinh giỏi mơn Ngữ văn nói riêng. Trên đây là một
vài kinh nghiệm nhỏ của chúng tôi trong việc giúp học sinh tạo chất văn trong bài


văn nghị luận xã hội nhằm mang đến một bài viết hoàn thiện, đáp ứng những yêu
cầu của công tác bồi dưỡng học sinh giỏi. Chuyên đề chắc chắn không tránh khỏi
những khiếm khuyết, rất mong nhận được sự đóng góp ý kiến của các đồng
nghiệp.


<b>TƯ LIỆU THAM KHẢO</b>


1. Hạt giống tâm hồn – First News (11 tập), NXB TPHCM, năm 2006.


2. Phương pháp dạy học Ngữ văn – Phan Trọng Luận, NXB Giáo dục, 2007.
3. Phương pháp dạy học nêu vấn đề - Nguyễn Thanh Hùng, NXB Đại học sư


phạm, 2004.


</div>
<span class='text_page_counter'>(143)</span><div class='page_container' data-page=143></div>

<!--links-->

×